Câu 1. - Hoc360.net · 2019-10-19 · 1hìn vào bài toán ta có thể đoán được đạt...

155
1

Transcript of Câu 1. - Hoc360.net · 2019-10-19 · 1hìn vào bài toán ta có thể đoán được đạt...

1

2

3

Câu 1.

Cho x, y thỏa mãn x ≤ 2, x + y ≥ 2. Tìm giá trị nhỏ nhất của biểu thức :

A = 14x2 + 9y

2 + 22xy – 42x – 34xy + 35.

Phân tích.

Khi nhìn vào biểu thức cần đánh giá ta nghĩ ngay đến phương án đưa biểu thức

này về dạng tổng các bình phương, tuy nhiên việc đưa biểu thức A về dạng tổng

các bình phương có quá nhiều các lựa chọn, và việc « mò mẫm » có khả năng

dẫn tới bế tắc. Để ý từ điều kiện hai biến x, y ta thấy rằng x ≤ 2 ; y ≥ 0 do đó có

thể đổi hai biến x, y thành hai biến a, b với điều kiện a ≥ 0, b ≥ 0 (hiển nhiên ta

nghĩ ngay đến việc đặt a = 2 - x ; b = x + y – 2) thì bài toán có lẽ sẽ trở nên đơn

giản hơn. Và sau khi đổi biến như vậy cùng với việc đưa biểu thức A về tổng

các bình phương ta có thể giải bài toán đơn giản hơn.

Lời giải.

Đặt a = 2 – x , b = x + y – 2 (a, b ≥ 0) suy ra y = a + b

Ta có biểu thức A trở thành :

A = 14(2 – a)2 + 9(a + b)

2 + 22(2 – a)(a + b) – 42(2 – a) – 34(2 – a)(a + b) + 35.

= a2 + 9b

2 – 4ab – 4a + 10b + 7

= a2 + 4b

2 + 4 – 4ab – 4a + 8b + 15b

2 + 2b + 3

= (a - 2b – 2)2 + 15b

2 + 2b + 3 ≥ 3 ∀a, b

Dấu « = » xảy ra khi a – 2b – 2 = 0 và b = 0

Vậy giá trị nhỏ nhất của A là 3 khi x = 0 và y = 2

Bình luận.

Đây là bài toán có thể gây mất thời gian Cho học sinh trong quá trình mò mẫn

đưa biểu thức A về dạng tổng các bình phương và nhiều khả năng sẽ gặp bế tắc

trong việc đánh giá biểu thức. Và ngay sau đây là phương án giải quyết bài toán

bằng cách mò mẫn.

Lời giải.

Ta có : A = 14x2 + 9y

2 + 22xy – 42x – 34xy + 35.

= x2 + 9(x + y)

2 + 4x(x + y) – 8x – 34(x + y) + 35

Đặt x + y = t , t ≥ 2. Ta có :

A = x2 + 9t

2 + 4xt – 8x – 34t + 35

= (x + 2t – 2)2 + (t – 2)(5t – 8) + 3 ≥ 3 ∀x, t

bob
Typewriter
Luyện thi AMAX
bob
Rectangle

4

Dấu « = » xảy ra khi x + 2t – 2 = 0 và t = 2 .

Vậy A đạt giá trị nhỏ nhất là 3 khi x = 0 và y = 2.

Câu 2.

Cho a, b, c [0 ; 1]. Chứng minh rằng : a + b2 + c

3 – ab – bc – ca ≤ 1.

Phân tích.

Đây là dạng toán chứng minh BĐT không đối xứng kèm theo điều kiện biên của

các biến. Dựa vào điều kiện ta dễ dàng thấy ngay 1 – a ≥ 0 ; 1 – b ≥ 0 ; 1 - c ≥ 0

nhân các đại lượng 1 – a ; 1 – b ; 1 – c với nhau kèm với sự đánh giá b2 ≤ b ; c

3 ≤

c sẽ cho ta lời giải bài toán trên cụ thể lời giải bài toán trên như sau :

Lời giải.

Vì a, b, c [0 ; 1] nên 1 – a ≥ 0 ; 1 – b ≥ 0 ; 1 - c ≥ 0.

Do đó : (1 – a)(1 – b)(1 – c) ≥ 0

⇔ a + b + c – ab – bc – ca + abc ≤ 1 (1)

Vì a, b, c [0 ; 1] nên b2 ≤ b ; c

3 ≤ c ; a, b, c ≥ 0

⇒ a + b2 + c

3 – ab – bc – ca ≤ a + b + c – ab – bc – ca + abc (2)

Từ (1) và (2) ta có : a + b2 + c

3 – ab – bc – ca ≤ 1

Vậy ta có điều phải chứng minh.

Bình luận.

Đây là dạng toán khó, thường xuất hiện trong các câu phân loại trong các kì thi

lớn, kể cả thi đại học. Thường không có phương pháp chung nào để giải các bài

toán như vậy mà tùy thuộc vào sự nhạy bén của người giải. Tuy nhiên dựa vào

điều kiện của bài toán ta có thể đưa ra định hướng ban đầu cho bài toán. Cụ thể

trong bài toán này ta dùng phương pháp đánh giá điều kiện biên.

Câu 3.

Cho các dương a, b, c thỏa mãn 2a + 3b ≤ 4. Tìm giá trị nhỏ nhất của biểu thức :

Q = 2002 2017

2996 5501a ba b

.

Phân tích.

Quan sát bài toán cho 2 số dương a, b và biểu thức Q , giúp ta định hướng ngay

đến việc sử dụng BDDT Cô – si để giải toán. Dự đoán dấu « = » xảy ra khi a =

1; 1

2b . Để bảo toàn được dấu « = » khi sử dụng BĐT Cô – si, ta sẽ chọn số α ,

β sao cho

5

Lời giải.

Q = 2002 2017

2996 5501a ba b

.

2002 2017

( 8008 ) ( 2017 ) 2506(2 3 ).a b a ba b

Áp dụng BĐT Cô- si và sử dụng giả thiết 2a + 3b ≤ 4 ta có :

Q ≥ 2002 2017

2. .8008 2. .2017 2506.4a ba b

≥ 8008 + 4034 – 10024 = 2018.

Dấu « = » xảy ra khi :

20028008

12017

2017 2

12 3 4

aa

ab

bb

a b

Vậy min 2018Q khi 1

; 1.2

a b

Bình luận.

Với dạng toán này khi chọ được điểm rơi thì việc biến đổi tương đối đơn giản,

áp dụng BĐT Cô – si quen thuộc và từ giả thiết bài toán để hoàn thành lời giải.

Bài tập tƣơng tự.

Cho a ≥ 10, b ≥ 100, c ≥ 100. Tìm giá trị của biểu thức :

1 1 1.P a b c

a b c

Đáp án.

Pmin = 1110111

1000 đạt được khi a = 10, b = 100, c = 1000.

Câu 4.

Cho 4 số dương x, y, z, t thỏa mãn x + y + z + t = 2. Tìm giá trị nhỏ nhất của

biểu thức ( )( )

.x y z x y

Axyzt

Phân tích.

Nhìn vào bài toán ta có thể đoán được A đạt giá trị nhỏ nhất khi các biến x, y, z,

t được giá trị không bằng nhau. Do đó ta rất khó đoán được điểm rơi của bài

toán, bài tập này đoài hỏi học sinh phải giải nhiều bài tập Cô – si trước đó mới

có kinh nghiệm giải bài toán này.

6

Lời giải.

Với x, y, z, t > 0 theo BĐT Cô – si ta có :

x y x y ≥ 2 xy

( )x y z ≥ 2 ( ).x y z

( )x y z t ≥ 2 ( ).x y z t

Suy ra : ( )( )( )x y x y z x y z t ≥ 8 ( )( )xyzt x y x y z

Mà x + y + z + t = 2

suy ra : ( )( ).2x y x y z ≥ 8 ( )( )xyzt x y x y z

⇔ ( )( )x y x y z ≥ 4 ( )( )xyzt x y x y z

⇔ ( )( )x y x y z ≥ 4. xyzt

⇔ ( )( )x y x y z ≥ 16. xyzt

⇔( )( )x y x y z

xyzt

16xyzt

xyzt = 16

Dấu “=” xảy ra khi

1

4

1

2

12

x yx y

x y zz

x y z t

tx y z t

Vậy Amin= 16 khi x = y = 1 1

, , 1.4 2

z t

Bình luận.

Học sinh phải thành thục BĐT Cô – si Cộng thêm một chút may mắn để giải bài

toán này.

Câu 5.

Cho hai số dương x, y thỏa mãn điều kiện x + y ≤ 4. Tìm giá trị nhỏ nhất của

biểu thức:

2 2

2 352P xy

x y xy

Phân tích.

Khi nhìn vào biểu thức của P ta nhận thấy đây là một biểu thức đối xứng. Khi ta

thay đổi vị trí của x và y cho nhau thì biểu thức không thay đổi, vì vậy ta nghĩ

7

ngay đến giải bằng cách sử dụng điểm rơi rồi dùng BĐT Cô – si. Mặt khác giả

thiết là x + y ≤ 4 kết hợp với biểu thức P đối xứng nên ta có ngay điểm rơi là x =

y = 2. Sau khi xác định được điểm rơi, ta chỉ cần thêm bớt vào biểu thức P một

vài biểu thức phù hợp để sử dụng BĐT Cô – si. Lấy điểm rơi thay vào biểu thức P

ta sẽ biết trước được giá trị nhỏ nhất là của P là bao nhiêu, ở đây minP = 17.

Lời giải.

Ta có: 2 2

2 352P xy

x y xy

=2 2 2 2

2 2

2 35 35 352

32 16 32 16

x y xy x y xyxy

x y xy

= 2 2 2

2 2

2 35 35 ( )

32 16 32

x y xy x y

x y xy

8

xy.

Sử dụng Cô – si cho các cặp (2 2

32

x y;

2 2

2

x y) và (

35

xy ;

35

16

xy) ta có:

2 2

2 2

2

32

x y

x y

22

32 =

1

2 ;

35 35

16

xy

xy ≥

2352.

16 =

35

2

Mặt khác: x + y ≤ 4 ⇒ x.y ≤ 4 nên 8

xy ≤

1

2 ,

2( )

32

x y≤

1

2

Vậy minP = 1

2+

35

2-

1

2-

1

2 = 17. Dấu “=” xảy ra khi x = y = 2

Cách khác.

2 2

2 352P xy

x y xy

=

2 2

2 2 34 17

2 8 8

xy xy

x y xy xy

2

2 2

2.4 34.17 ( )2

2 8 32

x y

x y xy

≥ 8 1

17 1716 2

Dấu “=” xảy ra khi x = y = 2

Bình luận.

Những lời giải bằng cách tách như trên đoài hỏi học sinh phải có những kĩ năng

giải toán tương đối tốt. Với những bạn ở mức học TB-Khá và Khá muốn ăn

điểm được câu này thì nên làm cách ít cần kĩ năng. Mình xin trình bày lời giải

đơn giản nhất các bạn có thể kham khảo.

- Bước 1:Do bài toán đối xứng hoàn toàn với x và y nên dễ dàng dự đoán được

dấu bằng đạt được khi x = y = 2, thay giá trị x = 2, y = 2 vào biểu thức P ta được

giá trị nhỏ nhất của P là: Pmin= 1.

8

- Bước 2: từ đẳng thức (x + y)2 = x

2 + y

2 + 2xy

Ta biểu diễn x2 + y

2 = (x + y)

2 – 2xy ≤ 16 – 2xy ⇒

2 2

2

x y ≥

2

16 2xy

Đặt điều kiện cho biến: xy ≤ 2 2( ) 4

44 4

x y do đó 0 < xy ≤ 4.

Ta sẽ chứng theo phương pháp tương đương trực tiếp:

Ta có: 2 2

2 352P xy

x y xy

≥ f(xy)=

2

16 2xy+

352xy

xy

Đặt xy = t. Bài toán trở thành: P ≥ f(t) = 1

8 t +

352t

t với 0 < t ≤ 4

Ta sẽ chứng minh: f(t) = 1

8 t +

352t

t ≥ 17 (*)

Thật vậy: (*) ⇔ t + 35(8 – t) + 2t2(8 – t) ≥ 17t(8 – t)

⇔ t + 280 – 35t+ 16t2 – 2t

3 ≥ 136t – 17t

2

⇔ 2t3 – 33t

2 + 170t - 280 ≤ 0

⇔ (t – 4)(2t2 – 25t + 70) ≤ 0

⇔ 2t2 – 25t + 70 ≥ 0

⇔ (4 – t)(17 - 2t) + 2 ≥ 0 (Đúng với mọi 0 < t ≤ 4)

Vậy Pmin = 17 khi t = 4 hay x = y = 2

Nhận xét .

Do dự đoán x = y = 2 nên dấu “=” khi t = 4 đúng là cơ sở để ta dồn về nhân tử

(t – 4) để đánh giá

Câu 6.

Cho x, y, z là ba số thực dương thỏa mãn x + y + z = 3. Tìm giá trị nhỏ nhất của

biểu thức: 2 2 2

1 1 1.

1 1 1

x y zQ

y z x

Phân tích.

Đây là bài toán tìm giá trị lớn nhất, nhỏ nhất của một biểu thức khi biết điều

kiện cho trước (Tạm gọi điều kiện ban đầu). Ta nhận thấy biểu thức Q có dạng

đối xứng (nghĩa là vai trò của x, y, z trong bài toán là giống hệt nhau). Do đó ta

nghĩ đến việc sử dụng BĐT Cô – si để giải toán dấu bằng khi x = y = z. Tuy vậy

biểu thức Q khá phức tạp và không có dạng cơ bản, nên ta sẽ cố gắng thêm bớt

và tách các hạng tử nhỏ về các biến quen thuộc và đơn giản hơn để đánh giá.

9

Lời giải.

2 2 2

1 1 1

1 1 1

x y zQ

y z x

2 2 2 2 2 2

1 1 1( ) ( )1 1 1 1 1 1

x y z

y z x y z x

= M + N

Xét 2 2 21 1 1

x y zM

y z x

, áp dụng BĐT Cô – si ta có:

2 2 2

2 2 2

(1 )

1 1 1

x x y xy xyx

y y y

2

2 2

xy xyx x

y

Tương tự ta có: 21

y

z ≥

2

yzy ;

21

z

x ≥

2

zxz . Suy ra:

2 2 21 1 1

x y zM

y z x

≥ ( ) ( )

2 2 2

xy yz zxx y z = 3 -

2

xy yz zx

Lại có: 2 2 2x y z xy yz zx 2( ) 3( )x y z xy yz zx 3xy yz zx

Suy ra: 3

3 32 2

xy yz zxM

Dấu “=” xảy ra ⇔ x = y = z = 1.

Xét N 2 2 2

1 1 1

1 1 1y z x

ta có:

3 – N = 2 2 2

2 2 2 2 2 2

1 1 1(1 ) (1 ) (1 )

1 1 1 1 1 1

y z x

y z x y z x

2 2 2 3

2 2 2 2 2

y z x x y z

y z x

N

3 33

2 2N

Dấu “=” xảy ra khi x = y = z = 1. Từ đó suy ra: Q ≥ 3

Vậy Qmin = 3 ⇔ x = y = z = 1.

Câu 7. Cho x, y, z là các số thực không âm thỏa mãn x + y + z = 1. Chứng

minh: x + 2y + z ≥ 4(1- x)(1 – y)(1 – z)

Lời giải.

Đặt a = x + y, b = y + z, c = z + x thì bài toán trở thành:

Cho a, b, c thỏa mãn a + b + c = 2. Chứng minh rằng: a + b ≥ 4abc

Cách 1. 4(a + b) = (a + b)(a + b + c)2 ≥ (a + b).4.(a + b).c = 4c.(a + b)

2 ≥ 16abc.

(áp dụng BĐT (x + y) ≥ 4xy)

Vậy a + b ≥ 4abc

Cách 2. Ta có:

2 2 ( ). 2 2a b c a b c abc ⇔ 1 ≥ 2 .ab c ⇔ 1 ≥ 2 .ab c > 0

Mà a + b ≥ 2 ab nhân theo vế ta có: a + b ≥ 4abc

10

Đẳng thức xảy ra khi: a + b = c và a = b, khi đó:

1 11

2 22

1 11 0

2 21

1 12

a x yx

b y z y

c z x z

Câu 8. Cho x, y là hai số thực dương. Tìm giá trị nhỏ nhất cảu biểu thức:

2 216 xy x yP

x y xy

Phân tích.

Ta có P gồm 2 216

&xy x y

x y xy

, như vậy trong P xuất hiện xy , x + y. x

2 + y

2 .

Để đơn giải cần đưa P về xy và x + y như sau:

2 216 xy x yP

x y xy

=

216 ( )2

xy x y

x y xy

.

Do x, y đối xứng nên ta dự đoán P đạt giá trị nhỏ nhất là 10 khi x = y.

Do đó sử dụng kĩ thuật chọn điểm rơi tại x = y ta được:

168

xy

x y

,

2( )4

x y

xy

. Suy ra khi x = y thì

28 8 ( )4

xy xy x y

x y x y xy

Khi đó áp dụng BĐT Cô – si cho ba số không âm ta dễ tìm được giá trị nhỏ nhất

của P , ngoài ra ta có thể sử dụng điều kiện dấu “=” khi P đạt giá trị nhỏ nhất để

thêm bớt các hằng số vào biểu thức đánh giá các hằng số như sau:

22 2 216 8( )( )8 2 10 10

xy x yx y x yP

x y xy xy x y

Tới đây ta có thể kết hợp phương pháp biến đổi tương đương kết hợp AM – GM

(Cô – si) để chứng minh: 22 8( )( )

0x yx y

xy x y

Lời giải.

Áp dụng BĐT AM – GM cho ba số không âm ta có:

2 216 xy x yP

x y xy

=

216 ( )2

xy x y

x y xy

=

28 8 ( )2

xy xy x y

x y x y xy

2

38 8 ( )

3. . . 2 10xy xy x y

x y x y xy

11

Đẳng thức xảy ra khi 28 ( )xy x y

x y xy

⇔ x = y

Cách 2. Ta có: 2 216 xy x y

Px y xy

2 2168 2 10

xy x y

x y xy

22 8( )( )10

x yx y

xy x y

Ta chứng minh: 22 8( )( )

10 0x yx y

xy x y

(*)

Thật vậy: (*) ⇔ 2 2( )( ) 8 ( )

0( )

x y x y xy x y

xy x y

2 2( )( ) 8 ( ) 0x y x y xy x y 2 2 2( )( ) ( ) 8 ( ) 0x y x y x y xy x y

2 2( ) (( )( ) 8 ) 0x y x y x y xy (**)

Áp dụng BĐT AM-GM cho hai số không âm ta có:

2( )( ) 8 2 .4 8 0x y x y xy xy xy xy

Vậy (**) đúng hay (*) đúng. Đẳng thức xảy ra khi x = y

Do đó P ≥ 10 . Kết luận: min 10P khi x = y

Cách 3. Ta có: 2 216 xy x y

Px y xy

=

4( ) (4 )10 10

( )

x y xy x y

xy x y

Vậy giá trị nhỏ nhất của P là 10 khi x = y

Bình luận.

Bài toán khá đơn giản. Học sinh chỉ cần nắm vững BĐT AM-GM kết hợp kĩ

thuật chọn điểm rơi là có thể giải toán.

Câu 9. (Trích đề thi vào lớp 10 chuyên Lam Sơn – Thanh Hóa)

Cho a, b, c là độ dài của ba cạnh tam giác. Chứng minh rằng:

2 2 2 2 2 2 2 2 2

12 2 2

a b c b c a a c b

ab bc ca

Phân tích.

Đề bài cho a, b, c là 3 cạnh của tam giác nên ta sẽ lưu ý đến BĐT sau

a + b – c > 0. Bắt đầu bài toán bằng cách chuyển hết sang một vế và quy đồng ,

do mẫu là số dương ta chỉ cần xét tử là thỏa mãn yêu cầu bài toán.

i) Ta nhận thấy có xuất hiện a2 + b

2; b

2 + c

2; c

2 + a

2 khi đó ta nghĩ đến việc làm

suất hiện hằng đẳng thức bằng cách thêm bớt 2ab; 2bc; 2ca cụ thể ta thêm bớt

2abc.

12

ii) Sau khi thêm bớt các hằng đẳng thức, ta tiếp tục sử dụng hằng đẳng thức A2 –

B2, sau đó để ý ta sẽ có nhân tử chung a + b – c.

iii) Sau khi đặt nhân tử chung ta tiếp tục rút gọn để được (a + b – c)(b + c – a)(c

+ a – b) – gợi ý cho ta sử dụng BĐT tam giác.

Lời giải.

Ta có: 2 2 2 2 2 2 2 2 2

12 2 2

a b c b c a a c b

ab bc ca

2 2 2 2 2 2 2 2 2( ) ( ) ( ) 20

2

c a b c a b c a b a c b abc

abc

2 2 2 2 2 2 2 2 2( ) ( ) ( ) 2 0c a b c a b c a b a c b abc

2 2 2 2 2 2 2 2 2( 2 ) ( 2 ) ( 2 ) 0c a b c ab a b c a bc b a c b ca

2 2 2 2 2 2(( ) ) (( ) ) (( ) ) 0c a b c a b c a b c a b

( )( ) ( )( ) ( )( ) 0c a b c a b c a b c a b a c b c a b c b a

2 2 2

2 2

( )[c(a+b+c)+a(b-c-a)+b(a-c-b)] 0

( )( 2 ) 0

( )[ ( ) ] 0

( )( )( ) 0

a b c

a b c c a ab b

a b c c a b

a b c c a b c a b

Do a, b, c là ba cạnh của tam giác nên

0

0

0

a b c

c b a

c a b

Suy ra ( )( )( ) 0a b c c a b c a b ( luôn đúng)

Vậy ta chứng minh được BĐT ban đầu.

Câu 10. (Trích đề vào KHTN – ĐHQG Hà Nội năm 2017 – 2018)

Cho a, b là số các số thực dương. Tìm giá trị lớn nhất của biểu thức:

3 3

1 1 1( )( )M a b

a b a b ab

Phân tích.

Ở bài tập này, đề bài không cho bất kì dữ liệu gì ngoài a, b là các số thực dương,

vì vậy để đánh giá được GTLN của biểu thức ta nghĩ đến việc đánh giá từng

phần của biểu thức và rút gọn cho nhau.

13

Đầu tiên, ta thấy nếu nhân a3 + b với

1b

a và sử dụng bất đẳng thức BCS ta sẽ

thấy xuất hiện được (a + b)2 , tương tự với a + b

3 cũng xuất hiện (a + b)

2 .

3 2 3 21 1( )( ) ( ) ;( )( ) ( ) .a b b a b b a a a b

a b

Khi đó 3 3 2

1 1

1 1

( )

a ba b

a b a b a b

⇔ VT ≤

1 1

1 1 11 1.

a ba b

a b ab ab ab

Đẳng thức xảy ra khi a = b = 1

Vậy giá trị lớn nhất của M là 1 khi a = b = 1.

Bình luận.

Điểm mấu chốt của bài này là tìm ra được nhân tử nhân thêm vào rồi dùng BĐT

BCS đánh giá được một phần của biểu thức. Để tìm được nhân tử nhân thêm

này, ta cần quan sát xem ta cần xuất hiện đa thức gì. Để tra lời câu hỏi này, ta

cần quan sát biểu thức để nhận ra rằng ta càn rút gọn đa thức nào, và trong bài

này đó là a + b.

Câu 11. (Trích đề thi vào lớp 10 chuyên KHTN vòng 2 năm 2017 )

Với a, b, c là các số thực dương thỏa mãn ab + bc + ca + abc = 2. Tìm giá trị lớn

nhất cảu biểu thức:

2 2 2

1 1 1.

2 2 2 2 2 2

a b cM

a a b b c x

Phân tích.

Gần như việc áp dụng trực tiếp giả thiết ab + bc + ca + abc = 2 là điều không

thể. Do đó ta bắt đầu khai thác từ biểu thức M.

Do vài trò của a, b, c trong M là như nhau nên ta chỉ cần xét một phân thức

trong ba phân thức đó, thực hiện tương tự với hai phân thức còn lại.

Không mất tính tổng quát ta xét: 2 2

1 1

2 2 ( 1) 1

a a

a a a

Tưng tự với b, c ta thấy rằng biểu thức M hoàn toàn có thể biểu thị qua

1, 1, 1a b c . Ta có:

14

2

1 3

( 1)( 1)( 1) ( 1)( 1)( 1)

( 1) ( 1) ( 1)1

( 1)( 1)( 1)

1 1 11

( 1)( 1) ( 1)( 1) ( 1)( 1)

abc ab bc ca

abc ab ac bc a b c a b c

a b c a b c

a b c

a b c

a b b c c a

Để bài toán đơn giải ta cần thay đổi biến của bài toán. Đối chiếu với giả thiết

của bài toán ta nghĩ ngay đến việc đặt 1 1 1

1 , 1 , 1 .a b cx y z

Tuy nhiên nếu phân tích sâu hơn ta sẽ thấy điều kiện dấu “=” của bài toán là

3 1a b c hay 1 1 1 3a b c nên việc đặt

3 3 31 , 1 , 1a b c

x y z sẽ giúp bài toán gọn hơn.

Ngoài ra bài toán còn có hướng là đưa M về hai biến rồi đánh giá M .

Lời giải.

Trước hết ta chứng minh bất đẳng thức sau: 1 1 4

, , 0x yx y x y

(*)

Ta có (*) tương đương với: 1 1 4

, 0x yx y x y

2

1 1 40

( )0(**)

( )

x y x y

x y

xy x y

Ta có (**) luôn đúng nên (*) được chứng minh.

Quay trở lại bài toán ta có:

2

1 3

( 1)( 1)( 1) ( 1)( 1)( 1)

( 1) ( 1) ( 1)1

( 1)( 1)( 1)

1 1 11

( 1)( 1) ( 1)( 1) ( 1)( 1)

abc ab bc ca

abc ab ac bc a b c a b c

a b c a b c

a b c

a b c

a b b c c a

Đặt 3 3 3

1 , 1 , 1a b cx y z

15

Khi đó giả thiết bài toán trở thành: xy + yz + zx = 3 và

2 2 2

2 2 2

2 2 2

1 1 1

2 2 2 2 2 2

1 1 1

( 1) 1 ( 1) 1 ( 1) 1

1 1 1

1 1 11 1 1

1 1 1

1 1 1

3 3 3

3 3 3

3( )1 1 1

3( )( )( ) ( )( ) ( )( )

a b cM

a a b b c x

a b c

a b c

a b ca b c

x y z

x y z

x y z

x y z

x y z

x y x z y z y x z x z y

3( )

4

3 3.

4

x x y y z z

x y x z y z y x z x z y

(Áp dụng bất đẳng thức (*))

Đẳng thức xảy ra khi x = y = z = 3 1

Ngoài ra khi đưa M về dạng:

3(( )( ) ( )( ) ( )( )

x y zM

x y x z y z y x z x z y

Ta còn có thể giải cách khác như sau:

Cách 2. Trước tiên ta chứng minh hai BĐT phụ sau.

i) (x + y)(y + z)(z + x) ≥ 8

( )( ), , , 0.9

x y z xy yz zx x y z

Biến đổi tương đương BĐT này ta được:

2 2 2( ) ( ) ( ) 0.x y z y z x z x y (luôm đúng)

Vậy bất đẳng thức này được chứng minh.

16

ii) 2( ) 3( ) , , 0.x y z xy yz zx x y z

Biến đổi tương đương BĐT này ta được:

(x – y)2 + (y – z)

2 + (z –x)

2 ≥ 0 (luôn đúng)

Đẳng thức xảy ra khi x = y = z.

Quay trở lại bài toán ta có:

3(( )( ) ( )( ) ( )( )

( ) ( ) ( ) 2 3( ) 9 3( )3( )

( )( )( ) ( 0( )( ) 4( )( )

9 3 9 3 3 3

4( ) 44 3( )

x y zM

x y x z y z y x z x z y

x y z y z x z x y xy yz zx xy yz zx

x y y z z x x y y z z x x y z xy yz zx

x y z xy yz zx

Dấu « = » xảy ra khi a = b = c = 3 1

Bình luận.

Đây là bài toán khá quen thuộc ở THPT, tuy nhiên trong chương trình THCS thì

đây là dạng bài tương đối khó trong việc khai thác giả thiết để đưa vào biểu thức

M. Để giải quyết được bài toán yêu cầu học sinh phải bám sát biểu thức cần

chứng minh kết kĩ năng biến đổi đại số.

Câu 12. (Trích đề thi vào lớp 10 chuyên Bình Dương 2017)

Cho x, y là số thực dương thỏa mãn x ≥ 2y. Tìm giá trị nhỏ nhất của biểu thức

2 2

.x y

Mxy

Phân tích.

Trong giả thiết bài toán ta để ý đến x và 2y ta sẽ chứng minh Cô – si cho 2 số x2

và 4y2. Khi đó, đẳng thức xảy ra khi x = 2y.

Lời giải.

Ta có : 2 2 2 2 24 3 4 3 3 5

4 .2 2

x y x y y xy yM

xy xy xy xy x

Đẳng thức xảy ra khi 2 24

22

x yx y

x y

Vậy Mmin= 5

2 khi x = 2y.

Bình luận.

17

Đây là Bất đẳng thức có điều kiện biến đơn giản, chỉ cần để ý điều kiện là dễ dàng

tìm hướng đi đúng.

Câu 13. (Trích đề chuyên Lê Hồng Phong – Nam Định 2017)

Xét các số thực a, b, c không âm, khác 1 thỏa mãn a + b + c = 1. Tìm giá trị nhỏ

nhất cảu biểu thức 1 1

( )(4 5 ).P a b ca bc b ac

Phân tích.

Với bài toán ban đầu ta để ý khi cộng mẫu thức của hai phân số lại ta sẽ có tích

( )(1 )a b c nên nếu sau đó ta dùng BĐT Cô – si ta sẽ rút gọn đi được nhân tử (a +

b) và cuối cùng kết hợp với điều kiện 0 ≤ c ≠ 1sẽ cho ra giá trị nhỏ nhất cảu P .

Và điều kiện a+ b + c được dùng để giải điều kiện dấu « = » xảy ra.

Lời giải.

Áp dụng BĐT : 1 1 1

( , 0)x yx y x y

Tacó :

1 1 4( )(5 4) ( )(5 4)

( )( 1)

4 5 4(1 )(5 4) 4 4 4 8

(1 )(1 ) 1 1

P a b c a b ca bc b ac a b c

c cc c

c c c c

Vậy minP = 8. Dấu « = » xảy ra khi 1

0, .2

c a b

Bình luận.

Ở bài toán này học sinh có thể gặp khó khăn trong việc giải dấu « = » do chưa tận

dụng được giả thiết a + b + c = 1.

Câu 14. (Trích đề thi vào lớp 10 chuyên Lê Quý Đôn – Bà Rịa Vũng Tàu)

Cho x, y là các số thực dương. Tìm giá trị nhỏ nhất của biểu thức :

2 2

2 2

1 1( ) 2( ).

xyP x y

x y x y

Phân tích.

Nhận thấy vài trò của x và y trong biểu thức P là như nhau, ta có thể dễ thấy điểm

rơi của bài toán là x = y, khi đó P = 9

.2

18

Do x, y là hai số thực dương ta có thể áp dụng BĐT Cô – si. Ta có một vài bất

đẳng thức sau : 2 2 2 ;x y xy 1 1 2

.x y xy

. Nếu áp dụng đơn thuần các BĐT trên ta được :

2 2

1;

2

xy

x y

2 21 1

( ) 2( ) 4x yx y

Thì BĐT sẽ bị ngược chiều. Do đó ta tập trung khai thác riêng lẻ 1 1

x y và

2 22( )x y theo một hướng khác.

Ta có x2 + y

2 ≥ 2xy nên : 2(x

2 + y

2) ≥ (x + y)

2 ta có thể đưa ra ngoài căn thức. Hơn

nữa kết hợp với 1 1

( )x y ta sẽ được

2 2 2( )2

x y x y

xy xy

có vẻ liên quan đến hạng

tử thứ nhất 2 2

xy

x y .

Hơn nữa 2 2x y

xy

2 2

xy

x y là hai số nghịch đảo áp dụng BĐT Cô – si cho tổng

của chúng sẽ được lướn hơn một hằng số là một yêu cầu khi muốn dùng BĐT Cô

– si để tìm GTLN và GTNN.

Lời giải.

Ta có : x2 + y

2 ≥ 2xy nên : 2(x

2 + y

2) ≥ (x + y)

2. Do đó :

2 22 2

2 2 2 2 2 2

2 2 2 2

2 2 2 2 2 2

1 1 1 1( ) 2( ) ( )( ) 2

4 3 3( )2 2 4 2

2( )

3 96 .

2 2

xy xy xy x yP x y x y

x y x y x y x y x y xy

xy x y xy x y

x y xy x y x y

Dấu « = » xảy ra khi

2 2

2 2

2 2

2

.4

x y xy

x yxy x y

x y xy

Vậy minP = 9

2 khi x = y.

Bình luận.

Khi chứng minh BĐT hoặc tìm giá trị lớn nhất nhỏ nhất ta thường gặp các trường

hợp « ngược chiều » như phân tích ở trên.

19

Việc xác định được điểm rơi của bài toán sẽ giúp nhiều cho việc định hướng giải

bài toán. Chẳng hạn khi phân tích tới 2 2

2 22

xy x yP

x y xy

nếu không để ý

đếnđiểm rơi ta dễ dàng mắc phải sai lầm khi sử dụng BĐT Cô – si cho 2 2x y

xy

2 2

xy

x y suy ra

2 2

2 22 . 2 4

x y xyP

xy x y

. Tuy nhiên dấu « = » xảy ra khi

2 2x y

xy

=

2 2

xy

x y⇔ xy = x

2 + y

2 (vô lý).

Với điểm rơi là x = y khi đó 2 2

1

2

xy

x y

2 2

2x y

xy

nên suy ra

2 2x y

xy

=

2 2

4xy

x y

và áp dụng BĐT Cô – si cho hai số này.

Câu 15. (Trich đề tuyển sinh vào lớp 10 – chuyên Bạc Lưu năm 2017)

Cho a, b, c thỏa mãn 1; 4; 9a b c . Tìm giá trị nhỏ nhất của biểu thức :

1 4 9.

bc a ac b ab cM

abc

Phân tích.

Dễ thấy 1 4 9

.a b c

Ma b c

Do vài trò của a, b, c không bình đẳng nên để tìm giá trị nhỏ nhất của M ta đi tìm

giá trị nhỏ nhất của 1 4 9

; ; .a b c

a b c

Xét 1a

a

. Áp dụng BĐT Cô – si cho hai số không âm ta có :

21 1 1. 1 1. .

2

a a a

a a a

Để tìm được giá trị lớn nhất của 1a

a

thì 2 1 0 hay 1

Khi đó : 1 1

.2

a

a

Tương tự, ta chứng minh được : 4 1 9 1

; .4 6

b c

b c

Lời giải.

20

Ta có : 1 4 9

.a b c

Ma b c

Do 1; 4; 9a b c . Áp dụng BĐT AM –GM cho các cặp số không âm, ta có :

1 1

2( 1) 1 2 14 1

( 4) 4 4 44

( 9) 9 6 99 1

6

a

aa a ab

b b bb

c c cc

c

. Do đó : 1 1 1 11

.2 4 6 12

M

Dấu « = » xảy ra khi

1 1 2

4 4 8

9 9 18

a a

b b

c c

Bình luận.

Đây là bài toán BĐT khá đơn giản, chỉ cần nắm vững BĐT AM-GM và cách

tham số hóa để triệt tiêu các hằng số tự do là có thể đạt điểm tối đa bài toán.

Câu 16. Cho hai số dương a, b thỏa mãn: a + b 2 2 . Tìm giá trị nhỏ nhất của

biểu thức: P = 1 1

a b .

Phân tích.

Việc tìm GTNN của biểu thức P bao giờ cũng vận hành theo sơ đồ "bé dần": P

B, (trong tài liệu này chúng tôi sử dụng B - chữ cái đầu của chữ bé hơn).

1) Giả thiết a + b 2 2 đang ngược với sơ đồ "bé dần" nên ta phải chuyển

hoá a + b 2 2 1 1

2 2a b

.

Từ đó mà lời giải đánh giá P theo 1

a b.

2) 1 1 4

a b a b

với a > 0, b > 0 là một bất đẳng thức đáng nhớ. Tuy là một hệ

quả của bất đẳng

Cô-si, nhưng nó được vận dụng rất nhiều. Chúng ta còn gặp lại nó trong một

số đề sau.

3) Các bạn tham khảo lời giải khác của bài toán như là một cách chứng minh

bất đẳng thức trên.

21

Với hai số a > 0, b > 0 ta có 1 1 2 2.2 4 4

22 2

Co si Co si

Pa b a b a bab

. Dấu

đẳng thức có khi a = b = 2 . Vậy minP = 2 .

Lời giải.

Ta có (a + b)2 – 4ab = (a - b)

2 0(a + b)

2 4ab

a + b 4 1 1 4

ab a + b b a a + b

4

Pa + b

, mà a + b 2 2

4 4

a + b 2 2 P 2 . Dấu “ = ” xảy ra

2

a - b 0a = b = 2

a + b = 2 2

. Vậy:

min P = 2 .

Câu 17.

Giải phương trình:y - 2010 1x - 2009 1 z - 2011 1 3

x - 2009 y - 2010 z - 2011 4

Phân tích.

1) Việc đặt a, b, c thay cho các căn thức là cách làm để dễ nhìn bài toán,

Với mọi số dương a, b, c ta luôn có

2 2 2

1 1 1 3

4

a b c

a b c

. (1)

Thay vì đặt câu hỏi khi nào thì dấu đẳng thức xẩy ra, người ta đặt bài toán

giải phương trình

2 2 2

1 1 1 3

4

a b c

a b c

. (2)

Vai trò của a, b, c đều bình đẳng nên trong (1) ta nghĩ đến đánh giá 2

1 1

4

a

a

.

Thật vậy 2

1 1

4

a

a

2

1 10

4

a

a

2

2

( 2)0

a

a

. Dấu đẳng thức có khi và

chỉ khi a = 2. Tương tự ta cũng có 2

1 1

4

b

b

,

2

1 1

4

c

c

. Dấu đẳng thức có khi và

chỉ khi b = 2, c = 2.

2) Mỗi giá trị của biến cân bằng bất đẳng thức được gọi là điểm rơi của

bất đẳng thức ấy.

Theo đó, bất đẳng thức (1) các biến a, b, c đếu có chung một điểm rơi là a =

b = c = 2.

Khi vai trò của các biến trong bài toán chứng minh bất đẳng thức bình

đẳng với nhau thì các biến ấy có chung một điểm rơi.

22

Phương trình diễn tả dấu bằng trong bất đẳng thức được gọi là "phương

trình điểm rơi".

3) Phương trình (2) thuộc dạng "phương trình điểm rơi"

Tại điểm rơi a = b = c = 2 ta có 2 2 2

1 1 1 1

4

a b c

a b c

.

Điều đó cắt nghĩa điểm mấu chốt của lời giải là tách 3 1 1 1

4 4 4 4 :

(2) 2 2 2

1 1 1 1 1 10

4 4 4

a b c

a b c

.

4) Phần lớn các phương trình chứa hai biến trở lên trong chương trình

THCS đều là "phương trình điểm rơi".

Lời giải.

Đặt x - 2009 a; y - 2010 b; z - 2011 c

(với a, b, c > 0). Khi đó phương trình đã cho trở thành:

2 2 2

a - 1 b - 1 c - 1 3

a b c 4

2 2 2

1 1 1 1 1 1 1 1 10

4 a a 4 b b 4 c c

2 2 21 1 1 1 1 1

02 a 2 b 2 c

a = b = c = 2

Suy ra: x = 2013, y = 2014, z = 2015

Câu 18.

Tìm giá trị nhỏ nhất của biểu thức:

P = 2x - x y + x + y - y + 1

Lời giải.

ĐK: y > 0 ; x R. Ta có: P =

2x - x y + x + y - y + 1

2

2y 1 y3y 3

= x - x( y - 1) + + - + 4 4 2 4

22

y 1 3 1 2 2x - y

2 4 3 3 3

. Dấu “=” xảy ra

- 1x =

3

1y =

9

.

Suy ra: 2

Min P = 3

.

Câu 19.

Cho a, b, c là độ dài 3 cạnh của một tam giác. Chứng minh:

23

ab + bc + ca a2 + b

2 + c

2 < 2(ab + bc + ca ).

Lời giải.

Ta có: 2 2 2

a - b b - c c - a 0 2 2 22 a b c 2 ab + bc + ca

2 2 2a b c ab + bc + ca (1).

Vì a, b, c là độ dài 3 cạnh của một tam giác nên ta có: a2 < a.(b+ c)a

2 < ab +

ac.

Tương tự: b2 < ab + bc; c

2 < ca + bc. Suy ra: a

2 + b

2 + c

2 < 2(ab + bc + ca) (2).

Từ (1) và (2) suy ra điều phải chứng minh.

Câu 20.

Cho biểu thức A = 2x - 2 xy + y - 2 x + 3 . Hỏi A có giá trị nhỏ nhất hay không?

Vì sao?

Phân tích.

Cảnh báo. Các bạn cùng theo dõi một lời giải sau :

Biểu thức A có nghĩa khi và chỉ khi 0

0

x

y

. Biến đổi

2 2

1 2A x y x .

Suy ra minA = 2, đạt được khi x = y = 1 (!).

Kết quả bài toán sai thì đã rõ. Nhưng cái sai về tư duy mới đáng bàn hơn.

1) Điều kiện xác định của P(x; y) chứa đồng thời x và xy là

0 0

0

x xD

y y

Do vậy để tìm GTLN, GTNN P(x; y) cần phải xét độc lập hai trường hợp

0x

y

0

0

x

y

2) Không thể gộp chung 0 0

0

x x

y y

thành

0

0

x

y

24

3) Do cho rằng điều kiện xác định của P(x; y) là 0

0

0y

xD

y

(bỏ sót

0

0

0y

xD

y

)

Vậy nên A = 2 là GNNN của A trên 0yD , chưa đủ để kết luận đó là GTNN của

A trên D.

4) Nhân đây liên tưởng đến phương trình ( ) ( ) 0P x Q x . (1)

Biến đổi đúng (1)

( ) 0

( ) 0

( ) 0

Q x

Q x

P x

. Cách biến đổi sau là sai (1) ( ) 0

( ) 0

Q x

P x

.

Lời giải.

A = 2 - 2 - 2 3 x xy y x .

Trước hết ta thấy biểu thức A có nghĩa khi và chỉ khi: 0

0

x

xy(1).

Từ (1) ta thấy nếu x = 0 thì y nhận mọi giá trị tùy ý thuộc R (2).

Mặt khác, khi x = 0 thì A = y + 3 mà y có thể nhỏ tùy ý nên A cũng có thể nhỏ

tùy ý. Vậy biểu thức A không có giá trị nhỏ nhất.

Câu 21.

Chứng minh rằng:

a + b 1

2a 3a + b b 3b + a

với a, b là các số dương.

Lời giải.

Ta có:

a + b 2(a + b)(1)

a 3a + b b 3b + a 4a 3a + b 4b 3b + a

Áp dụng bất đẳng thức Cô-si cho các số dương ta được:

4a + (3a + b) 7a + b4a 3a + b 2

2 2

4b + (3b + a) 7b + a4b 3b + a 3

2 2

Từ (2) và (3) suy ra: 4a 3a + b 4b 3b + a 4a + 4b 4

Từ (1) và (4) suy ra:

a + b 2(a + b) 1

4a + 4b 2a 3a + b b 3b + a

. Dấu bằng xảy ra khi và chỉ khi a = b.

25

Bình luận.

Các bạn được sử dụng bất đẳng thức Cô-si để làm toán như một định lý

(không phải chứng minh)

Bất đẳng thức Cô-si chỉ áp dụng cho các số không âm. Cụ thể là :

+ Với hai số a 0, b 0 ta có 2

a bab

, dấu đẳng thức có khi và chỉ khi

a = b.

+ Với ba số a 0, b 0, c 0 ta có 3

3

a b cabc

, dấu đẳng thức có khi

và chỉ khi a = b = c.

Câu 22.

Cho x > 0, y > 0 và x + y ≥ 6. Tìm giá trị nhỏ nhất của biểu thức :

P = 3x + 2y + 6 8

+ x y

.

Lời giải.

Ta có : P = 3x + 2y + 6 8 3 3 3 6 y 8

+ = ( x + y) + ( x + ) + ( + )x y 2 2 2 x 2 y

Do 3 3 3 3

x + y = x + y . 6 = 9.2 2 2 2

3x 6 3x 6 + 2 . = 6

2 x 2 x ,

y 8 y 8 + 2 . = 4

2 y 2 y

Suy ra P ≥ 9 + 6 + 4 = 19

Dấu bằng xẩy ra khi

x + y = 6

x = 23x 6 =

y = 42 x

y 8 =

2 y

Vậy min P = 19.

Bình luận.

Việc tìm GTNN của biểu thức P bao giờ cũng vận hành theo sơ đồ "bé

dần": P B, (trong tài liệu này chúng tôi sử dụng B - chữ cái đầu của chữ bé

hơn).

26

1) Do giả thiết cho x + y 6, đã thuận theo sơ đồ "bé dần": P B, điều ấy

mách bảo ta biểu thị P theo (x + y). Để thực hiện được điều ấy ta phải khử 6

x và

8

y.

Do có x > 0; y > 0 nên việc khử được thực hiện dễ dàng bằng cách áp dụng

bất đẳng thức Cô-si cho các từng cặp số Ax và 6

x, By và

8

y.

Bởi lẽ đó mà lời giải đã "khéo léo" tách 3 3

32 2

x x x , 3 1

22 2

y y y .

2) Tuy nhiên mấu chốt lời giải nằm ở sự "khéo léo" nói trên. Các số 3

2,

1

2

được nghĩ ra bằng cách nào?

Với mọi số thực a < 2, ta có

6 8

3 2P x yx y

=6 8

( ) (3 ) (2 )a x y a x a yx y

(1)

6 2 6(3 ) 2 8(2 )P a a a (2)

Ta có 6

(3 ) 2 6(3 )a x ax

, dấu đẳng thức có khi 6

3x

a

; (3)

8

(2 ) 2 8(2 )a y ay

, dấu đẳng thức có khi 8

2y

a

. ; (4)

Để (2) trở thành đẳng thức buộc phải có x + y = 6 6 8

63 2a a

(5)

Thấy rằng 3

2a là một nghiệm của (5). Thay

3

2a vào (2) ta có sự phân tích

như lời giải đã trình bày. Các số 3

2,

1

2được nghĩ ra như thế đó.

3) Phương trình (3) là phương trình "kết điểm rơi". Người ta không cần biết phương

trình "kết điểm rơi" có bao nhiêu nghiệm. Chỉ cần biết (có thể là đoán) được một

nghiệm của nó là đủ cho lời giải thành công. (Việc giải phương trình "kết điểm rơi"

nhiều khi phức tạp và cũng không cần thiết.)

Câu 23.

Các số thực x, a, b, c thay đổi, thỏa mãn hệ:

27

2 2 2 2

x + a + b + c = 7 (1)

x + a + b + c = 13 (2)

Tìm giá trị lớn nhất và giá trị nhỏ nhất của x.

Lời giải.

Tìm GTLN, GTNN của x thoả mãn.

2 2 2 2

x + a + b + c = 7 (1)

x + a + b + c = 13 (2)

Từ (1) a + b + c = 7 - x.. Từ (2) a2 + b

2 + c

2 = 13 - x

2.

Ta chứng minh: 3(a2 + b

2 + c

2) ≥ (a + b + c)

2.

3a2 + 3b

2 + 3c

2 - a

2 - b

2 - c

2 - 2ab - 2ac - 2bc ≥ 0

(a - b)2 + (b - c)

2 + (c - a)

2 ≥ 0 (đpcm)

Suy ra 3 (13 - x2) ≥ (7 - x)

2. 3 (13 - x

2) ≥ 49 - 14x + x

2.

4x2 - 14x + 10 ≤ 0 1 ≤ x ≤

5

2.

5 3x khi a b c , x 1 khi a b c 2

2 2 .

Vậy max x = 5

2, min x = 1.

Câu 24.

Cho các số dương a, b, c. Chứng minh rằng:

a b c

1 + + 2a + b b + c c + a

Lời giải.

Ta có a

a + b + c <

a

b + a <

a + c

a + b + c (1)

b

a + b + c <

b

b + c <

b + a

a + b + c (2)

c

a + b + c <

c

c + a <

c + b

a + b + c (3)

Cộng từng vế (1), (2), (3), ta được : 1 <a

a + b +

b

b + c +

c

c + a < 2, đpcm.

Câu 25.

Cho x, y là hai số thực thoả mãn: (x + y)2 + 7(x + y) + y

2 + 10 = 0

Tìm giá trị lớn nhất và giá trị nhỏ nhất của biểu thức A = x + y + 1

Lời giải.

28

Từ giả thiết: (x + y)2 + 7(x + y) + y

2 + 10 = 0

2 2

2 27 7 7 x +y + 2. x +y . + - + 10 = - y 0

2 2 2

2 27 9 7 9

x + y + - 0 x + y + 2 4 2 4

.

Giải ra được - 4 ≤ x + y + 1 ≤ - 1.

A = -1 khi x = - 2 và y = 0, A = - 4 khi x = -5 và y = 0.

Vậy giá trị nhỏ nhất của A là - 4 và giá trị lớn nhất của A là - 1.

Cách khác.

Từ A = x + y +1 y = A x 1, thế vào giả thiết có phương trình bậc hai đối

với x. Từ 0 ta tìm được minA, maxA .

Câu 26.

Tìm giá trị nhỏ nhất của biểu thức: P = 4 2

2

x + 2x + 2

x + 1.

Lời giải.

P = x2 + 1 +

2

1

x + 1 ≥ 2

2

1 2 x + 1

x + 1, P = 2 x

2 + 1 =

2

1

x + 1 x = 0. Vậy

min P = 2.

Câu 27.

Cho các số dương a, b, c. Chứng minh bất đẳng thức:

4a b b c c a a b c

c a b b c c a a b

.

Lời giải.

Với các số dương x, y ta có: 2

4x y xy 4x y

xy x y

1 1 4

x y x y

Áp dụng bất đẳng thức trên ta, có:

1 1 1 1 1 1a b b c c aa b c

c a b b c c a a b

4 4 4. . .a b cb c c a a b

= 4a b c

b c c a a b

Vậy bất đẳng thức được chứng minh.

Câu 28. Tìm các giá trị x để 1

342

x

x là số nguyên âm.

29

Lời giải.

Đặt 1

342

x

xy .

Khi đó ta có 3412 xxy 034. 2 yxxy (1).

Ta tìm điều kiện của y để (1) có nghiệm.

Nếu 0y thì (1) có nghiệm 3

4x .

Nếu 0y , (1) có nghiệm 032' 2 yy 0432 yy 41 y .

Kết hợp lại thì (1) có nghiệm 41 y .

Theo giả thiết y là số nguyên âm 1y . Khi đó thay vào trên ta có 2x .

Bình luận.

1) Từ cách giải bài toán trên ta suy biểu thức 2

4 3

1

xy

x

có GTNN bằng 1 và

GTLN bằng 4.

2) Phương pháp giải bài toán trên cũng là phương phương pháp tìm GTNN,

GTLN của các biểu thức dạng 2

2' ' '

ax bx cP

a x b x c

(với b'

2 4ac < 0), chẳng hạn

2

2

20 10 3

3 2 1

x xP

x x

;

2 2

2 2

8 7x xy yQ

x y

với x

2 + y

2 > 0;

F = x2 + 2xy y

2 với 4x

2 + 2xy + y

2 = 3.

Câu 29.

Cho các số thực dương a, b, c thoả mãn 1

a b cabc

.

Tìm giá trị nhỏ nhất của biểu thức P = a b a c .

Lời giải.

Từ giả thiết ta có: 1abc a b c . Do đó, áp dụng bất đẳng thức Côsi,

P = a b a c = 2a ab ac bc = a a b c bc 2 a a b c bc = 2.

Đẳng thức xảy ra

1

a a b c bc

a b cabc

1

1

a a b c

bc

.

Hệ này có vô số nghiệm dương, chẳng hạn ta chọn b = c = 1 a = 2 1 .

Vậy giá trị nhỏ nhất của biểu thức P là 2.

Câu 30.

30

: Cho hai số dương x, y thỏa mãn điều kiện x + y = 1.

Hãy tìm giá trị nhỏ nhất của biểu thức: A = 2 2

1 1

x y xy

Lời giải.

A = 2 2

1 1

x y xy

=

2 2

1 1 1

x y 2xy 2xy

Áp dụng bất đẳng thức Côsi cho hai số dương ta có:

1x + y 2 xy 1 2 xy 1 4xy 2

2xy (1)

Đẳng thức xảy ra khi x = y.

Tương tự với a, b dương ta có:

1 1 1 2 42 2.

a b ab a + b a + b (*)

Áp dụng bất đẳng thức (*) ta có:

22 2

1 1 44

x y 2xy x + y

(2)

Dấu đẳng thức xảy ra khi x2 + y

2 = 2xy x = y.

Từ (1) và (2) suy ra: A 6 . Dấu "=" xảy ra 1

x = y = 2

. Vậy minA = 6.

Câu 31.

Cho các số dương cba ,, . Chứng minh bất đẳng thức:

2

ba

c

ac

b

cb

a.

Lời giải.

Vì các số cba ,, dương nên áp dụng bất đẳng thức Côsi cho hai số ta có:

2

)( cbacba

cba

a

cba

a

cb

a

2

Tương tự ta cũng có:

cba

b

ac

b

2,

cba

c

ba

c

2

Cộng các bất đẳng thức cùng chiều trên ta có

2222

cba

cba

ba

c

ac

b

cb

a.

31

Dấu bằng xảy ra

bac

acb

cba

0 cba , không thoả mãn.

Vậy 2

ba

c

ac

b

cb

a

Câu 32

Tìm giá trị nhỏ nhất của hàm số: y = xx

1

1

2

, với 0 < x < 1

Lời giải.

Ta có y = x

xx

x

xx

xx

)1(

1

2)22(1

1

2

= 2 + 1 + 2231

.1

223

1

1

2

x

x

x

x

x

x

x

x (áp dụng BĐT Côsi với 2 số

dương)

Đẳng thức xảy ra <=> 121

1

2

x

x

x

x

x (loại nghiệm x = - 1 - 2 )

Vậy giá trị nhỏ nhất của y bằng 3 + 2 2 khi x = 2 -1

Câu 33.

Tìm giá trị nhỏ nhất của biểu thức: y = 2

2

x x 1

x 2x 2

.

Lời giải.

y = 2

2 2

2

x x 1y(x 2x 2) (x x 1) 0

x 2x 2

(y - 1)x2 + (2y - 1)x + (2y - 1) = 0 (1)

- Nếu y = 1 thì x = - 1

- Nếu y 1 thì (1) là phương trình bậc hai đối với x. Để (1) có nghiệm thì phải

= (2y - 1)2 - 4 (y - 1)(2y-1)

0

1 3(2y 1)(2y 3) 0 y

2 2 .

1y

2 khi x = 0. Vậy min y =

1

2.

Câu 34.

Cho x và y là hai số thỏa mãn đồng thời : x 0 , y 0, 2x + 3y 6 và 2x + y 4.

Tìm giá trị nhỏ nhất và giá trị lớn nhất của biểu thức K = x 2 - 2x – y.

Lời giải.

32

Từ 2x + 3y 6 2 2

y 2 - x - y x - 23 3

K = x2 - 2x - y

2 22x 2 22 - 22x - 2x + - 2 = (x - ) -

3 3 9 9

Suy ra : min K = - 22

9 khi x =

2

3 ; y =

14

9

Ta có : 2x2 + xy 4x ( x0)

2- y x + 2xy

x - 2x - y - - y = 02 2

Suy ra : max K = 0 khi y = 0

x = 0

hoặc y = 0

x = 2

Bình Luận.

Nhiều khi tìm trực tiếp GTNN của biểu thức K thật khó khăn. "Cái khó ló cái

khôn", người ta bắc cầu K qua biểu thức B (bé hơn) theo sơ đồ "bé dần": K B .

Rồi đi tìm GTNN của B, từ đó mà suy ra GTNN của biểu thức K. Các mối liên

hệ giữa K và giả thiết sẽ chỉ dẫn chúng ta tìm đến B.

+ Trong bài toán trên, thấy trong biểu thức K = x2 2x y có chứa y, nên để

thuận theo sơ đồ "bé dần" ta biến đổi :

2x + 3y 6 2

23

xy

Thay y bởi 2

23

x ta có

22 22

3 9K B x

.

Cũng vậy, đối với tìm GTLN thì việc bắc cầu phải theo sơ đồ "lớn dần": K

L

+ Trong các giả thiết không thể suy ra y h(x) để tìm L (lớn hơn) trong sơ đồ

"lớn dần" . Vậy nên để có biểu thức L buộc phải đánh giá bộ phận còn lại x2

2x g(x).

Ta có 2x + y 4 22

yx

0x

2 22

xyx x . (ở đây ( )

2

xyg x )

Thay x2 2x bởi

2

xy ta có ( 2)

2

yK L x .

Chắc chắn bạn còn thắc mắc là bài toán có hai giả thiết, thế nhưng khi tìm

GTNN (GTLN) lại sử dụng giả thiết này mà không sử dụng giả thiết kia ?

33

+ Trong quá trình đánh giá có thể tìm được nhiều biểu thức B. Gọi Bk là một

trong số các biểu thức B tìm được và có minBk = . Thế thì chưa hẳn đã là

GTNN của K. Chỉ trong trường hợp khi minBk = mà ta cũng có K = Bk (hoá

giải được dấu "=" trong sơ đồ "lớn hơn") thì mới có minK = minBk = .

Trong trường hợp đó biểu thức Bk được gọi là "kết". Lời giải chỉ thành công khi

tìm được "kết". Trong bài toán trên, sử dụng giả thiết còn lại không dẫn tới

"kết".

Tình huống cũng tương tự đối với việc tìm biểu thức L. Biểu thức L dẫn tới

maxK cũng được gọi là "kết".

+ Trong bài toán trên, hình thức các giả thiết chưa đủ để chỉ dẫn "bắt mạch" sử

dụng giả thiết này hay giả thiết kia. Nhiều bài toán phức tạp có thể cần sự kết

hợp của tất cả các giả thiết mới tìm được "kết".

Mấu chốt của bài toán tìm GTNN, GTLN là tìm "kết".

Câu 35.

Hai số thực x, y thoả mãn hệ điều kiện : 3 2

2 2 2

x 2y 4y 3 0 (1)

x x y 2y 0 (2)

.

Tính giá trị biểu thức P = 2 2x y .

Lời giải.

Từ (1) ta có: 3 2x 2(y 1) 1 1 x 1 (3)

Từ (2) ta có: 2 2

2

2yx 1 x 1 1 x 1

y 1

(4)

Từ (3) và (4), suy ra x = -1, thay vào hệ đã cho ta được y = 1.

Vậy P = 2.

Câu 36.

Cho các số dương x, y, z thỏa mãn: x + y + z ≤ 3.Tìm giá trị lớn nhất của biểu

thức:

A = 2 2 21 x 1 y 1 z 2 x y z .

Lời giải.

: Áp dụng các BĐT:

2 2a + b 2 a b ; a + b + c 2 2 23 a b c

(được suy ra từ bất đẳng thức Bunhiacôpski)

34

Ta có:

2 2

2 2

2 2

1 + x 2x 2 1 x 2x 2 x + 1

1 + y 2y 2 1 y 2y 2 y + 1

1 + z 2z 2 1 z 2z 2 z + 1

x y z 3 x + y + z

Lại có: A = 2 2 21 x 1 y 1 z 2x 2y 2z

+ 2 2 x y z

A 2 x + y + z + 3 2 2 3 x + y + z

A 6 + 3 2 (do x + y + z 3). Dấu “=” xảy ra khi và chỉ khi x = y = z = 1.

Vậy maxA = 6 3 2.

Câu 37.

Tìm GTLN và GTNN của biểu thức: P = 2x2 - xy - y

2 với x, y thoả mãn điều

kiện sau: x2 + 2xy + 3y

2 = 4.

Lời giải.

Đưa về bài toán tìm P để hệ phương trình: 2 2

2 2

2x xy y p

x 2xy 3y 4

có nghiệm.

Hệ trên 2 2

2 2

8x 4xy 4y 4p (1)

px 2pxy 3py 4p (2)

. Lấy (1) - (2), ta có:

(8 - p)x2 - 2y(2 + p)x - (4 + 3p)y

2 = 0 (3)

- Nếu y = 0 => (8 - p)x2 = 0 <=> x = 0 hoặc p = 8 p 0;p 8.

- Nếu y 0 chia 2 vế pt (3) cho y2 ta có :

(8 - p)t2 - 2(2 + p)t - (4 + 3p) = 0 (4) với t =

y

x.

+ Nếu p = 8 thì t = - 7

5.

+ Nếu p 8: Phương trình (2) có nghiệm <=> ' = (2 + p)2 + (8 - p)(4 + 3p) > 0

<=> p2 - 12p - 18 < 0 <=> 6 - 3 6366 p . Dấu “=” có xảy ra.

Vậy min P = 6 - 3 6 , max P = 6 +3 6 .

35

Câu 38.

a) Cho a, b, c là độ dài 3 cạnh tam giác, chứng minh:

2 2 2

1 1 1 a + b + c + +

a + bc b + ac c + ab 2abc .

b) Cho biểu thức: A = x - 2 xy +3y - 2 x + 1. Tìm giá trị nhỏ nhất của A.

Lời giải.

a) Vì a, b, c là độ dài 3 cạnh của tam giác nên a, b, c > 0

Áp dụng BĐT Cô-si ta có:

a2 + bc

≥ 2a 2 2bc, b + ac 2b ac ; c + ab 2c ab .

Do đó 2 2 2

1 1 1 1 1 1 1 + + + +

a + bc b + ac c + ab 2 a bc b ac c ab

=

a +b b + c c + a + +

1 ab + bc + ca 1 a + b + c2 2 2 . . = 2 abc 2 abc 2abc

, đpcm.

Dấu bằng xẩy ra khi và chỉ khi a = b = c, tức là tam giác đã cho là tam giác đều.

b) Điều kiện x ≥ 0; y ≥ 0

Ta có: A = (x - 2 xy + y) + 2y - 2 x +1

2

= [ x - y - 2 x - y + 1] - 2 y + 2y

2 1 1

= x - y - 1 + (2y - 2 y + ) - 2 2

2 21 1 1

= x - y - 1 + 2 y 1 - - 2 2 2

9x =

x - y - 1 = 01 4A= -

12 2 y - 1 = 0 y = 4

Vậy minA = 1

2

Câu 39.

a) Cho x và y là 2 số thực thoả mãn x2 + y

2 = 4. Tìm giá trị lớn nhất của biểu

thức : A = xy

x + y + 2.

36

b) Cho x, y, z là 3 số thực dương thoả mãn x2 + y

2 + z

2 = 2. Chứng minh:

3 3 3

2 2 2 2 2 2

2 2 2 x + y + z + + + 3

x + y y + z z + x 2 xyz .

Lời giải.

a) Từ x2 + y

2 = 4 2xy = (x + y)

2 - 4 = (x + y + 2) (x + y - 2)

Vì x + y + 2 ≠ 0 nênxy x + y

= - 1x + y + 2 2

(1)

Áp dụng BĐT Bunhiacopski, ta có:

x + y ≤ 2 22 x + y x + y ≤ 2 2 (2)

Từ (1), (2) ta được: xy

2 - 1x + y + 2

. Dấu "=" 2 2

x 0, y 0

khi x = y x = y = 2

x + y = 4

.

Vậy maxA = 2 - 1 .

b) Vì x2 + y

2 + z

2 = 2 nên:

2 2 2 2 2 2 2 2 2

2 2 2 2 2 2 2 2 2 2 2 2

2 2 2 x + y + z x + y + z x + y + z + + = + +

x + y y + z z + x x + y y + z z + x

= 2 2 2

2 2 2 2 2 2

z x y + + + 3

x + y y + z x + z

Ta có x2 + y

2 ≥ 2xy

2 2

2 2

z z

x + y 2xy ,

Tương tự 2 2

2 2

x x

y + z 2yz ,

2 2

2 2

y y

x + z 2xz

Vậy 2

2 2

z

x + y+

2

2 2

x

y + z+

2

2 2

y + 3

x + z

2z

2xy +

2x

2yz +

2y

2xz + 3

3 3 3

2 2 2 2 2 2

2 2 2 x + y + z + + + 3

x + y y + z z + x 2xyz , đpcm.

Câu 40.

: 1) Tìm giá trị nhỏ nhất biểu thức: A = 2

5 - 3x

1 - x.

2) Cho a, b, c là độ dài 3 cạnh của tam giác. Chứng minh:

2 2 2 2 2 2 a + b + b + c + c + a 2 (a + b + c).

Lời giải.

37

1) Điều kiện: 1 - x2 > 0 - 1 < x < 1 2 - 3x > 0 A ≥ 0

Vậy A2 =

2 2

2 2

25 - 30x + 9x (3 - 5x)= +16 16

1 - x 1 - x .

Dấu bằng xẩy ra khi 3 - 5x = 0 3

x = 5

Vậy minA = 4.

2) Chứng minh: 2 2 2 2 2 2 a + b + b + c + c + a 2 (a + b + c) (1)

Sử dụng bất đẳng thức: 2 2 22(x y ) (x y) , ta có:

2 2 2 2 22(a + b ) (a b) 2. a + b a + b (2)

Tương tự, ta được: 2 22. b + c b + c (3) và

2 22. c + a c + a (4)

Lấy (2) + (3) + (4) theo từng vế và rút gọn, suy ra (1) đúng, đpcm.

Câu 41.

a) Cho a, b, c > 0 thoả mãn: 1 35 4c

+ 1 + a 35 + 2b 4c + 57

. Tìm giá trị nhỏ nhất của

A = a.b.c.

b) Giả sử a, b, c, d, A, B, C, D là những số dương và

a b c d

= = = A B C D

. Chứng minh rằng:

aA + bB + cC + dD = (a + b + c + d) (A +B + C + D)

Lời giải.

Ta có:

4c 1 35 35 + 2. > 0

4c + 57 1 + a 35 2b 1 + a 2b + 35

(1)

Mặt khác 1 4c 35 1 4c 35

- - 1 + a 4c + 57 35 + 2b 1 + a 4c + 57 35 + 2b

1 4c 35 2b - + 1 1 - = 1 +a 4c + 57 35 + 2b 35 + 2b

2b 1 57 57

+ 2.35 + 2b 1 + a 4c + 57 1 + a 4c + 57

> 0 (2)

Ta có: 1 4c 35

1 - 1 - + 1 + a 4c + 57 35 + 2b

38

a 57 35 35 . 57

+ 2.1 + a 4c + 57 35 + 2b 4c + 57 35 + 2b

> 0 (3)

Từ (1), (2), (3) ta có:

8abc 35 . 57

8 . 1 + a 4c + 57 2b + 35 1 + a 2b + 35 4c + 57

Do đó abc ≥ 35.57 = 1995.

Dấu “=” xảy ra khi và chỉ khi a = 2, b = 35 và c = 57

2.

Vậy min (abc) = 1995.

b) Đặt t = A B C D

= = = a b c d

A = ta, B = tb, C = tc, D = td.

t = A + B + C + D

a + b + c + d

Vì vậy 2 2 2 2aA + bB + cC + dD = a t + b t + c t + d t

= (a + b + c + d)A + B + C + D

t = (a + b + c + d)a + b + c + d

= (a + b + c +d)(A + B + C + D)

Câu 42.

Cho a, b, c [0; 2] và a + b + c = 3. Chứng minh a2 + b

2 + c

2 < 5.

Lời giải.

Vì a, b, c [0; 2] nên: (2 - a)(2 - b)(2 - c) > 0

<=> 8 - 4(a + b + c) + 2(ab + bc + ca) - abc > 0

<=> 2(ab + bc + ca) > 4(a + b + c) - 8 + abc

nên 2(ab + bc + ca) > 4 (vì a + b + c = 3 và abc 0)

Suy ra (a + b + c)2 - (a

2 + b

2 + c

2) > 4

<=> a2 + b

2 + c

2 5 (vì (a + b + c)

2 = 9)

Dấu “=” xẩy ra khi một trong 3 số a, b, c có một số bằng 2, một số bằng 0 và

một số bằng 1.

Câu 43.

Chứng minh rằng nếu phương trình x4 + ax

3 + bx

2 + ax +1 = 0 có nghiệm thì

5(a2 + b

2) ≥ 4.

39

Lời giải.

Giả sử x0 là một nghiệm của phương trình, dễ thấy 0x 0 .

Suy ra 2

0x + ax0 + b + 2

0 0

a 1 + = 0

x x

2

0 02

0 0

1 1 x + + a x + + b = 0

x x

Đặt x0 + 2 2

0 0 0 02

0 0

1 1 = y x + = y - 2 , y 2

x x 2

0 0 y - 2 = - ay - b

Áp dụng bất đẳng thức Bunhiacốpxki ta có:

2 22 2 2 2

0 0 0y - 2 = ay + b a + b y + 1 2 2

2 2 0

2

0

(y 2)a b

y 1

(1)

Ta chứng minh 2 2

0

2

0

(y 2) 4

y 1 5

(2)

Thực vậy: (2) 4 2 2 4 2

0 0 0 0 05(y 4y 4) 4(y 1) 5y 24y 16 0

2 2

0 0

45(y 4)(y ) 0

5 đúng với y 2 nên (1) đúng

Từ (1), (2) suy ra 2 2 2 24a + b 5(a + b ) 4

5 , đpcm.

Câu 44.

Cho a, b, c > 0 thỏa mãn 3a2 + 4b

2 7c

2. CMR:

3 4 7

a b c

Lời giải.

Chứng minh: 3 4 49

3 4a b a b

Thật vậy: 23 4 49

3 4 3 4 49 12 03 4

b a a b ab a ba b a b

Mặt khác, ta lại chứng minh được: 2 23 4 7 3 4a b a b

Do đó, 22 2

3 4 49 49 49 7

3 4 7.77 3 4a b a b cca b

Dấu bằng xảy ra khi a = b = c

Câu 45.

Giả sử x, y là hai số thực phân biệt thỏa mãn 2 2

1 1 2

1 1 1x y xy

40

Tính giá trị biểu thức 2 2

1 1 2

1 1 1P

x y xy

Lời giải.

2 2 2 2

2 22 2 2 2

2 2

2

1 1 2 1 1 1 10

1 1 1 1 1 1 1

0 1 1 01 1 1 1

1 0 1 ( ) 2

x y xy x xy y xy

xy y xy xxy y y xy x x

x xy y xy

x y xy xy vi x y S

Câu 46. (Trích đề tuyển sinh Chuyên sư phạm Hà Nội)

Các số thực không âm 1 2 3 9, , ,....,x x x x thỏa mãn

1 2 3 9

1 2 3 9

.... 10

2 3 .... 9 18

x x x x

x x x x

Chứng minh rằng : 1 2 3 91.19 2.18 3.17 .... 9.11 270x x x x

Lời giải.

1 2 3 9

1 2 3 9

1 2 3 9

1 2 3 9

1 2 3 9

1 2 3 9 2 3 4 8

1

9 ... 90

9 ... 9019 29 39 ... 99 270

10 2 3 ... 9 180

1.19 2.18 3.17 ... 9.11

(19 29 39 ... 99 ) 7 12 15 ... 7 270

9

" "

x x x x

x x x xx x x x

x x x x

Mat khac

x x x x

x x x x x x x x

x

Dau xay ra

9

2 3 8

1

... 0

x

x x x

Câu 47.

Cho các số dương a,b,c,d . Chứng minh rằng trong 4 số

2 2 2 21 1 1 1 1 1 1 1;b ;c ;da

b c c d c d a b Có ít nhất một số không nhỏ

hơn 3.

Lời giải.

Giả sử cả bốn số đều nhỏ hơn 3 thì

2 2 2 21 1 1 1 1 1 1 1b c d 3P a

b c c d c d a b

41

Mặt khác

2 2 2 2 2 2 2 2

22 2 2 2

2 2

3

1 1 1 1 1 1 1 1 1 1 1 1b c d 2

1 1 1 1 44 ;

16 16 16 163 . . 12

4 4

P a a b c db c c d c d a b a b c d

Do a b c d a b c da b c d a b c d

a b c d a b c dP

a b c d a b c d a b c d a b c d

= 12.

Trái điều giả sử suy ra có ít nhất một số không nhỏ hơn 3.

Câu 48. (Trích đề thi HSG huyện Cẩm Thủy – Thanh Hóa năm học 2011 –

2012)

Cho 0 < a, b,c <1 .Chứng minh rằng :

Lời giải.

Do a <1 <1 và b <1. Nên

Hay . Mặt khác 0 <a,b <1 ;

Vậy

Tương tự ta có

(đpcm)

Câu 49. (Trích đề thi học sinh giỏi lớp 9)

a. Tìm giá trị nhỏ nhất của biểu thức 12

6832

2

xx

xxA

b. Chứng minh rằng với mọi số thực a,b,c ta có cabcabcba 222

Lời giải.

Viết được 2)1(

)2(2

12

442422

2

2

22

x

x

xx

xxxxA

Lập luận min A = 2 khi x-2= 0 => x= 2

b. biến đổi cabcabcba 222

<=> 2a2+2b

2+2c

2≥2ab+2bc+2ca

<=> a2-2ab+b

2+b

2-2bc +c

2 +c

2 -2ca+a

2 ≥0

accbbacba 222333 3222

2a 2 2 21 . 1 0 1 0a b a b a b

baba 221 32 aa 3bb

332 baab

baba 233 1

acca

cbcb

233

233

1

1

accbbacba 222333 3222

42

<=> (a-b)2+(b-c)

2+(c-a)

2 ≥ 0 (luôn đúng)

Vậy BĐT đã được chứng minh.

Câu 50. (Trích đề thi học sinh giỏi huyện Kim Thành năm học 2012 – 2013)

Cho a; b; c là các số thuộc đoạn thỏa mãn: a2 + b

2 + c

2 = 6 hãy chứng

minh rằng: a + b + c 0.

Lời giải.

Do a; b; c thuộc đoạn nên a + 1 0; a – 2 0 nên (a + 1)(a – 2) 0

Hay: a2 – a – 2 0 a

2 a + 2

Tương tự: b2 b + 2; c

2 c + 2

Ta có: a2 + b

2 + c

2 a + b + c + 6 theo đề bài: a

2 + b

2 + c

2 = 6 nên: a + b + c 0

Bài 51. (Trích đề thi HSG TP. Thanh Hóa năm 2016 – 2017)

Cho x, y là 2 số thực thoả mãn: x2 + 2y

2 + 2xy + 7x + 7y + 10 = 0.

Tìm giá trị nhỏ nhất và giá trị lớn nhất của biểu thức: A = x + y + 1.

Lời giải.

Ta có: x2 + 2y

2 + 2xy + 7x + 7y + 10 = 0.

2 2

2

7( ) 10

( 2)( 5) 0

4 1 1

x y x y y

x y x y y

x y

* x + y + 1 = - 4 khi x = - 5; y = 0

* x + y + 1 = - 1 khi x = - 2; y = 0

Vậy Amin = - 4 khi x= - 5; y = 0

Amax = - 1 khi x = -2; y = 0

Bài 52. (Trích đề thi HSG TP. Thanh Hóa năm 2016 – 2017)

Cho a, b, c > 0. Chứng minh rằng:

a b c a b c

a b b c c a b c c a a b

Lời giải.

* Vì a, b, c > 0 nên 1a a a c

a b a b a b c

.

Tương tự: ;b b a c c b

b c a b c c a a b c

1;2

1;2

43

2a b c

a b b c c a

(1)

* Ta có: ( )

a a

b c a b c

Vì a, b, c > 0 nên theo bất đẳng thức Cô- si ta có:

( )( ) 0

2

2 1

( )

a b ca b c

a b c a b c

2 2

( )

a a a a

a b c a b c b ca b c

Tương tự: 2 2

;b b c c

a b c a c a b c b a

2a b c

b c c a a b

Dấu „ =” xảy ra khi a = b + c; b = c + a; c = a +b

tức là a = b = c (vô lý).

2a b c

b c c a a b

(2)

Từ (1) (2) ta có đpcm.

Câu 53. (Trích đề thi HSG tỉnh Hải Dương năm học 2013 – 2014)

Cho a, b, c là các số thực dương thỏa mãn 2 6 2 7ab bc ac abc . Tìm giá trị

nhỏ nhất của biểu thức 4 9 4

2 4

ab ac bcC

a b a c b c

.

Lời giải.

Từ gt : 2 6 2 7ab bc ac abc và a,b,c > 0

Chia cả hai vế cho abc > 0 2 6 2

7c a b

đặt 1 1 1

, ,x y za b c

, , 0

2 6 2 7

x y z

z x y

Khi đó 4 9 4

2 4

ab ac bcC

a b a c b c

4 9 4

2 4x y x z y z

44

4 9 42 4 (2 4 )

2 4C x y x z y z x y x z y z

x y x z y z

2 22

2 3 22 4 17 17

2 4x y x z y z

x y x z y z

Khi 1

x ,y z 12

thì C = 17

Vậy GTNN của C là 17 khi a =2; b =1; c = 1

Bài 54. (Trích đề thi HSG lớp 9 huyện Ba Vì năm học 2017 – 2018)

Cho biểu thức: 2 215 8 2017A x y xy x y

Tìm giá trị nhỏ nhất của biểu thức A.

Lời giải.

2 215 8 2017A x y xy x y

⇒ 2 24 4 60 4 32 4 8068A x y xy x y

2 2 2 2(4 4 ( 8) ( 8) ) 60 4 ( 8) 8068x x y y y y y

2 2 2(4 2( 8).(2 ) ( 8) ) 59 12 8004x y x y y y

= 2 2 26 6 36

(2 8) 59(( 2. . ( ) ) 800459 59 59

x y y y

= 2 26 47200

(2 8) 59( )59 59

x y y ≥ 473200

59

Do đó: A ≥ 118050

59 . Dấu bằng xảy ra khi: 2x + y + 8 = 0 và y =

6

59

Hay x = 239

59 và y =

6

59

Vậy minA = 118050

59 khi x =

239

59 và y =

6

59

Câu 55. (Trích đề HSG tỉnh Nghệ An năm học 2010 – 2011)

a) Cho x > 0, y > 0, z > 0 và 1 1 1

4x y z .

Chứng minh rằng: 1 1 1

12x+y+z x 2y z x y 2z

b) Cho x > 0, y > 0, z > 0 thỏa mãn 2011 2011 2011x y z 3 .

Tìm giá trị lớn nhất của biểu thức: 2 2 2M x y z

Lời giải.

45

Áp dụng bất đẳng thức

1 1 4

x y x y

(với x,y > 0)

Ta có:

1 1 1 1( )

2x+y+z 4 2x y z

;

1 1 1

y z 4y 4z

Suy ra:

1 1 1 1 1( )

2x+y+z 4 2x 4y 4z

(1)

Tương tự:

1 1 1 1 1( )

x+2y+z 4 4x 2y 4z

(2)

1 1 1 1 1( )

x+y+2z 4 4x 4y 2z

(3)

Từ (1),(2),(3)

1 1 1 1 1 1 1( )

2x+y+z x+2y+z x+y+2z 4 x y z

1 1 11

2x+y+z x+2y+z x+y+2z

Dấu "=" xảy ra

3x y z

4

Áp dụng bất đẳng thức Cô-Si cho 2011 2011x ,x và 2009 số 1 ta có:

2011 2011 2 20112011x x 1 1 ... 1 2011 (x )

2009

2011 22x 2009 2011x (1)

Tương tự: 2011 2

2y 2009 2011y (2)

2011 22z 2009 2011z (3)

Từ (1), (2), (3)

2011 2011 20112 2 2 2(x y z ) 3.2009x y z

2011

2 2 2x y z 3

Giá trị lớn nhất của M là 3 khi và chỉ khi x = y = z = 1

Câu 56. (Trích đề thi HSG tỉnh Vĩnh Phúc năm học 2011 – 2012)

Cho , , , a b c d là các số thực thỏa mãn điều kiện:

2012abc bcd cda dab a b c d

Chứng minh rằng: 2 2 2 21 1 1 1 2012a b c d .

46

Lời giải.

Ta có: 2

2012 abc bcd cda dab a b c d

2

1 1ab c d cd a b

2 2 2 2

1 1ab a b cd c d

2 2 2 2 2 2 2 2 2 2 2 21 1 1 1 1 1a b a b c d c d a b c d

Suy ra 2 2 2 21 1 1 1 2012a b c d

Câu 57. (Trích đề thi HSG tỉnh Phú Thọ năm 2012 – 2013)

Cho a,b,c là các số thực dương. CMR:

Lời giải.

Dự đoán a=b=c tách mẫu để a+c=b+c=2b

Tacó áp dụng BĐT

Tương tự

Từ (1) (2) (3)

Dấu “=” xảy ra khi a=b=c

Câu 58. (Trích đề thi HSG tỉnh Nghệ An 2010 – 2011)

Tìm giá trị nhỏ nhất của biểu thức: 2

4x+3A

x 1

Lời giải.

3 2 3 2a 3 2 6

ab bc ca a b c

a b c b c c a b

zyxzyxzyxzyx

111

9

119

111)(

1 1 1 1(1)

3 2 ( ) ( ) 2 9 2 9 2

ab ab ab ab ab a

a b c a c b c b a c b c b a c b c

1 1 1 1(2)

2 3 ( ) ( ) 2 9 2 9 2

1 1 1 1(2)

3 2 ( ) ( ) 2 9 2 9 2

bc bc bc bc bc b

a b c a b a c c a c b c b a b b c

ac ac ac ac ac c

a b c a b b c a a b b c a a b b c

629

1 cbacba

ca

abbc

cb

acab

ba

bcacP

47

Ta có: 2

2 2

4x+3 x 4x+4A 1

x 1 x 1

2

2

(x 2)A 1 1

x 1

Dấu "=" xảy ra x 2 0 x 2

Vậy minA 1 khi x = -2

Câu 59. (Trích đề thi HSG tỉnh Hòa Bình năm học 2011 – 2012)

T×m x ®Ó biÓu thøc ®¹t gi¸ trÞ nhá nhÊt.

Lời giải.

Ta có:

Từ đó t×m ®ược gi¸ trÞ nhá nhÊt cña biÓu thøc A b»ng - 1 khi x = 0.

Câu 60. (Trích đề thi HSG tỉnh Ninh Bình Năm 2012 – 2013)

1. Cho các số thực m, n, p thoả mãn: n2 + np + p

2 = 1 - . Tìm giá trị lớn nhất

và nhỏ nhất của biểu thức S = m + n + p.

2. Cho các số thực dương a, b, c thoả mãn abc = 1. Chứng minh rằng:

.

Đẳng thức xảy ra khi nào?

Lời giải.

Áp dụng BĐT Cauchy - Schwarz ta có:

1

1

xA

x

21

1A

x

23m

2

2 2 2

a b c 1

(ab a 1) (bc b 1) (ca c 1) a b c

2 2 2

2 2 2

2

( ) ( ) ( )1 1 1

( 1) ( 1) ( 1)

a b ca b c

ab a bc b ca c

a b c

ab a bc b ca c

2. .

. 1

a bc b c b

ab bc abc bc bc b cab bc b

21

11 1 1

b bc

b bc bc b bc b

48

Bài 61. (Trích đề thi HSG huyện Thanh Oai 2016 – 2017)

Tìm giá trị nhỏ nhất của biểu thức

(Trong đó a, b, c là độ dài 3 cạnh của 1 tam giác)

Lời giải.

Đặt x = b + c – a, y = a + c – b, z=a + b – c thì `

Ta có

Vậy

Dấu đẳng thức xảy ra khi

Câu 62. (Trích đề thi chọn HSG huyện Nghi Xuân năm 2013 – 2014)

a. Cho các số x; y; z không âm, không đồng thời bằng 0 và thỏa mãn:

.

Tìm giá trị nhỏ nhất của biểu thức:

b. Cho các số dương x, y, z thoả mãn điều kiện: xy + yz + zx = 671.

Chứng minh rằng:

Lời giải.

a) Trước tiên ta chứng minh bất đẳng thức: Với a, b, c R và x, y, z > 0 ta có

2 2 2

1

( 1) ( 1) ( 1)

a b c

ab a bc b ca c a b c

cba

c

bca

b

acb

aP

1694

0,, zyx

2

2

2

yxc

zxb

yza

zcba

ybca

xacb

2 2 9 9 8 8

2

2 9 2 8 9 82 9 2 16 2 36 26

2 2

y z z x x yP

x y z

y x z x z y

x y x z y z

z

y

y

z

z

x

x

z

y

x

x

y

8

2

9

82

2

92

22

22

22

89

82

94

yz

xz

xy

yz

xy

zx

3

4

2

3

2

1 1 11

x 1 y 2 z 3

1P x y z

x y z

2 2 2

1

2013 2013 2013

x y z

x yz y zx z xy x y z

49

(*) Dấu “=” xảy ra

Thật vậy, với a, b R và x, y > 0 ta có (**)

(luôn đúng)

áp dụng bất đẳng thức (**) ta có

Dấu “=” xảy ra

Áp dụng với a = b= c = 1 ta có

=> =>

( Có thể chứng minh BĐT trên nhờ áp dụng BĐT Bunhicopski )

Áp dụng BĐT Côsi cho 2 số dương ... ta có:

Dấu “=” xảy ra khi và chỉ khi các số x; y; z không âm và không đồng thời bằng

0 thỏa mãn : ( Thỏa mãn)

Vậy Min x = 2; y = 1; z = 0.

b) Áp dụng bất đẳng thức (*) ta có

(1)

Chú ý: xy + yz + zx = 671 nên

22 2 2 a b ca b c

x y z x y z

a b c

x y z

22 2 a ba b

x y x y

22 2a y b x x y xy a b

20bx ay

2 22 2 2 2a b a b ca b c c

x y z x y z x y z

a b c

x y z

2

1 1 11 1 1 91

1 2 3 6 6x y z x y z x y z

6 9x y z 3x y z

1 8(x y z) x y z 1 8.3 x y z 1 10P x y z 2. .

x y z 9 9 x y z 9 9 x y z 3

x y z 3

x y z 1

9 x y z

x 1 y 2 z 3

1 1 11

x 1 y 2 z 3

x 2

y 1

z 0

10P

3

2 2 22013 2013 2013

x y zVT

x yz y zx z xy

2 2 2

2 2 22013 2013 2013

x y z

x x yz y y zx z z xy

2

3 3 3 3 2013

x y z

x y z xyz x y z

50

= , và

Chứng minh:

(2)

= = (3)

Từ (1) và (3) ta suy ra

Dấu “=” xảy ra x = y = z = .

Câu 63. (Trích đề thi HSG huyện Thanh Oai 2014 – 2015)

b) Cho x, y, z là các số dương thoả mãn 1 1 1

6x y y z z x

.

Chứng minh rằng: 1 1 1 3

3 3 2 3 2 3 2 3 3 2x y z x y z x y z

.

Bài làm.

Ta có:

1 1 1 1 1

3 3 2 2 2 4 2 2x y z x y z x y z x y z x y z

1 1 1 1 1 1 1 1 1

4 4 4x y x z x y y z x y x z x y y z

1 2 1 1

16 x y x z y z

Tương tự: 1 1 2 1 1

3 2 3 16x y z x z x y y z

1 1 2 1 1

2 3 3 16x y z y z x y x z

Cộng vế theo vế, ta có:

1 1 1 1 4 4 4

3 3 2 3 2 3 2 3 3 16x y z x y z x y z x y x z y z

2 2013x x yz 2 1342 0x x xy zx 2 2013 0y y zx

2 2013 0z z xy

3 3 3 2 2 23x y z xyz x y z x y z xy yz zx

2

3x y z x y z xy yz zx

3 3 3 3 2013x y z xyz x y z 2

3 2013x y z x y z xy yz zx

2

3.671 2013x y z x y z

3x y z

2

3

1x y zVT

x y zx y z

2013

3

51

4 1 1 1 1 3.6

16 4 2x y x z y z

Câu 64. (Trích đề thi HSG huyện Thanh Oai 2014 – 2015)

Tìm giá trị nhỏ nhất của

A = y

zx

x

yz

z

xy với x, y, z là các số dương và x

2 + y

2 + z

2 = 1

Lời giải.

A = y

zx

x

yz

z

xy Nên A

2 = 2

2

22

2

22

2

22

y

xz

x

zy

z

yx( vì x

2+y

2+z

2 =1)

= B +2

Áp dụng bất đẳng thức Cô si cho 2 số dương ta có

2

22

2222

2

22

2

22

2yxz

zyyx

x

zy

z

yx

Tương tự 2

2

22

2

22

2zy

xz

x

zy ; 2

2

22

2

22

2xy

xz

z

yx

Cộng vế với vế ta được 2B 2 1 B

Do đó A2 = B +2 3 nên A 3

Vậy Min A = 3 x=y=z= 3

3

Bài 18. (Trích đề thi HSG TP. Hải Phòng 2016 – 2017)

Cho ba số thực a, b, c dương. Chứng minh rằng:

.

Lời giải.

Với x là số dương, áp dụng bất đẳng thức Cauchy ta có:

Dấu “ =” xảy ra khi x = 2

Áp dụng bất đẳng thức (*) ta được:

3 3 3

3 3 33 3 3

a b c1

a b c b c a c a b

2 2

3 2 x 1 x x 1 x 2x 1 x 1 x x 1

2 2

3 2

1 2(*)

x 1 x 2

52

Suy ra:

Tương tự ta có:

Cộng vế với vế của ba bất đẳng thức (1), (2) và (3) ta được:

Dấu “=” xảy ra khi a = b = c.

Câu 65. (Trích đề thi HSG tỉnh Bắc Giang năm 2012 – 2013)

Cho ba số dương ,a b và c thoả mãn 1abc . Chứng minh rằng:

2 2 2 2 2 2

1 1 1 1

2 3 2 3 2 3 2a b b c c a

.

Lời giải.

Ta có: 2 2 2 2 22 3 ( ) ( 1) 2 2 2 2a b a b b ab b

Tương tự: 2 22 3 2 2 2b c bc c , 2 22 3 2 2 2c a ac a

Suy ra:

2 2 2 2 2 2

1 1 1 1 1 1 1( )

2 3 2 3 2 3 2 1 1 1

1 1 1 1 1( ) .

1 1 12 1 21 1

a b b c c a ab b bc c ac a

ab ba

a ab b

Câu 66. (Trích đề thi HSG tỉnh Quảng Bình Năm 2012 – 2013)

Cho c¸c sè thùc d¬ng a, b, c tho¶ m·n a + b + c =6. Chứng minh rằng:

3 2

3 3 2 23 2

a 1 2 2a

a b c b c 2ab c b c1 2

a a

3 2 2

3 2 2 22 2 23

a 2a a(1)

a b c2 b c 2aa b c

3 2

3 2 2 23

b b(2)

a b cb a c

3 2

3 2 2 23

c c(3)

a b cc a b

3 3 3

3 3 33 3 3

a b c1

a b c b a c c a b

53

. DÊu ®¼ng thøc x¶y ra khi nµo?

Lời giải.

Đặt. ( ) (x, y, z >0)

Dấu bằng xảy ra khi x=y=z, suy ra a=3, b=2, c=1.

Câu 67. (Trích đề thi HSG T.P Bắc Giang năm 2016 – 2017)

Cho a, b, c>0 thỏa mãn abc=1 . Chứng minh

1 1 1 3

22 2 2ab a bc b ca c

Lời giải.

Ta có 2 2 2 22 2 23 ... 0x y z x y z x y y z x z

2 2 2 23x y z x y z nên với x,y,z>0 ta có

2 2 23x y z x y z , áp dụng ta có

1 1 1 1 1 13

2 2 22 2 2 ab a bc b ca cab a bc b ca c

-Với x,y>0 ta có 2 1 1 1 1

2 44

x y xy x y xyx y x y

áp dụng ta có

1 1 1 1

2 1 1 1 ( 1) ( 1)

1 1 1 1 1 1 1

4 ( 1) 1 4 ( 1) 1 4 1 1

ab a ab a ab abc a ab c a

abc c

ab c a ab c a c a

Vây ta có 1 1 1

2 4 1 1

c

ab a c a

Tương tự ta có 1 1 1

2 4 1 1

a

bc b a b

;

1 1 1

2 4 1 1

b

ca c b c

nên

5 4 36

1 2 3

b c c a a b

a b c

1; 2; 3x a y b z c

2 . 2 . 2 . 6

y z z x x y y x x z y zVT

x y z x y z x z y

y x z x y z

x y x z z y

54

1 1 13

2 2 2

1 1 1 1 33

4 1 1 1 1 1 1 2

ab a bc b ca c

c a b

c a a b b c

Vậy 1 1 1 3

22 2 2ab a bc b ca c

dấu “=” có khi a=b=c=1

Câu 68. (Trích đề thi HSG tỉnh Quảng Bình năm 2012 – 2013)

Cho các số thực thoả mãn: . Chứng minh:

Lời giải.

Ta có

Câu 69. (Trích đề thi vào lớp 10 chuyên tỉnh Ninh Bình năm 2013 – 2014)

Cho x, y là các số thực thoả mãn . Tìm giá trị lớn

nhất và giá trị nhỏ nhất của biểu thức .

Lời giải.

Với thì ta có

;

Vậy minC = 1 khi và ; maxC = 3 khi và .

Câu 70 . (Trích đề thi HSG tỉnh Thanh Hóa)

a,b,c ab bc ca 2 4 4 4 4a b c

3

4 4 4 2 2 2 2 2 2a b c a b b c c a , a,b,c

24 4 22 24 2 23 3( , , ,) ba b c a aab b cc c

22 2 2 2 2 23 a b b c c a ab bc ca , a,b,c

24 4 4 1 4

a b c ab bc ca3 3

2

2 2 2 2x x 2y 3 y 2 1

2 2C x y

22 2 2 2 4 2 2 2 4 2

4 2 2 4 2 2 2

22 2 2 2 2

x x 2y 3 y 2 1 x 2x y 3x y 4y 4 1

x 2x y y x y x

x y x y x

4 3 0

4 3 0 x

2 2x y C 22 2C 4C 3 0 C 4C 4 1 C 2 1

C 2 1 1 C 2 1 1 C 3

2 2

x 0 x 0C 1

y 1x y 1

2 2

x 0x 0C 3

x y 3 y 3

x 0 y 1 x 0 y 3

55

Giải hệ phương trình 4 4 4

1x y z

x y z xyz

.

Lời giải.

Ta có:

=

= =

= xyz (x + y + z) = xyz ( vì x + y + z = 1).

Dấu bằng xảy ra

Vậy nghiệm của hệ phương trình là:

Câu 71. (Trích đề thi HSG huyện Phú Lộc)

Tìm giá trị lớn nhất, giá trị nhỏ nhất của biểu thức: 2

1

1

xA

x x

Lời giải.

Tìm giá trị lớn nhất, giá trị nhỏ nhất của biểu thức:

2

1

1

xA

x x

Ta có: 2

2 1 31 0,

2 4x x x x

2 2 2

2 2 2

1 11 1

1 1 1

x x x x xA

x x x x x x

(vì

2

20,

1

xx

x x

)

Đẳng thức xảy ra khi x = 0, suy ra: maxA = 1 khi x = 0

2 2

2 2 2

4 4 11 3 33

1 1 1

x x x xx xA A

x x x x x x

2

2

2 = 1 1

1

x

x x

(vì

2

2

20,

1

xx

x x

)

Suy ra:1

3A , đẳng thức xảy ra khi 2 0 2x x

Suy ra: minA = 1

3 , khi 2x

Câu 72. (Trích đê thi HSG huyện Phú Lộc)

4 4 4 4 4 44 4 4

2 2 2

x y y z z xx y z

2 2 2 2 2 2x y y z z x

2 2 2 2 2 2 2 2 2 2 2 2

2 2 2

x y y z y z z x z x x yxyyz yzzx zxxy

1

1 3

x y zx y z

x y z

1 1 1; ;

3 3 3x y z

56

Cho a, b, c là ba số dương thỏa mãn: a + b + c = 1. Chứng minh rằng:

2c ab a bc b ac

a b b c a c

Lời giải.

Vì a + b + c = 1 nên

c + ab = c(a + b + c) + ab = (c + a)(c + b)

a + bc = a(a + b + c) + bc = (b + a)(b + c)

b + ac = b(a + b + c) + ac = (a + b)(a + c)

nên BĐT cần chứng minh tương đương với:

2 2 2

2

2

c a c b b a b c a b a c

a b a c b c

c a c b b a b c a b a c

a b a c b c

Mặt khác dễ thấy: 2 2 2x y z xy yz zx , với mọi x, y, z (*)

Áp dụng (*) ta có:

2VT b c a b c a

Dấu “=” xảy ra khi và chỉ khi a = b =c = 1

3 đpcm

Câu 73. (Trích đề thi HSG tỉnh Hà Nam)

Cho a, b, c là ba số thực dương thỏa mãn: 3a b c . Chứng minh rằng:

2 2 2

1 1 13

1 1 1

a b c

b c a

Lời giải.

Theo bất đẳng thức Cauchy ta có: 21 2b b nên:

2 2

2 2

1 ( 1) ( 1)( 1) ( 1) 1

1 1 2 2

a b a b a ab ba a a

b b b

2

11

1 2

a ab ba

b

Tương tự ta có:

2

11

1 2

b bc cb

c

(2)

2

11

1 2

c ca ac

a

(3)

Cộng vế theo vế (1), (2) và (3) ta được:

57

2 2 2

1 1 13

1 1 1 2

a b c a b c ab bc ca

b c a

(*)

Mặt khác: 2

3( ) 9 02

a b c ab bc caab bc ca a b c

Nên (*) 2 2 2

1 1 13

1 1 1

a b c

b c a

(đpcm)

Dấu "=" xảy ra khi và chỉ khi 1a b c

Câu 74. (Trích đề thi HSG huyện Bình Giang)

Cho x 3y 1 . Tìm giá trị nhỏ nhất của biểu thức: 2 2C x y

Lời giải.

Do x 3y 1 , đặt x 3y 1 a với a 0 x = 1 + a – 3y, thay vào biểu thức C:

2 2C 10y 6ay 6y a 2a 1

2

23 1 1 1C 10 y a 1 a 2a

10 10 10 10

.

1min C

10 khi:

33 3 3 y

y a 1 0 y y 1010 10 10

1a 0 a 0 x 3y 1 x

10

Câu 75.

Tìm giá trị nhỏ nhất của 2 2A x 4x 12 x 2x 3 .

Lời giải.

. Tập xác định : 2

2

x 4x 12 0 (x 2)(6 x) 0

1 x 3

(x 1)(3 x) 0x 2x 3 0

(1)

Xét hiệu : (- x2 + 4x + 12)(- x

2 + 2x + 3) = 2x + 9. Do (1) nên 2x + 9 > 0 nên A

> 0.

Xét : 2

2

A (x 2)(6 x) (x 1)(3 x) . Hiển nhiên A2 ≥ 0 nhưng dấu “ = ”

không xảy ra (vì A > 0). Ta biến đổi A2 dưới dạng khác :

A2 = (x + 2)(6 – x) + (x + 1)(3 – x) - 2 (x 2)(6 x)(x 1)(3 x) =

= (x + 1)(6 – x) + (6 – x) + (x + 2)(3 – x) – (3 – x) - 2 (x 2)(6 x)(x 1)(3 x)

58

= (x + 1)(6 – x) + (x + 2)(3 – x) - 2 (x 2)(6 x)(x 1)(3 x) + 3

= 2

(x 1)(6 x) (x 2)(3 x) 3 .

A2 ≥ 3. Do A > 0 nên min A = 3 với x = 0.

Câu 76.

Tìm GTNN, GTLN của : 2 2a) A 2x 5 x b) A x 99 101 x

Lời giải.

a) Điều kiện : x2 ≤ 5.

* Tìm giá trị lớn nhất : Áp dụng bất đẳng thức Bunhiacôpxki :

A2 = (2x + 1. 2

5 x )2 ≤ (2

2 + 1

1)(x

2 + 5 – x

2) = 25 A

2 ≤ 25.

2

2 2 2

22

x 0x5 x

A 25 x 4(5 x ) x 22

x 5 x 5

.

Với x = 2 thì A = 5. Vậy max A = 5 với x = 2.

* Tìm giá trị nhỏ nhất : Chú ý rằng tuy từ A2 ≤ 25, ta có – 5 ≤ x ≤ 5, nhưng

không xảy ra

A2 = - 5. Do tập xác định của A, ta có x

2 ≤ 5 - 5 ≤ x ≤ 5 . Do đó : 2x ≥ -

2 5 và

2

5 x ≥ 0. Suy ra :A = 2x + 2

5 x ≥ - 2 5 . Min A = - 2 5 với x = - 5

b) Xét biểu thức phụ | A | và áp dụng các bất đẳng thức Bunhiacôpxki và

Cauchy :

2 2 2

2 2

A x 99. 99 1. 101 x x (99 1)(99 101 x ) x .10. 200 x

x 200 x10. 1000

2

2

2

2 2

x 101

99 99A 1000 x 10

1 101 x

x 200 x

. Do đó : - 1000 < A < 1000.

min A = - 1000 với x = - 10 ; max A = 1000 với x = 10.

Câu 77.

59

Cho a,b,c là 3 số thực dương thõa mãn: a2 + 2b

2 3c

2 . CMR:

cba

321

Lời giải.

Với a, b, c là các số dương ta có:

(+) 1 2 9

+ (1) (a + 2b)(b + 2a) 9a + 2ba b

ab

2 2 22a - 4ab + 2a 0 2(a - b) 0 (đúng). Dấu bằng xảy ra khi a = b

(+) 2 2 2 2 2a + 2b 3( 2 )(2) ( 2 ) 3( 2 )a a a b a b

2 22 0 2( ) 02a - 4ab+2a a -b (đúng). Dấu bằng xảy ra khi a = b

(+) Từ (1) và (2) suy ra 2 2

1 2 9 9 3+

3(a + 2b )a b a+2b c (do 2 2 2+ 2 3a b c ).

Suy ra 1 2 3

+a b c

. Dấu bằng xảy ra khi a =b = c

Câu 78.

Cho các số thực dương x, y, z thỏa mãn xyz = 1.

Tìm giá trị lớn nhất của biểu thức 1 1 1

Q = + +x + y +1 y + z +1 z + x +1

Lời giải.

Với x, y, z là các số dương thỏa mãn xyz = 1 ta đặt x = a3, y = b

3, z = c

3 abc =

1

Khi đó ta có:

3 3 2 2x + y +1 = a + b + abc = a + b a - ab + b + abc a + b ab + abc = ab( a + b + c)

Tương tự: y + z +1 bc(a + b +c)

z + x +1 ca(a + b +c)

1 1 1 abc abc abcQ = + + + + 1

x + y +1 y + z +1 z + x +1 ab(a + b + c) bc(a + b + c) ca(a + b + c)

Vậy GTLN của Q = 1 khi a = b = c, hay x = y = z =1

Câu nàu la anh em với đề thi HSG lớp 9 huyện H.Hóa 2009 - 2010

Câu 79.

Cho hai sè d­¬ng x, y tho¶ m·n ®iÒu kiÖn: x + y = 2. Chøng minh: x2y2(x2+y2)

2.

Lời giải.

60

x2y2(x2+y2) = 1

2xy. [2xy.(x2 + y2)]

1

2xy.

22 22

2

x xy y

= 1

2 xy.

22( )

2

x y

=

2xy 22

2

x y

= 2 (Áp dung Cô si cho 2 số dương và x + y = 2 ).

Câu 80. Tìm giá trị nhỏ nhất của biểu thức:

2 24 2 – 6 –10 – 32D x y xy y x y .

Lời giải.

2 2 2

2 2

2 2

D - 2 – 2.5( - ) 25 3 6 7

– – 2.5 25 3 2 1 4

– – 5 3 –1 4 4

x xy y x y y y

x y x y y y

x y y

MinD = 4 khi 5 0 6

1 0 1

x y x

y y

Bài 81.

Với a, b, c là các số dương thỏa mãn điều kiện 2a b c . Tìm giá trị lớn

nhất của biểu thức Q 2a bc 2b ca 2c ab

Lời giải.

Ta có a+b+c=2 nên 2a+bc=(a+b+c)a+bc = (a+b)(a+c)

Áp dụng bất đẳng thức Cosi với 2 số dương u a b và v = a + c, ta có:

22 ( )( )

2 2

a b a c a b ca bc a b a c

(1)

Tương tự 2

22

b a cb ac

(2);

22

2

c a bc ab

(3)

Cộng các bđt (1), (2), (3) ta được:

2 2 22 2 2

2 2 2

2 2 2 2( ) 4

a b c b a c c a bQ a bc b ca c ab

Q a bc b ca c ab a b c

61

Dấu "=" xảy ra khi a = b = c =2

3

Vậy Max Q = 4 khi a = b = c =2

3.

Bài 82.

a) Với a, b là các số dương. Chứng minh rằng: a b 4

ab a b

b) Cho các số thực dương x,y,z thỏa mãn x y z 4 .

Chứng minh rằng: 1 1

1xy xz

Lời giải.

a. Với a,b dương nên ta có:

2

2 a b 4ab a b 4a b 4ab

a b .ab a b .ab ab a b

Dấu “=” xảy ra khi a = b

b. Áp dụng bất đẳng thức trên ta có :

1 1 4 1 1 4

xy xz xy xz xy xz x(y z)

Mà x + y + z = 4 nên y + z = 4 – x > 0

2 2

1 1 4 1 1 4 1 1 4

xy xz x(4 x) xy xz x 4x 4 4 xy xz (x 2) 4

(*)

Vì y + z = 4 – x > 0 nên x.(4-x) > 0 . Suy ra 24 (x 2) 4 0

Do đó 2

41

(x 2) 4

(**)

Từ (*) và (**) suy ra 1 1

1xy xz

Dấu “=” xảy ra khi

x 2x 2

xy xzy z 1

x y z 4

(thoả mãn điều kiện x,y,z>0)

Câu 83.

Cho các số dương x, y, z thỏa mãn: x + y + z = 1.

62

Chứng minh rằng:

Lời giải.

Câu 84.

Cho x,y,z là ba số thực dương có tổng bằng 3.Tìm giá trị nhỏ nhất của biểu

thức

2 2 23( ) 2P x y z xyz .

Lời giải.

Ta c ó:

23 ( ) 2( ) 2

3 9 2( ) 2

27 6 ( ) 2 ( 3)

P x y z xy yz zx xyz

xy yz zx xyz

x y z yz x

2

3 2

( )27 6 (3 ) ( 3)

2

1( 15 27 27)

2

y zx x x

x x x

63

Xét hàm số 3 2( ) 15 27 27f x x x x , với 0<x<3

, 21

( ) 3 30 27 09

xf x x x

x

Từ bảng biến thiên suy ra MinP=7 1x y z .

Câu 85.

Cho a,b, c dương và a2+b

2+c

2=3. Tìm giá trị nhỏ nhất của biểu thức

3 3 3

2 2 23 3 3

a b cP

b c a

Lời giải.

Ta có:

3 3 2 6 2

3

2 2

3 33

16 64 42 3 2 3

a a b a a

b b

(1)

3 3 2 6 2

3

2 2

3 33

16 64 42 3 2 3

b b c c c

c c

(2)

3 3 2 6 2

3

2 2

3 33

16 64 42 3 2 3

c c a c c

a a

(3)

Lấy (1)+(2)+(3) ta được:

2 2 2

2 2 29 3

16 4

a b cP a b c

(4)

Vì a2+b

2+c

2=3

Từ (4)3

2P vậy giá trị nhỏ nhất

3

2P khi a=b=c=1.

Câu 86.

Cho ba số thực dương a, b, c thỏa mãn 1a b c . Tìm giá trị nhỏ nhất của biểu

thức 1 1 1

Pa bc b ac c ab

Lời giải.

64

Câu 87.

Cho , ,x y z là các số thực dương thỏa mãn 3xy yz zx . Tìm giá trị nhỏ nhất

của biểu thức 2 2 2

2 2 2

3 3 31

8 8 8

x y zS x y z

y z x

Lời giải.

Xét hàm số 2( ) 3f t t t có 2

'( ) 1 0, ( )3

tf t t f t

t

đồng biến trên

. Vậy 2

11 0

2 1 2 1 3 132 ( 1)

2

xx

f x f x x xx x x

3 13 5 13

2 2x y

Với 2 2y x thay vào PT (2) ta được

+ Chứng minh: 1 1 1 1 1 1 9

9x y zx y z x y z x y z

1 1 1 9P

a bc b ca c ab a bc b ca c ab

+ Ta có: a + bc = a(a + b + c) + bc = (a + b)(a + c).

Suy ra ( ) ( ) 2

( )( )2 2

a b a c a b ca bc a b a c

+ Tương tự: ( ) ( ) 2

( )( )2 2

b a b c a b cb ca b a b c

( ) ( ) 2

( )( )2 2

c a c b a b cc ab c a c b

Suy ra: 2 2 2

2 2 2

a b c a b c a b ca bc b ca c ab

4( )2

2

a b ca bc b ca c ab

Vậy 9

2P

+ Vậy P đạt GTNN bằng 9

2

1

1 1

3

a b c

a b a c a b ca b c

b a b c a b c

c a c b

65

2 2 2 2 2 22 6 2 4 2 1 6 2 4 1x x x x x x x x x x

2 2

1 2 2( 1)( 1)

2 1 6 2 4

x xx x

x x x x

2 2

1 0 1 3

1 2 7 811

2 1 4 166 2 4

x x y

x x yx x x x

Vậy hệ có 3 nghiệm là 3 13 5 13 7 81

; , 1;3 , ;2 2 4 16

Tìm min của biểu thức 2 2 2

2 2 2

3 3 31

8 8 8

x y zS x y z

y z x

Ta có 2( ) 3( ) 9 3x y z xy yz zx x y z

Mặt khác 2 2 2 2 2 2 2 1( 1) 4( 1) 1 ( 1) 2

2x y z x y z x y z x y z

Đẳng thức xảy ra 1x y z

2 23 2 (y 2) (y 2 y 4) 6

0 8 (y 2)(y 2 y 4)2 2

y yy

2 2

23

2

68

x x

y yy

. Tương tự cộng lại ta được

2 2 2 2 2 2

2 2 23 3 32

6 6 68 8 8

x y z x y z

y y z z x xy z x

Đẳng thức xảy ra 1x y z

Ta lại có 2 2 2 2

2 2 2 2 2 2

( )

6 6 6 6 6 6

x y z x y z

y y z z x x y y z z x x

2

2

( )

( ) ( ) 12

x y z

x y z x y z

Đặt , 3t x y z t và xét hàm số 2

2( ) , 3

12

tf t t

t t

Ta có 2

2 2

24'( ) , '( ) 0 0, 24

( 12)

t tf t f t t t

t t

t 3 24

f t + 0

f t 48

47

66

1

2

1

3;

1min ( ) 3, 3 1

2f t S S x y z

. Vậy minS 3

Câu 88.

Cho a, b, c là các số thực dương. Chứng minh

4 4 4 4 4 4 9

ab bc ca a b c

b c a c a b a b c

.

Lời giải.

Ta có: 1 1 1 9

2 2 2 4 4b c b c c a b c a

1 1 2 1.

4 4 9 2 2b c a b c c a

1 2.

4 4 9 2 2

ab ab ab

b c a b c c a

Tương tự: 1 2

.4 4 9 2 2

bc bc bc

a c b c a a b

;

1 2.

4 4 9 2 2

ca ca ca

a b c a b b c

Vậy VT 1 2 2 2

9 2 2 2 2 2 2

ab ab bc bc ac ac

b c c a c a a b a b b c

9

a b c

Dấu “ = ” xảy ra khi và chỉ khi a = b = c

KĨ THUẬT HỆ SỐ KHÔNG XÁC ĐỊNH (UCT)

TRONG CHỨNG MINH BẤT ĐẲNG THỨC

NGUYỄN THÚC HOÀNG (Học sinh chuyên Toán – Tin)

VÕ QUỐC BÁ CẨN (GV. Toán)

Có bao nhiêu điều bí ẩn mà bạn chưa biết đến ? Câu trả lời là rất rất nhiều và

đôi khi bạn cảm thấy bực bội, khó chịu khi không thể tìm ra một lời giải thích

thỏa đáng cho bí ẩn nào đó. Nhưng bạn hãy quan niệm rằng đằng sau bất kì một

điều gì luôn hàm chứa một ý nghĩa nhất định. Và cũng không phải ngẫu nhiên

67

mà sự lí giải lại được hình thành. Trong thế giới bất đẳng thức cũng vậy. Đôi khi

bạn không thể hiểu được tại sao người ta lại có thể tìm ra một lời giải trông có

vẻ “kì cục” như thế Phải chăng là lần mò và may rủi lắm mới tìm ra được ?

Câu trả lời lại một lần nữa được nhắc lại: mỗi lời giải đều có sự giải thích của

riêng bản thân nó. Việc tìm ra lời giải đó phải đi qua một quá trình lập luận, thử,

sai và đúng. Trong chuyên đề nho nhỏ này chúng tôi muốn giới thiệu đến các

bạn một kĩ thuật cơ bản nhưng không kém phần hiệu quả trong việc chứng minh

một số dạng của bất đẳng thức. Nó không giúp ta giải quyết tất cả các bài toán

mà chỉ giúp ta tìm ra những lời giải ngắn gọn và ấn tượng trong một lớp bài toán

nào đó. Một số bài toán tuy dễ đối với phương pháp này nhưng lại là khó đối với

kỹ thuật kia. Đây cũng là điều hiển nhiên và dễ hiểu.

1. Bài toán mở đầu

Bài toán.

Cho , ,a b c là các số thực dương thỏa mãn 3 cba . Chứng minh rằng

2 2 2

2 2 2

1 1 1 2( )5

3

a b c

a b c

Chứng minh. Ta sử dụng bất đẳng thức sau đây

2

2

1 2 7 2

3 3 3

a a

a

Thật vậy bất đẳng thức trên tương đương với

03

)362()1(2

22

a

aaa

Hiển nhiên đúng với a là số thực dương.

Sử dụng các bất đẳng thức tương tự với b và c. Ta có điều phải chứng minh.

Đẳng thức xảy ra khi 1a b c .

Chắc chắn ngay khi đọc lời giải cho bài toán “ đơn giản” này bạn có phần lúng

túng và không hiểu tại sao lại có thể tìm ra bất đẳng thức phụ một cách “khó

hiểu” như vậy. Phải chăng là dự đoán một cách “vô hướng”. Hoặc cũng có

người sẽ nghĩ bài toán trên được tạo ra từ chính bất đẳng thức phụ đó. Câu trả

lời là hoàn toàn không phải. Tất cả đều đi theo 1 qui luật của nó. Ở các phần tiếp

theo chúng tôi sẽ phân tích về một kỹ thuật phân tích giúp tìm ra các bất đẳng

68

thức phụ và mở rộng vấn đề này theo chiều hướng khá mới mẻ. Kỹ thuật này có

tên là U.C.T, là viết tắt của 3 chữ cái đầu của cụm từ tiếng Anh Undefined

Coefficient Technique. Hay còn gọi là Kỹ Thuật Hệ số bất định. Đây là một kỹ

thuật cơ bản và là nền tảng quan trọng trên con đường tìm kiếm lời giải cho

những bất đẳng thức khó.

2. Khởi đầu ng m t ố ài toán ơ ản

Chúng ta sẽ khởi đầu kỹ thuật này bằng việc đưa ra cách giải thích cho việc tìm

ra bất đẳng thức phụ trên và nó cũng chính là cách giải thích cho các bài toán

sau này của chúng ta.

Bài toán trên các biến trong cả 2 vế và điều kiện đều không ràng buộc nhau điều

này khiến ta nghĩ ngay sẽ tách theo từng biến để chứng minh được đơn giản hơn

nếu có thể. Nhưng rõ ràng ta chỉ từng đó thôi là không đủ. Nếu ta chứng minh

bất đẳng thức sau

03

)32)(1)(1(

3

5

3

212

22

2

a

aaaa

a

Rõ ràng không hoàn toàn đúng với a thực dương.

Đừng bỏ cuộc tại đây bởi vì ở cách trên ta chưa sử dụng điều kiện 3a b c .

Như vậy ta sẽ không đi theo đường lối suy nghĩ đơn giản ban đầu nữa mà sẽ đi

tìm hệ số để bất đẳng thức sau là đúng

nmaa

a

3

5

3

21 2

2 (1)

Trong đó m và n là các hệ số chưa xác định.

Tương tự với biến b và c. Cộng vế theo vế ta có

)(33

53)(

3

5

3

222111 222

222nmncbam

cba

cba

Như vậy ở đây 2 hệ số m và n phải thỏa mãn điều kiện mnnm 0 . Thế

vào (1) dẫn đến

)1(3

5

3

21 2

2 am

a

a (2)

Đến đây ta chỉ cần xác định hệ số duy nhất là m để bất đẳng thức (2) là đúng.

69

Chú ý ở bài toán này điểm cực trị đạt được tại 1a b c nên ta cần xác định m

sao cho

03

)32)(1()1()1(

3

5

3

212

22

2

m

a

aaaam

a

a

Khi cho 1a thì ta có 3

2

3

)32)(1(2

2

a

aa từ đó ta dự đoán rằng

3

2m để tạo

thành đại lượng bình phương 2( 1)a trong biểu thức. Từ đó ta sẽ chứng minh bất

đẳng thức phụ

2

2

1 2 7 2

3 3 3

a a

a

Bài toán 1.

Cho , , ,a b c d là các số thực dương thỏa mãn 4a b c d . Chứng minh rằng

21

1

1

1

1

1

1

12222

dcba

Chứng minh. Ta sẽ xác định hệ số m để bất đẳng thức sau là đúng

01

1)1()1(

1

)1)(1()1(1

1

2222

m

a

aaam

a

aaam

a

Khi 1a ta sẽ có 111

12

m

a

a. Ta dự đoán bất đẳng thức sau đúng và

thật vậy

01

)1(2

1

22

2

2

a

aaa

a

Tương tự với các biến còn lại. Cộng vế theo vế ta có điều phải chứng minh.

Đẳng thức xảy ra khi và chỉ khi 1a b c d .

Bài toán 2.

Cho , ,a b c là các số thực dương thỏa mãn 3 cba . Chứng minh rằng

1111

222

bacacbcba

Chứng minh. Ở đây ta cần tìm m để bất đẳng thức dưới là đúng

)1()3(3

)1()1(

3

1

3

11222

am

aa

aaam

aacba

70

Tương tự như trên ta tìm dự đoán rằng với 9

1m thì bất đẳng thức phụ đúng.

Thật vậy

)3(3

)()1(0

)3(3

)3()1(0

99

4

3

12

2

2

2

2

aa

cba

aa

aaa

aa

Bài toán 3.

Cho , , ,a b c d là các số thực không âm thỏa 42222 dcba . Chứng minh rằng

dcbdbcadacabdcba 22

32)(2 3333

Chứng minh. Theo bài ra , , ,a b c d là các số thực dương thỏa mãn

2 2 2 2

2

4

( ) 2(2 )

( ) 2(2 )

a b c d

a b c d ab ac ad bc bd cd

a b c d ab ac ad bc bd cd

Bất đẳng thức cần chứng minh tương đương với

)(2

32)(2 3333 dcbadcba

Ta cần xác định hệ số m để bất đẳng thức sau đúng

)1(2

)1()12()1(

2

132

23

am

aaam

aa

Dễ dàng dự đoán 2

9m . Ta sẽ chứng minh điều đó, thật vậy

0)2()1(22

)1(9

2

132 23

aa

aaa

Điều này hiển nhiên đúng. Đẳng thức xảy ra khi và chỉ khi 1.a b c d

Bài toán 4.

Cho , ,a b c là các số thực dương thỏa mãn 3333 cba . Chứng minh rằng

27)(5111

4 222

cba

cba

Chứng minh.

Ta cần tìm hệ số m sao cho

)1)(1()455)(1(

)1(954 2

232

aaam

a

aaaama

a

Ta dễ dàng nhận ra đẳng thức xảy ra khi và chỉ khi 1.a b c

71

Khi cho 1a thì ta có thể dự đoán rằng 2m . Ta sẽ chứng minh rằng với 2m

thì bất đẳng thức phụ trên là đúng. Thật vậy

0)42()1(

2754 22

32

a

aaaaa

a

Do 0423 23 aaa . Vậy bất đẳng thức phụ trên là đúng.

Đẳng thức xảy ra khi và chỉ khi 1.a b c

Phần 3. K thuật huẩn h và U.C.T

Bây giờ chúng ta sẽ bước sang một khoảng không gian mới với lớp bất đẳng

thức thuần nhất đối xứng ba biến và kĩ thuật chuẩn hóa kết hợp với U.C.T.

Đa thức ( , , )f a b c đối xứng định nghĩa dưới dạng: / / / /( , , ) ( , , )f a b c f a b c trong đó

/ / /( , , )a b c là một hoán vị tùy ý của ( , , )a b c . Hay nói cách khác là

),,(),,(),,( bacfacbfcbaf

Tính thuần nhất của một đa thức đối xứng ba biến trên miền D có nghĩa là

),,(),,( cbafkkckbkaf n với mọi , , , , constk a b c D n chỉ phụ thuộc vào hàm

( , , )f a b c . Hiểu một cách đơn giản đa thức thuần nhất nếu nó là tổng của các đơn

thức đồng bậc. Do một số tính chất của hàm thuần nhất ta có thể chuẩn hóa điều

kiện của biến để đơn giản hóa việc chứng minh. Ta có thể chuẩn hóa một đa

thức thuần nhất đối xứng ba biến bằng cách đặt

,...,, rcabcabpabckcba nnn Đây là kỹ thuật rất quan trọng giúp ta

đơn giản hóa và qui bất đẳng thức về chứng minh theo từng biến. Hãy cùng đến

với một số bất đẳng thức thuần nhất đối xứng ba biến để thấy công dụng của

U.C.T

Bài toán 6.

Cho , ,a b c là các số thực không âm. Chứng minh rằng

3

2

a b c

b c c a a b

Chứng minh. Không mất tính tổng quát chuẩn hóa 3a b c .

Bài toán qui về việc chứng minh

3

3 3 3 2

a b c

a b c

Ta cần chứng minh bất đẳng thức

72

1 3( 1)( 1) ( 1)

3 2 2(3 )

a am a m a

a a

Dễ dàng dự đoán 3

4m . Ta chứng minh bất đẳng thức với m như vậy thì luôn

đúng

23 1 3( 1)0

3 4 4(3 )

a a a

a a

Điều này hiển nhiên đúng.

Sử dụng tương tự với các biến còn lại. Cộng vế theo vế ta có điều phải chứng

minh. Đẳng thức xảy ra khi .a b c

Bài toán 7.

Cho , ,a b c là các số thực không âm. Chứng minh rằng

2

222

22

2

22

2

22

2

)(

)(3

)(2

)(

)(2

)(

)(2

)(

cba

cba

abc

cba

cab

bca

cba

acb

Chứng minh. Chuẩn hóa 3a b c . Khi đó bất đẳng thức cần chứng minh

tương đương với

222

2

2

2

2

2

2

32

)23(2

32

)23(2

32

)23(2cba

cc

c

bb

b

aa

a

Ta cần xác định hệ số m để bất đẳng thức sau là đúng

)1(32

)23(2 2

2

2

ama

aa

a

Ta lại có

32

)64)(3)(1(

32

)23(22

22

2

2

aa

aaaaa

aa

a

Từ đây dễ dàng dự đoán với 6m thì bất đẳng thức phụ trên là đúng. Thật vậy

2 22

2 2

2(3 2 ) ( 1) (6 )6( 1) 0

2 3 2 3

a a a aa a

a a a a

Điều này hiển nhiên đúng do (0,3).a

Tương tự với các biến còn lại. Đẳng thức xảy ra khi và chỉ khi .a b c

Bài toán 8 Cho , ,a b c là các số thực dương. Chứng minh rằng

5

6

)(

)(

)(

)(

)(

)(222222

cba

bac

bac

acb

acb

cba

Chứng minh. Không mất tính tổng quát, chuẩn hóa 3a b c . Ta có bất đẳng

thức cần chứng minh tương đương với

73

5

6

269

)3(

269

)3(

269

)3(222

cc

cc

bb

bb

aa

aa

Tương tự như trên ta dễ dàng tìm ra bất đẳng thức phụ sau:

2

2 2

(3 ) 21 9 ( 1) (18 9)0

9 6 2 25 25(9 6 2 )

a a a a a

a a a a

Điều này hiển nhiên đúng. Đẳng thức xảy ra khi và chỉ khi .a b c

Phần 1 . Bài tập áp ụng

Bài toán 1 Cho , , ,a b c d là các số thực dương thỏa mãn 4a b c d . Chứng

minh rằng

3 3 3 3 2 2 2 2 16( ) ( )

8a b c d a b c d

Bài toán 2. Cho , ,a b c là các số thực dương nhỏ thỏa mãn 1a b c . Chứng

minh rằng

3 3 3 2 2 26( ) 1 5( )a b c a b c

Bài toán 3. Cho , ,a b c là các số thực dương nhỏ thỏa mãn 1a b c . Chứng

minh rằng

2 2 2

1 1 1 27

1 1 1 10a b c

Bài toán 6 Cho , , (1,2)a b c . Chứng minh rằng

14 4 4

b a c b a c

b c c a c a a b a b b c

Bài toán 7. Cho , ,a b c là các số thực dương thỏa mãn 1abc . Chứng minh rằng

2 2 2

1 1 13

1 1 c 1a a b b c

TÌM CỰC TRỊ CỦA BIỂU THỨC NHIỀU BIẾN BẰNG CÁCH QUY VỀ

MỘT BIẾN

BÙI TUẤN ANH

(GV THPT Yên Thủy, Hòa Bình)

74

Để tìm giá trị nhỏ nhất (GTNN), giá trị lớn nhất (GTLN) của một biểu thức chứa

nhiều hơn một biến số ta có thể biểu diễn các biến số của biểu thức đó theo một

biến số mới, khi đó việc tìm GTNN, GTLN của biểu thức theo biến số mới này

(chú ý đến điều kiện của nó) cũng chính là việc tìm GTNN, GTLN của biểu thức

ban đầu. Dưới đây là một số thí dụ minh họa cho phương pháp tìm GTNN,

GTLN bằng cách quy về một biến nói trên.

Thí dụ 1. Cho , , ,a b c d là các số thực dương. Tìm GTNN của biểu thức:

4

4.

a b c d abcdA

a b c dabcd

Lời giải: Đặt 4

a b c dt

abcd

, áp dụng BĐT Cauchy cho hai số dương ta có:

42 2 4 .a b c d ab cd abcd

Suy ra 4.t Đẳng thức xảy ra khi và chỉ khi .a b c d

Khi đó bài toán quy về tìm GTNN của

1( ) , 4.A t t t

t

Xét 1 2 4.t t Ta có

2 1 2 12 1

2 1

( )( 1)( ) ( ) 0.

t t t tA t A t

t t

Vậy khi 4t thì hàm số ( )A t đồng biến. Do đó GTNN của ( )A t bằng 17

4 đạt

được tại 4t khi và chỉ khi .a b c d

Thí dụ 2. Tim GTNN và GTLN của biểu thức 2 2 2 22 3 ,P x y x y biết ,x y thỏa

mãn

2 2 2 2 2 2 2( 1) 4 5 3 1 0 (1)P x y x y x y

Lời giải. Ta có

2 2 2 2 2 2 2 2(1) ( ) 3( ) 2 3 (2)x y x y x x y

Dễ thấy 2 2 23 0x x y với mọi ,x y . Đẳng thức xảy ra khi và chỉ khi 0.x

Đặt 2 2t x y . Từ (2) ta có

2 3 2 0 1 2t t t và P có dạng

2( ) 2P t t t .

Bài toán quy về tìm GTNN và GTLN của

2( ) 2P t t t với 1 2.t

75

Xét 1 21 2t t ta có

2 1 2 1 2 1( ) ( ) ( )( 1) 0.P t P t t t t t

Suy ra với 1 2t thì hàm số 2( ) 2P t t t đồng biến. Do đó:

2( ) 2P t t t đạt GTNN bằng 2 tại 1t khi và chỉ khi

2 2

00 & 1;

1

xx y

x y

2( ) 2P t t t đạt GTLN bằng 4 tại 2t khi và chỉ khi

2 2

00 & 2.

2

xx y

x y

Thí dụ 3. Cho hai số thực ,a b thuộc đoạn [2007;2008]. Tìm GTNN của biểu thức

2 2

2( ).

a bB a b

ab

Lời giải. Nếu 2007 2008,a b suy ra 2007

1,2008

t trong đó .a

tb

Lúc đó B có

dạng

1( ) ( 1)( ).B t t t

t

Bài toán quy về tìm GTNN của

2 1 2007( ) 1 , 1.

2008B t t t t

t

Xét 1 2

20071

2008t t ta có

2 1 2 1 2 1

2 1

1( ) ( ) ( )( 1 ) 0.B t B t t t t t

t t

Vậy với 2007

12008

t thì hàm số ( )B t dồng biến, do đó GTNN của B bằng

2 22007 2008(2007 2008 )

3007 2008

, đạt được khi và chỉ khi

2007

2008

a

b

Nếu 2007 2008.b a Lập luận tương tự thì B đạt GTNN bằng 4 khi và chỉ

khi .a b

Thí dụ 4. Cho hai số ,x y khác 0 thay đổi thỏa mãn 2 2( ) .x y xy x y xy Tìm

GTLN của biểu thức 3 3

1 1.C

x y

76

Lời giải. Cách 1. Đặt ,u x y v xy (điều kiện 2 4u v ). Khi đó theo giả thiết

của bài toán ta có 2 3 ,uv u v dễ thấy 0, 3u v do đó 2

3

uv

u

và điều kiện trở

thành

22 1

0 13 3

u uu u

u u

hoặc 3 (*)u

Sử dụng giả thiết ta biểu diễn được C theo u :

2 2 2

3 3 2

( )( ) 6 9( ).

x y x y xy u uC C u

x y u

Từ đây và (*) để tìm GTLN của C ta chỉ cần xét trên khoảng 1.u Với 2 1 1u u

ta có

1 2 1 22 1 2 1 2 2

1 2

6( ) ( ) ( ) 0.

u u u uC u C u u u

u u

Suy ra khi 1u thì hàm số ( )C u nghịch biến, do đó GTLN của C bằng 16 đạt

được khi và chỉ khi 1u hay 1

.2

x y

Cách 2. Đặt 1 1

.tx y

Từ giả thiết có 2 2

1 1 1 1 1

x y x y xy (chia cả hai vế cho 2 2x y

). Suy ra 2C t và

2 2 2

1 1 1 1 1 3 1 1 1 1 1

4 4 4x y x y x y x y

(đẳng thức xảy ra khi x y )

2 24 0 4 16.t t t t

Từ đó suy ra C đạt GTLN bằng 16 khi và chỉ khi 1

.2

x y

Các bạn hãy sử dụng phương pháp trên để giải các bài toán sau đây.

Bài toán 1. Cho , ,x y z là ba số thực dương thay đổi thỏa mãn điều kiện

20.

11x y z Tìm GTNN của biểu thức

1

1 1 1.T x y z

x y z

Bài toán 2. Cho ,x y là hai số thực thay đổi thỏa mãn điều kiện 2 22( ) 1.x y xy

Tìm GTNN, GTLN của biểu thức

2 2 2 2

2 7( ) 4 .T x y x y

77

Bài toán 3. Cho ba số thực , ,x y z thuộc đoạn [ ; ]a b với 0 < .a b Tìm GTLN của

biểu thức

3

1 1 1( ) .T x y z

x y z

Chuyên đề: ĐẶT ẨN PHỤ trong ài toán: “Chứng minh bất đẳng thứ ”

----------------------------------------------

Thí dụ 1: Cho ba số thực , ,a b c thỏa mãn điều kiện 1a b c

Chứng minh rằng: 2 2 2 1a

3b c

Lời giải:

Đặt 1

3a x ;

1

3b y ;

1

3c z

Từ 1a b c suy ra 0x y z

Khi đó: 2 2 2

2 2 2 1 1 1 1 1a

3 3 3 3 3b c x y z

2 2 2 20

3x y z x y z

2 2 2 0x y z (luôn đúng). Do đó suy ra điều phải chứng minh.

Thí dụ 2: Cho bốn số thực , , ,a b c d thỏa mãn 0a b c d . Chứng minh các

bất đẳng thức sau:

a) 22 2 2a b c a b c (1)

b) 22 2 2 2a b c d a b c d (2)

Lời giải:

Do ;a b c d nên tồn tại 0 ; 0x y sao cho ;a b x c d y

Khi đó:

a) (1) 2 22 2b x b c x c

2 2 2 2 2 22 2b bx x b c x cx c

2 ( ) 0x b c (đúng vì 0 ;x b c )

Từ đó bất đẳng thức (1) được chứng minh.

b) (2) 2 2 22 2b x b d y d x y

78

2 2 2 2 2 2 2 22 2 2b bx x b d dy y d x xy y

bx dy xy

Mặt khác ta có: b c b d y bx d y x dx xy xy

Vậy bx dy xy

Từ đó bất đẳng thức (2) được chứng minh.

Thí dụ 3: Cho a, b, c là độ dài ba cạnh của một tam giác thỏa mãn 2a b c

Chứng minh rằng: 2 2 2 2 2a b c abc (1)

Lời giải:

Từ giả thiết suy ra 0 1 ; 0 1 ; 0 1a b c

Đặt 1 ; 1 ; 1a x b y c z với 0 1 ; 0 1 ; 0 1x y z

Do 2a b c nên 1x y z

Khi đó, bất đẳng thức (1) 2 2 2

1 1 1 2 1 1 1 2x y z x y z

2 2 23 2 2 1 2x y z x y z x y z xy yz xz xyz

2 2 23 2.1 2 1 1 2x y z xy yz xz xyz ( vì 1x y z )

2 2 2 2 2 2 2 1x y z xy yz xz xyz

2

2 1x y z xyz

1 2 1xyz 1 2 1 2 0xyz xyz (hiển nhiên)

Do đó bất đẳng thức (1) được chứng minh.

Thí dụ 4: Cho 0a b . Chứng minh rằng: 2 2 22a b ab b a (1)

Lời giải:

Do 0a b nên đặt ( 0)a b x x

Khi đó bất đẳng thức (1) 2 2 22b x b b x b b b x

2 22 2bx x b bx b x

Mặt khác do , ,a b x là các số dương nên:

2 22bx x x x và 2 22b bx b b

Cộng theo vế hai bất đẳng thức trên ta có: 2 22 2bx x b bx b x (đpcm)

79

Thí dụ 5: Cho các số thực a, b, c thuộc khoảng (0 ; 1) thỏa mãn:

1 1 1abc a b c (1)

Chứng minh rằng: 2 2 2 3

4a b c (2)

Lời giải:

Từ giả thiết ta suy ra:

-Nếu cả ba số a, b, c đều lớn hơn 1

2 thì

1

8abc còn

11 1 1

8a b c , không

thỏa mãn (1)

-Nếu cả ba số a, b, c đều bé hơn 1

2 thì

1

8abc còn

11 1 1

8a b c , không

thỏa mãn (1)

Vậy trong ba số a, b, c có hai khả năng sau:

Trường hợp 1: Hai số lớn hơn hoặc bằng 1

2, số còn lại bé hơn hoặc bằng

1

2

Giả sử: 1 1 1

; ;2 2 2

a b c

Đặt 1 1 1

; ;2 2 2

a x b y c z , trong đó 1 1 1

0 ; 0 ; 02 2 2

x y z

Khi đó: (1) 1 1 1 1 1 1

2 2 2 2 2 2x y z x y z

Dẫn đến 1

2 02

x y z xyz (do 0 ; 0 ; 0x y z ), suy ra 0x y z (3)

Mặt khác bất đẳng thức (2) 2 2 2

1 1 1 3

2 2 2 4x y z

2 2 2 0x y z x y z

(hiển nhiên đúng theo (3) và 2 2 2 0x y z )

Trường hợp 2: Một số lớn hoặc bằng 1

2, hai số còn lại bé hơn hoặc bằng

1

2

Giả sử: 1 1 1

; ;2 2 2

a b c

Đặt 1 1 1

; ;2 2 2

a x b y c z , trong đó 1 1 1

0 ; 0 ; 02 2 2

x y z

Khi đó biến đổi tương đương (1) ta được 4x y z xyz

80

Suy ra: 1

4 2 . 2 2 . . 22

x y z xyz y xz xz (do 1

02

y ) 2 2 2 2 2x z x y z

(4)

Lại có bất đẳng thức (2) 2 2 2

1 1 1 3

2 2 2 4x y z

2 2 2 0x y z x y z (hiển nhiên theo (4))

Vậy bất đẳng thức (2) được chứng minh.

Thí dụ 6: Cho các số thực a, b, c thỏa mãn:

2 2 2

4 ; 5 ; 6

90

a b c

a b c

Chứng minh rằng: 16a b c

Lời giải:

Đặt 4 , 5 , 6a x b y c z

Từ giả thiết suy ra: 0 , 0 , 0x y z . Ta giả sử ngược lại: 16a b c , dẫn đến

1x y z

Mặt khác từ 2 2 2 90a b c

2 2 2

4 5 6 90x y z

2 2 2 8 10 12 13x y z x y z (1)

Do 0 1x y z 2

1x y z

2 2 2 2 1x y z xy yz xz

2 2 2 1x y z (vì 0 ; 0 ; 0x y z )

Khi đó ta có: 2 2 2 8 10 12x y z x y z

2 2 2 12 4 2 1 12 13x y z x y z x y

Điều này mâu thuẫn với đẳng thức (1)

Từ đó ta có điều cần chứng minh.

*Để bạn đọc tiếp thu đƣợc bài viết, xin mời các bạn hãy giải bài tập ƣới

đây:

Bài 1: Cho n số 1 2, ,......, na a a thỏa mãn điều kiện 1 2 ...... 1na a a

Chứng minh rằng: 2 2 2

1 2

1...... na a a

n

81

Bài 2: Chứng minh rằng: Nếu 2 23 , 3 , 25a b a b thì 7a b

Bài 3: Cho 0;a c b c . Chứng minh rằng: c a c c b c ab

Bài 4: Cho ba số a, b, c thỏa mãn 0 2 , 0 2 , 0 2a b c và 3a b c

Chứng minh rằng:

a) 2 2 2 5a b c b) 3 3 3 9a b c

Bài 5: Cho P a b b c c a abc với , ,a b c là các số nguyên. Chứng minh

rằng nếu a b c chia hết cho 4 thì P chia hết cho 4

(Gợi ý: Đặt 4a b c k với k là số nguyên)

Bài 6: Cho bốn số , , ,a b c d thuộc khoảng (0 ; 1) thỏa mãn:

1 1 1 1a b c d abcd . Chứng minh rằng: 2 2 2 2 1a b c d

MỘT HƯỚNG CHỨNG MINH BẤT ĐẲNG THỨC CÓ ĐIỀU KIỆN

Hoàng Hải Dương (GV THCS Chu Mạnh Trinh, Vân Giang, Hưng Yên)

Trong các đề thi chọn học sinh giỏi và các đề thi vào lớp 10 THPT chuyên, ta

thường thấy xuất hiện các bài toán chứng minh bất đẳng thức có điều kiện. Đây

là một dạng toán khó bởi lẽ nó còn rất “mới mẻ” với học sinh THCS và thường

không có phương pháp chung để giải các bài toán dạng này.

Tuy nhiên trong nhiều bài toán chứng minh bất đẳng thức có điều kiện, chúng ta

có thể dựa vào điều kiện của biến để đặt ẩn phụ, đưa bài toán về dạng đơn giản

hơn có thể đánh giá được trực tiếp mà không cần sử dụng đến các kiến thức cao.

Dưới đây là một số thí dụ.

Thí dụ 1: Cho 2x y . Chứng minh rằng 5 5x 2y

Nhận xét: Dự đoán đẳng thức xảy ra khi x 1y , dẫn đến cách đặt

x 1 ; 1 ,a y a a giúp cho lời giải bài toán đơn giản hơn.

Lời giải: Đặt 1x a với a Từ giả thiết suy ra 1y a

Ta có: 5 55 5 2 41 1 2 20 10 2x y a a a a

Đẳng thức xảy ra khi và chỉ khi 0a hay x 1y

Thí dụ 2: Cho 3.x y z Chứng minh bất đẳng thức sau:

2 2 2 x 6x y z xy yz z

Lời giải: Dự đoán đẳng thức xảy ra khi 1x y z

Đặt 1 ; 1 ,x a y b a b . Từ giả thiết suy ra 1z a b

82

Ta có

2 2 2

2 2 2

2 2

x

(1 ) (1 b) (1 ) 1 1 1 1 1 1

36 6

2 4

x y z xy yz z

a a b a b b a b a b a

b ba

Đẳng thức xảy ra khi và chỉ khi

00

100

2

ba

x y zbba

Thí dụ 3: Cho a b c d Chứng minh bất đẳng thức: 2 2 d 3ab c c b

Lời giải: Dự đoán đẳng thức xảy ra khi a b c d

Đặt c a x với x . Từ giả thiết suy ra d b x . Ta có:

2 22 2

2 2

d

3x3a 3a

2 4

c d c a x b x a x b x

xa b b b

Đẳng thức xảy ra khi và chỉ khi

0

002

xa b

a b c dxxa b

Thí dụ 4: Cho 4x . Tìm giá trị nhỏ nhất của biểu thức 2A 2x x

Lời giải: Dự đoán giá trị nhỏ nhất đạt được khi x 4

Đặt 4x t , từ giả thiết suy ra 0t

Ta có 2 23 2A 4 2 4 10 32 32 5 7 32 32t t t t t t t t

Đẳng thức xảy ra khi và chỉ khi t =0 hay x = 4

Thí dụ 5: Cho x 3, 1y x . Chứng minh rằng 3 3 2 26 9 0y x y x y

Lời giải: Đặt x 1 ; 0a a . Từ giả thiết suy ra 2y a . Lúc này bất đẳng thức

cần chứng minh tương đương với:

3 3 2 2 23 22 1 6 2 1 9 2 0 2 0 1 0a a a a a a a a a a

(đúng vì 0a )

Đẳng thức xảy ra khi và chỉ khi 0a hoặc 1a tức là khi x=1,y=2 hoặc

x 0, 3y

Thí dụ 6: Cho 1; 3x x y . Tìm giá trị nhỏ nhất của biểu thức 2 23 3xB x y y

Lời giải: Đặt 1x a và 3x y b . Từ giả thiết suy ra , 0a b

Ta có: 2y a b . Từ đó

83

2 2

2 2

2

2 2 2

3x 3x

3 1 2 3 1 2

5 3 9 27 275 7 13

2 2 4 2 4 4

B y y

a a b a a b

ba b a b ab a b b

Đẳng thức xảy ra khi và chỉ khi

5

2

0

a

b

tức là 3 9

;2 2

x y

Vậy B đạt giá trị nhỏ nhất bẳng 27

2khi

3 9;

2 2x y

Thí dụ 7:Cho 2a b . Chứng minh bất đẳng thức sau: 3 3 4 4a b a b

Lời giải: Dự đoán bất đẳng thức xảy ra khi 1a b . Đặt 1 ; 1a x b y Từ giả

thiết suy ra 0x y . Ta có bất đẳng thức cần chứng minh tương đương với:

3 3 4 4

3 3

2 2 2 2 4 4

1 1 1 1

0 1 1

3 3 0

x y x y

x x y y

x y x y x xy y x y x y

(đúng vì x 0y )

Đẳng thức xảy ra khi và chỉ khi x 0y hay 1a b

Thí dụ 8: Cho 1ab . Chứng minh rằng: 2 2a b a b

Lời giải: Dự đoán đẳng thức xảy ra khi 1a b . Đặt 1 ; 1a x b y Từ giả thiết

suy ra 0x y . Ta có :

1 1 1 1 0ab x y x y xy

Ta có: 2 22 2 2 21 1 2 0x y x y x y x y x y x y

Lại có 2 2 2xx y y với mọi x,y nên ta có:

2 2 2 210

2x y x y x y xy x y (đúng vì 0x y xy )

Đẳng thức xảy ra khi và chỉ khi 0x y hay 1a b

Bài tập

1. Cho 3a b c . Chứng minh rằng: 4 4 4 3 3 3a b c a b c

2. Cho , y 0x thỏa mãn 1x y . Chứng minh rằng: 2 2

2 314

xy x y

3. Cho 1a b c d . Chứng minh rằng: 1

2 22

a c b d ac bd

4. Cho 8a b và 3b . Chứng minh rằng: 2 327 10 945a b

84

SỬ DỤNG BẤT ĐẲNG THỨC ĐỂ CHỨNG MINH BẤT ĐẲNG THỨC

( dành cho THCS)

Hiện nay có rất nhiều phương pháp chứng minh BĐT nhưng để có 1 lời giải đẹp

và đơn giản thì không dễ gì thực hiện được. Trong bài viết này, chúng tôi giwois

thiệu với bạn đọc một kĩ thuật chứng minh BĐT với những lời giảng thực sự đẹ

và ấn tượng : chứng minh BĐt nhờ sử dụng các đẳng thức. Dưới đây là ví dụ:

Ví dụ 1 : cho số thực , ( 0).x y x y

Chứng minh rằng

2

2 2 12

xyx y

x y

.

Lời giải :

Đặt 1 xy

zx y

ta có : 1xy yz zx và BĐT đã trở thành:

2 2 2 2 2 2 22 2( ) ( ) 0x y z x y z xy yz zx x y z ( luôn đúng)

Vậy BĐT được chứng minh.

Ví dụ 2 : cho các số thực phân biệt a,b,c. chứng minh rằng:

2 2 2 2 2 2

2 2 2

1 1 1 3

( ) ( ) ( ) 2

a b b c c a

a b b c a b

Lời giải:

Ta có hệ thức:

1 1 1 1 1 1. . . 1

ab bc bc ca ca ab

a b b c b c c a c a a b

Mặt khác, ta có

2 2 2( ) ( ) ( ) 0x y y z z x

2 2 2x y z xy yz zx

Dẫn đến : 2 2 2

1 1 1 1 1 1 1 1 1. . . 1

ab bc ca ab bc bc ca ca ab

a b b c c a a b b c b c c a c a a b

.

Lại có : 1 1 1 1 1 1

. . . 1ab bc bc ca ca ab

a b b c b c c a c a a b

Từ 2 2 2 2( )x y z xy yz zx dẫn đến

2 2 21 1 1 1 1 1 1 1 1

2 . . . 2ab bc ca ab bc bc ca ca ab

a b b c c a a b b c b c c a c a a b

Vậy

85

2 2 2 2 2 2

2 2 2

2(1 ) 2(1 ) 2(1 )

( ) ( ) ( )

a b b c c a

a b b c a b

2 2 2 2 2 2 2 2 2 2 2 2

2 2 2

(1 ) (1 ) (1 ) (1 ) (1 ) (1 )

( ) ( ) ( )

a b a b b c b c c a c a

a b b c c a

2 2 2 2 2 21 1 1 1 1 1

3ab bc ca ab bc ca

a b b c c a a b b c c a

.

Ví dụ 3: cho các số dương , ,a b c thỏa mãn 1abc . Chứng minh rằng:

2 2 2

4 1

( 1) ( 1) ( ) ( 1)( 1)( 1) 4

a b c

a b a b a b c

Lời giải: đặt 1 1 1

, ,1 1 1

x y za b c

x y z

Suy ra 1 1 1

1 , , 1, , ,1 1 1

x y zx y z a b c

x y z

Dễ thấy : (1 )(1 )(1 ) (1 )(1 )(1 )x y z x y z , dẫn đến 0x y z xyz .

Mặt khác : 2

2

4 21 , 1 ,

( 1) 1

ax x

a a

Nên BĐT tương đương với

2 2 21 1 1 1 2(1 )(1 )(1 )x y z x y z

2 2 2 2( ) 2( ) 0x y z xy yz zx x y z xyz

2( ) 0x y z ( luôn đúng).

Ví dụ 4: cho các số dương a,b,c thỏa mãn 1abc . Chứng minh rằng:

2 2 2

1 1 1 21

(1 ) (1 ) (1 ) (1 )(1 )(1 )a b c a b c

Lời giải: đặt tương tự ví dụ , ta có BĐT cần chứng minh tương đương với:

2 2 2(1 ) (1 ) (1 ) (1 )(1 )(1 ) 4x y z x y z

2 2 2 2 2 2 4x y z x y z xyz

Sử dụng BĐT Cauchy cho 2 số dương, ta có

2 2 2 2 2 2 2 2 2 2 2 2 4 4 442 2 4 4 4x y z x y z x y z x z y x y z xyz xyz

Ví dụ 5: Cho các số thực phân biệt a,b,c. Chứng minh rằng :

2 2 2 2 2 2

2 2 2

9

( ) ( ) ( ) 4

a ab b b bc c c ca a

a b b c c a

Lời giải: ta có

86

2 2 2 23 1( ) ( )

4 4a ab b a b a b

Nên

2 2 22 2 2 2 2 2

2 2 2

31

( ) ( ) ( ) 4

a ab b b bc c c ca a a b b c c a

a b b c c a a b b c c a

Mặt khác có( xem tuyển chọn theo chuyên đề THTT, quyển 1, trang 12)

. . . 1a b b c b c c a c a a b

a b b c b c c a c a a b

Do đó :

2 2 2

2 . . . 2a b b c c a a b b c b c c a c a a b

a b b c c a a b b c b c c a c a a b

Dẫn đến :

2 2 2 2 2 2

2 2 2

3 9(2 1)

( ) ( ) ( ) 4 4

a ab b b bc c c ca a

a b b c c a

. (đpcm)

Ví dụ 6: Cho các số thực phân biệt a,b,c. Chứng minh rằng:

2 2 2

2 2 2

1 1 1 9( )

( ) ( ) ( ) 2a b c

a b b c c a

Lời giải: theo ví dụ 5 ta có:

2 2 2

2a b b c c a

a b b c c a

2 2 2

2 2 2a b b c c a

a b b c c a

2 2 2 2 2 2

2 2 2

( ) ( ) ( ) ( ) ( ) ( )

( ) ( ) ( )

a b a b b c b c c a c a

a b b c c a

2 2 2

3 5a b b c c a

a b b c c a

Mặt khác, ta thấy:

1 1 1 1 1 1a b c a b c

b c c a a b b c c a a b

. . . 1a b b c c a

b c c a c a a b a b b c

2 2 2

2 . . . 2a b c a b b c c a

b c c a a b b c c a c a a b a b b c

Do đó

2 2 2

2 2 2

1 1 1( )

( ) ( ) ( )a b c

a b b c c a

87

2 2 22 2 2 2 2 2

2 2 2

5 92

( ) ( ) ( ) 2 2

a b b c c a a b c

a b b c c a b c c a a b

(dpcm).

Ví dụ 7: cho các số không âm a,b,c và không có hai số ào trong số chúng đồng

thời bằng 0. Chứng minh rằng :

2 2 210

2( )( )( )

a b c abc

b c c a a b a b b c c a

.

Lời giải:

Đặt

2 2 2, y , z

a b cx

b c c a a b

, ta có:

2( ) 12

2 2( )

a b c b cx

b c x a b c

.

Làm tương tự với y,z rồi cộng lại ta được

1 1 11 4

2 2 2xy yz zx xyz

x y z

.

Ta phải chứng minh

2 2 2 5 8.x y z xyz

Đặt , ,p x y z q xy yz zx r xyz

Ta có 4q r

và BĐT trở thành

2 22 5 8 7 12 0p q r p q

- Nếu 4p

và BĐT Schur ( xem THTT số 348, tháng 6 năm 2006 ) dạng

( )( ) ( )( ) ( )( ) 0x x y x z y y z y x z z x z y

Ta có :

2 2 3(4 ) (4 ) 64

9 9 4 9

p q p p q p pr q q

p

32 2 7( 36)

7 12 124 9

pp q p

p

Do đó, ta chỉ cần chứng minh

32 27( 36)

12 0 ( 3)( 16) 04 9

pp p p

p

Điều này đúng vì 4 3.p p q

- Nếu 4p

, ta có

2 16 4p q

nên

22 22 5 2 8

2

pp q r p q

88

Bất đẳng thức được chứng minh.

Qua các ví dụ trên,hẳn các bạn đã thấy được nhiều điều thú vị từ kĩ thuật khá

đơn giản nhưng hiệu quả này. Hi cọng nó sẽ phát triển hơn nữa. Cuối cùng,

chúng tôi xin nêu ra 1 số bài tập tự luyện.

Bài 1: cho các số thực phân biệt a, b, c. Chứng minh rằng:

2 2 2 2 2 2

2 2 2

( 2 ) ( 2 ) (b 2 ) (b 2 ) (c 2 ) (c 2 )22

a b a c c a a b

b c c a a b

Bài 2: cho các số thực phân biệt a, b, c. Chứng minh rằng

2 2 2 2 2 2

2 2 2

(1 )(1 ) (1 )(1 ) (1 )(1 )1

a b b c c a

a b b c c a

Bài 3: cho các số dương a, b, c thỏa mãn abc=1. Chứng minh rằng:

2 2 2

3 3 33

1 1 1

a b c

a b c

Bài 4 : cho các số không âm a, b, c không có hai số nào đồng thời bằng 0.

Chứng minh rằng

42

( )( )( )

a b c abc

b c c a a b a b b c c a

.

(Thầy Trần Tuấn Anh – Khoa Toán – Tin, ĐHKHTN – ĐHQG tp HCM)

Trước hết ta nhắc lại các dạng bất đẳng thức ( BĐT) Cauchy hai số thường gặp:

Dạng 1: ab 2 2

2

a b (1)

Đẳng thức xảy ra khi và chỉ khi a b

Dạng 2:2

a bab

với 0, 0a b (2)

Đẳng thức xảy ra khi và chỉ khi a b

Bây giờ ứng dụng BĐT Cauchy hai số để giải các bài toán sau đây.

Bài toán 1: Cho a, b, c là các số thực dương sao cho ,a c b c . Chứng

minh rằng: ( ) ( )c a c c b c ab

89

Lời giải:

BĐT cần chứng minh tương đương với . . 1c a c c b c

b a a b

Áp dụng BĐT (2) ta có:

1 1. (1 )

2 2

c a c c a c c c

b a b a b a

,

1 1. (1 )

2 2

c b c c b c c c

a b a b a b

.

Cộng theo vesehai BĐT trên ta có điều cần chứng minh. Đẳng thức xảy ra

khi và chỉ khi c a c

b a

c b c

a b

. Tức là

abc

a b

.

Bài toán 2: Cho a, b là các số thực dương. Chứng minh rằng

2 22 22( )

a ba b

b a .

Lời giải: BĐT cần chứng minh tương đương với 3 3 2 22( )a b ab a b .

Hay 2 2 2 2( )( ) 2 ( )a b a ab b ab ab a b .

Áp dụng các BĐT (1) và (2) ta có:

02

a bab

2 22 2 2 2 2 22 ( )

0 2 ( ) 2( )2

ab a bab a b a b a b ab

Nhân theo vế hai BĐT trên ta có BĐT cần chứng minh.

Đẳng thức xảy ra khi và chỉ khi a = b > 0.

Bài toán 3: Cho a, b là các số thực dương. Chứng minh rằng:

2 22 27( ) 8 2( )

a ba b a b

b a .

90

Lời giải: BĐT cần chứng minh tương đương với

2 2 2 2( )( 6 ) 8 2 ( )(3)a b a b ab ab ab a b

Áp dụng các BĐT (1) và (2) ta có:

2 2 22 2 2 ( ) ( )

0 2 ( )2 2

ab a b a bab a b

Từ đó suy ra bất đẳng thức (3) đúng nếu ta có

2 2 2 2( )( 6 ) 4 ( )a b a b ab ab a b

Hay 2 2 4 4 ( )a b ab ab a b . Áp dụng BĐT (2) ta có

2 24 2 ( ) .4 4 ( ).a b ab a b ab ab a b

Từ đó ta có BĐT cần chứng minh.

Đẳng thức xảy ra khi và chỉ khi: 2 2

2

0, 0

2

( ) 4

a b

ab a b

a b ab

Tức là a=b > 0.

Bài toán 4: Cho a, b, c là các số thực dương. Chứng minh rằng:

2 2 2

2 2 2 2 2 2 2

a b c a b c

a b b c c a

.

Lời giải: BĐT cần chứng minh tương đương với

3 3 3

2 2 2 2 2 2( ) ( ) ( )

2

a b c a b ca b c

a b b c c a

Hay 2 2 2

2 2 2 2 2 2 2

ab bc ca a b c

a b b c c a

.

Áp dụng BĐT (1) ta có:

2

2 2 2 2.2( )

ab abb

a b a b

2 2

2 2.2( )

a bb

a b

nên

2

2 2 2

ab b

a b

.

91

Tương tự 2

2 2 2

bc c

b c

2

2 2 2

ca a

c a

.

Công theo vế các bất đẳng thức trên ta có bất dẳng thức cần chứng minh.

Đẳng thức xảy ra khi và chỉ khi a = b = c > 0.

Bài toán 5: Cho a, b, c là các số thức dương sao cho abc 1. CHứng

minh rằng 5 2 5 2 5 2

5 2 2 5 2 2 5 2 20

a a b b c c

a b c b c a c a b

( Thi Olympic Toán quốc tế lần thứ 46 – năm 2005)

Lời giải: BĐT cần chứng minh tương đương với

5 2 5 2 5 2

5 2 2 5 2 2 5 2 21 1 1 3

a a b b c c

a b c b c a c a b

Hay 5 2 2 5 2 2 5 2 2 2 2 2

1 1 1 3

a b c b c a c a b a b c

(4)

Từ abc 1 và áp dụng BĐT (1) ta có:

25 2 22 2

1 1

aa b cb c

abc

=

2 2

4 4 24 2 2

2 2 2 2

2 2

1 1

2 2

b c

a a a b cb c b cbc b c

.

Do 22 2 2 2 2

4 4 43

u v uv u v u v nên

2

4 2 2 2 2 222 ( )

3a b c a b c .

Suy ra 2 2

5 2 2 2 2 2 2

1 3( )

2( )

b c

a b c a b c

Tương tự 2 2

5 2 2 2 2 2 2

1 3( )

2( )

c a

b a c a b c

2 2

5 2 2 2 2 2 2

1 3( )

2( )

a b

c a b a b c

92

Cộng theo vế ba BĐT trên, ta được BĐT (4). Đẳng thức xảy ra khi và chỉ

khi a = b = c.

Cuối cùng, mời các bạn áp dụng BĐT Cauchy hai số để giải quyết các bài

toán sau đây.

Cho a, b, c là các số thực dương, chứng minh rằng:

1) 3

2

ab bc ca

c ab a bc b ca

,

2) 2 2 2( )( )( ) ( )( )( );ab c bc a ca b abc a b b c c a

1a b c a b b c

b c a b c a b

TÌM CỰC TRỊ CỦA MỘT BIỂU THỨC MÀ CÁC BIẾN

CÓ ĐIỀU KIỆN RẰNG BUỘC

THÁI NHẬT PHƯỢNG

(GV. THCS Cam Nghĩa, Cam Ranh, Khánh Hòa)

Trong bài viết này ta tìm giá trị nhỏ nhất (GTNN), giá trị lớn nhất (GTLN) của

một biểu thức mà các biến phải thỏa mãn một điều kiện nào đó, hoặc các biến là

nghiệm của phương trình (PT) hoặc bất phương trình (BPT) bằng cách đưa

phương trình hoặc bất phương trình nhiều ẩn đã cho về bất phương trình một ẩn

với ẩn số là biểu thức đang xét (hoặc sai khác một hằng số).

Ví dụ 1. Tìm GTNN và GTLN của xy biết x và y là nghiệm của phương trình:

x4 + y

4 – 3 = xy(1 – 2xy) (1)

Lời giải. PT (1) ⇔ xy + 3 = (x2 + y

2)

2 ≥ 4x

2y

2

Đặt t = xy thì BPT trên trở thành 4t2 – t – 3 ≤ 0 ⇔ (t – 1)(4t + 3) ≤ 0

≤ t ≤ 1.

Vậy GTNN của xy là

khi x

2 = y

2 và xy =

⇔ x = - y =

GTLN của xy là 1 khi x2 = y

2 và xy = 1 hay x = y = 1

Ví dụ 2. Các số dương x, y, z thỏa mãn: xyz ≥ x + y + z + 2.

Tìm GTNN của x + y + z.

Lời giải. (x + y + z)3 ≥ (3√xyz )

3 = 27xyz

⇒ (x + y + z)3 ≥ 27(x + y + z + 2). Đặt t = x + y + z ≥ 0 thì:

93

t3 – 27t – 54 ≥ 0 ⇔ (t – 6)(t + 3)

2 ≥ 0 ⇔ t ≥ 6.

Vậy GTNN của x + y + z là 6 khi x = y = z = 2.

Ví dụ 3. Cho các số thực x, y, z thỏa mãn:

x2 + 2y

2 + 2x

2z

2 + y

2z

2 + 3x

2y

2z

2 = 9 (2)

Tìm GTLN và GTNN của A = xyz

Lời giải. Ta có: (2) ⇔ (x2 + y

2z

2) + 2(y

2 + x

2z

2) + 3x

2y

2z

2 = 9

Sử dụng BĐT Cô – si ta có: 2|A| + 4|A| + 3A2 ≤ 9 ⇔ A

2 + 2|A| - 3 ≤ 0

⇔ (|A| - 1)(|A| + 3) ≤ 0 ⇔ |A| ≤ 1⇔ -1 ≤ A ≤ 1

Vậy GTNN của A là -1 khi 2 trong 3 số x, y, z đều bằng 1 (hoặc đều bằng -1), số

còn lại bằng -1 và GTLN của A là 1 khi 2 trong 3 số x, y, z đều bằng 1 (hoặc

đều bằng – 1).

Ví dụ 4. Cho x, y, z là các số thực thỏa mãn: x4 + y

4 + x

2 – 3 = 2y

2(1 – x

2)

Tìm GTNN và GTLN của x2 + y

2 +

.

Lời giải. Ta có: x4 + y

4 + x

2 – 3 = 2y

2(1 – x

2) ⇔ (x

2 + y

2)

2 + x

2 + y

2 – 3 = 3y

2

Đặt t = x2 + y

2 > 0 dẫn đến t

2 + t – 3 ≥ 0 ⇔ (t +

)(t -

) ≥ 0.

⇔ t ≥ √

. Vậy GTNN của x

2 + y

2 +

khi x = √

, y = 0.

Ta có: x4 + y

4 + x

2 – 3 = 2y

2(1 – x

2) ⇔ (x

2 + y

2)

2 – 2(x

2 + y

2) – 3 = -3x

2

Đặt t = x2 + y

2 > 0 dẫn đến t

2 – 2t – 3 ≤ 0 ⇔ (t + 1)(t – 3) ≤ 0 ⇔ t ≤ 3.

Vậy GTLN của x2 + y

2 +

là 3

khi x = 0, y = √3

Các bạn hãy biến đổi các điều kiện trong các bài tập tương tự dưới đây để tìm

GTNN và GTLN của các biểu thức.

Bài 1. Cho các số dương x, y, z thỏa mãn: 2xyz + xy + yz + zx ≤ 1.

Tìm GTLN của xyz. (ĐS: GTLN của xyz là

)

Bài 2. Cho các số dương x, y, z thỏa mãn:

(x + y + z) 3 + x

2 + y

2 + z

2 + 4 = 29xyz.

Tìm GTLN của xyz. (ĐS: GTLN của xyz là 8)

Bài 3 Tìm GTLN và GTNN của biểu thức S = x2 + y

2 biết x, y là nghiệm của

phương trình: 5x2 + 8xy + 5y

2 = 36.

(ĐS: GTNN của S là 4, GTLN của S là 36).

Bài 4. Cho x, y là các số thực thỏa mãn: (x2 + y

2)

3 + 4x

2 + y

2 + 6x + 1 = 0

Tìm GTLN của x2 + y

2. (ĐS: GTLN của x

2 + y

2 là 1).

94

Bài 5. Tìm các số nguyên không âm x, y, z, t để biểu thức x2 + y

2 + 2z

2 + t

2 đạt

GTNN biết rằng : x2 – y

2 + t

2 = 21 và x

2 + 3y

2 + 4z

2 = 101

(ĐS: GTNN của biểu thức là 61)

MỘT BÀI TOÁN CỰC TRỊ

CÓ NHIỀU CÁCH GIẢI

NGUYỄN DUY THÁI

(GV. THCS Nam Hồng, TX. Hồng Lĩnh, Hà Tĩnh)

Trong đề thi tuyển sinh vào lớp 10 năm 2015 – 2016 tỉnh Hà Tĩnh có bài toán

cực trị, mà không nhiều thí sinh giải được. Tôi xin nêu ra kĩ thuật phân tích để

tìm ra nhiều cách giải cho bài toán này.

Bài toán. Cho các số thực a, b, c thỏa mãn a2 + b

2 + c

2 = 1. Tìm giá trị nhỏ nhất

của biểu thức F = ab + bc + 2ca.

Phân tích. Từ a2 + b

2 + c

2 = 1 có -1 ≤ a, b, c ≤ 1.

* Xét b = 1⇒ b2 = 1⇒ a

2 + c

2 = 0 ⇒ a = c = 0 ⇒ F = 0.

* Xét b = 0 ⇒ a2 + c

2 = 1 và F = 2ac.

Ta có: 2ac ≥ -(a2 + c

2) ⇒ ac ≥

= -

⇒ F ≥ -1.

Dấu “=” xảy ra khi a = -c và a2 + c

2 = 1 ⇔ a = -c =

.

Từ các định hướng trên ta có các cách giải sau.

Cách 1. Vì a2 + b

2 + c

2 = 1 nên F = ab + bc + 2ca = a

2 + b

2 + c

2 + ab + bc + 2ca

– 1 = [(a2 + 2ac + c

2) + b(a + c) +

] + 3.

– 1.

= [(a + c)2 + b(a + c) +

] + 3.

– 1 = (a + c +

)

2 + 3

– 1 ≥ -1.

Dấu “=” xảy ra khi b = 0 và a = - c = √

.

Vậy MinF = -1.

Cách 2. Ta có: (a + b + c)2 ≥ 0 , (a + c)

2 ≥ 0 và b

2 ≥ 0.

Cộng theo vế ba BĐT trên ta có: (a + b + c)2 + (a + c)

2 + b

2 ≥ 0.

⇔ a2 + b

2 + c

2 + 2(ab + bc + ca) + a

2 + 2ac + c

2 + b

2 ≥ 0.

⇔ 2(a2 + b

2 + c

2) + 2(ab + bc + 2ca) ≥ 0.

⇔ ab + bc + 2ca ≥ -(a2 + b

2 + c

2) = -1.

Do đó: F ≥ -1.

Cách 3. Ta có: (a + b + c)2 ≥ 0 ⇒ ab + bc + ca ≥

( )

=

(1)

95

(a + c)2 ≥ 0 ⇒ ac ≥

=

(2)

Công theo vế (1) và (2) ta được: F = ab + bc + 2ac ≥ -1

Cách 4. Ta có 1 = a2 + b

2 + c

2 ⇔ 2 = 2(a

2 + b

2 + c

2)

⇒ 2F + 2 = 2(ab + bc + 2ca) + 2(a2 + b

2 + c

2)

= (a + b + c)2 + (a + c)

2 + b

2 ≥ 0 ⇒ F ≥ -1.

Cách 5. Xét F + 1 = ab + bc + 2ca + a2 + b

2 + c

2

= (a + c)2 + b(a + c) + b

2 – 1 ⇔ (a + c)

2 + b(a + c) + b

2 – F – 2 = 0. (6)

Ta có (6) là phương trình bậc 2 có ẩn t = a + c. Để phương trình (6) có nghiệm

thì = b2 – 4.(b

2 – F – 2) ≥ 0 ⇒ F ≥ - 1 +

b2 ≥ - 1.

Cách 6. Ta có: F = ab + bc + 2ac = a2 + b

2 + c

2 + ab + bc + 2ac – 1= [(a

2 + 2ac

+ c2) + b(a + c) + b

2 – 1 =

(a + b + c)

2 +

(a – b + c)

2 – 1 ≥ -1.

Trong các cách 2, 3, 4, 5 và 6 dấu bằng xảy ra chẳng hạn khi b = 0 và

a = -c = √

.

Cách bạn hãy tìm thêm các giải khác nhé!

Bài tập vận dụng

Bài 1. Cho các số thực a, b, c thỏa mãn. Tìm giá trị lớn nhất và nhỏ nhất của

biểu thức F = ab + bc + 2ca

Bài 2. Cho các số thực a, b, c thỏa mãn a2 + b

2 + c

2 = 2016. Tìm giá trị nhỏ nhất

của biểu thức:

a) P = ab + 2bc + ca

b) Q = 2ab + bc + ca

c) R = 2ab – bc – ac

MỘT KỸ THUẬT CHỨNG MINH

BẤT ĐẲNG THỨC CÓ ĐIỀU KIỆN

NGUYỄN ĐỨC TẤN (TP. Hồ Chí Minh)

Trong các kì thi học sinh giỏi lớp 8, lớp 9 và thi vào lớp 10 chuyên, chúng

ta thường gặp dạng bài toán chứng minh BĐT mà các ẩn có điều kiện rằng buộc.

Các bài toán Bất đẳng thức có dạng: “Cho C ≥ D chứng minh rằng A ≥ B”.

Có một kĩ thuật chứng minh là ta đi chứng minh (A – B) +(C – D)≥ 0, khi đó từ

điều kiện C ≥ D ta suy ra được A ≥ B, Đô khi việc chứng minh BĐT trung gian

trên khá dễ dàng. Sau đây là một số ví dụ:

96

Ví dụ 1. Cho a + b ≥ 1. Chứng minh rằng: a2 + b

2 ≥

(Đề vào lớp 10, THPT chuyên Lê Hồng Phong năm, TP.Hồ Chí Minh, 1998 -

1999)

Bài làm. Ta có: (a2 + b

2 -

) + (1 – a – b) ≥ 0 ⇔ (a

2 – a +

) + (b

2 – b +

) ≥ 0

⇔ (a -

)

2 + (b -

)

2 ≥ 0 (đúng). Mà a + b ≥ 1 ⇒ 1 – a – b ≤ 0

Suy ra: a2 + b

2 -

≥ ⇒ a2

+ b2 ≥

(Đpcm)

Ví dụ 2. Chứng minh rằng nếu: a + b ≥ 2. Thì a4 + b

4 ≥ a

3 + b

3

Bài làm. Ta có: (a4 + b

4 – a

3 – b

3) + (2 – a – b) = a

4 – a

3 – a + 1 + b

4 – b

3 – b + 1

= a3(a – 1) – (a – 1) + b

3(b – 1) – (b – 1) ≥ 0 = (a – 1)(a

3 – 1) + (b – 1)(b

3 – 1)

= (a – 1)2(a

2 + a + 1) + (b – 1)

2(b

2 + b + 1) = (a – 1)

2[(a +

)

2 +

] +

(b – 1)2[(b +

)

2 +

] ≥ 0. Mà a + b ≥ 2 ⇒ 2 – a – b ≤ 0

Do đó: a4 + b

4 – a

3 – b

3 ≥ 0 ⇒ a

4 + b

4 ≥ a

3 + b

3

Ví dụ 3. Cho x, y là các số thực dương thỏa mãn: x3 + y

4 ≤ x

2 + y

3. Chứng minh

rằng: x3 + y

3 ≤ x

2 + y

2 và x

2 + y

3 ≤ x + y

2

(Đề thi học sinh giỏi lớp 9 Quận 1, TP. Hồ Chí Minh, 2004 – 2005)

Lời giải. a) Ta có: (x2 + y

2 – x

3 – y

3 )+ (x

3 + y

4 – x

2 – y

3) ≥ 0

⇔ y4 – 2y3 + y2 ≥ ⇔ y2(y2 – 2y + 1) ≥ ⇔ y2(y – 1)2 ≥ (đúng)

Mà : x3 + y

4 ≤ x

2 + y

3 ⇒ x

3 + y

4 – x

2 – y

3 ≤ 0 ⇒ x2

+ y2 – x

3 – y

3 ≥ 0

Vậy: x3 + y

3 ≤ x

2 + y

2 (đpcm)

b) Ta có: (x + y2 – x

2 – y

3) + (x

3 + y

4 – x

2 – y

3) = x

3 - 2x

2 + x + y

4 – 2y

3 + y

2 =

x(x2 – 2x + 1) + y

2(y

2 – 2y + 1) = x(x – 1)

2 + y

2(y – 1)

2 ≥ 0 (vì x > 0)

Mà x3 + y

4 ≤ x

2 + y

3 ⇒ x

3 + y

4 – x

2 – y

3 ≤ 0

Do đó: x + y2 – x

2 – y

3 ≥ 0 ⇒ x

2 + y

3 ≤ x + y

2 (Đpcm)

Ví dụ 4. Chứng minh rằng nếu: a + b + c ≥ 3 thì a4 + b

4 + c

4 ≥ a

3 + b

3 + c

3

(Đề thi HSG toán Quận 3 TP. Hồ Chí Minh, THCS Lê Quý Đôn, năm học 205 –

2005)

Bài làm. Ta có: (a4 + b

4 + c

4 - a

3 - b

3 - c

3) + (3 – a – b – c) ≥ 0

⇔ (a4 – a

3 – a + 3) + (b

4 – b

3 – b + 3) + (c

4 – c

3 – c + 3) ≥ 0

⇔ (a – 1)2[(a +

)

2 +

] + (b– 1)

2[(b+

)

2 +

] + (c – 1)

2[(c +

)

2 +

] ≥ 0 (đúng)

Mà a + b + c ≥ 3 ⇒ 3 – a – b – c ≤ 0. Do đó: a4 + b

4 + c

4 - a

3 - b

3 - c

3 ≥ 0

Vậy a4 + b

4 + c

4 ≥ a

3 + b

3 + c

3 (Đpcm)

97

Ví dụ 5. Cho x, y là số thực dương thỏa mãn x3 + y

3 = x – y.

Chứng minh rằng: x2 + y

2 < 1. (Đề thi vào lớp 10 chuyên, TP Hồ Chí Minh,

2006 – 2007)

Bài làm. Ta có x2 + y

2 < 1 ⇔ (1 – x

2 – y

2) + (x

3 + y

3 – x + y) ≥ 0

⇔ (x3 – x

2 – x + 1) + (y

3 – y

2 + y) ≥ 0 ⇔ (x – 1)(x

2 – 1) + y(y

2 – y + 1) ≥ 0

⇔(x – 1)2(x + 1) + y[(y -

)

2 +

] > 0 (Vì x, y là các số dương)

Suy ra : x2 + y

2 < 1 (Đpcm)

Bài tập tự luyện.

Bài 1. Biết rằng: x2 + y

2 ≤ x + y. Chứng minh rằng: x + y ≤ 2

Bài 2. Biết ab ≥ 1. Chứng minh rằng: a2 + b

2 ≥ a + b

Bài 3. Biết rằng: x2 + y

2 ≤ x. Chứng minh rằng: y(x + 1) ≥ -1.

SUY NGHĨ VỀ

MỘT DÃY BẤT ĐẲNG THỨC

CAO MINH QUANG

(GV. THPT chuyên Nguyễn B nh Khiêm V nh Long)

Nhận xét. Với 2 số thực x, y dương ta có:

( )

( ) ≥

( )( )

≥ .

/ ≥

( )

(1)

Chứng minh. Từ (x+y)2 ≥ 2xy, suy ra

.

/ ≥

( )

.

Vì (x – y)2 ≥ 0 và (x + y)(x – y)

2 ≥ 0, suy ra

( )( )

=

( ), ( ) ( )-

≥ ( )( )

=

( )( )

=

( ), ( ) ( ) -

≥ .

/ .

Áp dụng bất đẳng thức AM-GM ta có:

x3 + y

3 =

( ) ( )

( ) ≤

,( ) ( ) ( )-

( )

= ( )

( )

Kết hợp các bất đẳng thức trên ta được dãy bất đẳng thức (1).

Các bất đẳng thức (1) xảy ra khi x= y.

98

Sau đây ta xét một số bài toán áp dụng bất đăng thức (1).

Bài toán 1. Cho a, b, c là các số thực dương chứng minh rằng:

.

/ (2)

Lời giải: Áp dụng bất đẳng thức (1) với số thực dương d ta có:

≥ .

/ ;

≥ .

/

Cộng theo vế các bất đẳng thức trên và áp dụng (1) ta có:

≥ .

/ + .

/ ≥ 2.0

.

+

/1

=

(a + b + c + d)

Với d =

, từ bất đẳng thức trên ta có:

(

)

.a + b + c +

/ = 2.

/

≥ .

/

Đẳng thức xảy ra khi a = b = c.

Bài toán 2: Cho a, b, c là các số thực dương chứng minh rằng:

+

( )( )( ) ≥ 2.

Lời giải: Áp dụng bất đẳng thức AM-GM và (2) ta có:

≥ .

/ = .

( ) ( ) ( )

/

.[3√(a + b)(b + c)(c + a)

]3 =

( )( )( )

+

( )( )( ) ≥

( )( )( )

+

( )( )( ) ≥ 2

Dấu “=” xẩy ra khi và chỉ khi a = b = c

Bài toán 3. Cho a, b, c > 0. Chứng minh rằng:

√ ( ) +

√ ( ) +

√ ( )

≤ 2

Lời giải: Theo bất đẳng thức (1) ta có: √4(b + c )

≥ b + c

Do đó: a + √4(b + c )

≤ a + b+ c ⇒

√ ( ) ≤

Tương tự ta có:

√ ( ) ≤

;

√ ( )

Cộng vế theo vế các bất đẳng thức trên, ta có điều phải chứng minh.

99

Đẳng thức xảy ra khi a = b = c.

Bài toán 4. Cho a, b, c > 0. Chứng minh rằng

+

+

.

Lời giải. Theo bất đẳng thức (1), ta có: a3 + b

3 ≥ ab(a + b)

Suy ra: a + b + abc ≥ ab(a + b + c). Do đó:

( )

Tương tự ta có:

( ) ;

( )

Cộng theo vế của các các bất đẳng thức trên ta có điều phải chứng minh.

Đẳng thức xảy ra khi a = b = c.

Bài toán 5. Cho x, y, z > 0 và xyz = 1. Chứng minh rằng

+

+

≥ 2.

Lời giải. Đặt a = x3, b = y

3, c = z

3 thì abc = 1. Bất đẳng thức trở thành

+

+

≥ 2

Theo bất đẳng thức (1), ta có:

.

Tương tự ta có:

;

Cộng theo vế các bất đẳng thức trên ta được:

+

+

+

+

=

(a+b+c)

.3√abc

= 2.

Suy ra đpcm.

Đẳng thức xảy ra khi a = b = c hay x = y = z = 1.

Bài toán 6. Cho a, b, c, d > 0 thỏa mãn:

+

+

+

= 4. Chứng minh rằng:

+ √

+ √

+ √

≤ 2.(a + b + c + d) – 4

Lời giải. Theo bất đẳng thức (1) ta có: √

.

Do đó: √

+ √

+ √

+ √

+

+

+

Ta sẽ chứng minh:

100

+

+

+

≤ 2.(a + b + c + d) – 4 (3)

Ta lại có: a + b -

=

=

. Do đó nếu đặt x =

; y =

; z =

;

t =

và chú ý rằng x + y + z + t = 4 bất đẳng thức (3) trở thành

+

+

+

≥ 2 (4) . Theo bất đẳng thức Bunhiacốpxki ta có: (x + y

+ z + z + t + t + x)(

+

+

+

) ≥ 4.

Do đó (4) đúng. Suy ra đpcm.

Đẳng thức xảy ra khi x = y = z = t = 1 hay a = b = c = d = 1.

Bài tập vận dụng

Bài 1. (Thái Nhật Phượng) Cho x, y, z > 0 thỏa mãn xyz = 1. Chứng minh rằng:

+

+

≥ 3.

Bài 2. (Nguyễn Bá Nam) Cho a, b, c > 0. Chứng minh rằng

(a3 + b

3 + c

3)(

+

+

) ≥

.(

+

+

).

Bài 3. Cho a, b, c > 0. Chứng minh rằng:

+

+

+

+

Bài 4. Cho a, b, c > 0. Chứng minh rằng:

√4a + 4b + √4b + 4c

+ √4a + 4a ≤

+

+

SỬ DỤNG BIỆT THỨC DELTA CỦA

PHƢƠNG TRÌNH BẬC HAI ĐỂ GIẢI TOÁN CỰC TRỊ

BÙI MẠNH TÙNG

(GV. THCS Trưng Vương, Q. Hoàn Kiếm, Hà Nội)

CAO VĂN DŨNG

(GV. THPT Tây Hồ, Q. Tây Hồ, Hà Nội)

Bài toán cực trị là một dạng toán thường xuất hiện trong kì thi học sinh giỏivà

thi vào THPT. Có rất nhiều phương pháp để giải các bài toán cực trị, trong bài

này tôi xin giới thiệu cách sử dụng điều kiện có nghiệm của phương trình bậc

hai để giải một số bài toán cực trị.

Xét hàm số y = f(x) = ax2 + bx + c (a ≠ 0), với biệt thức = b

2 – 4ac.

Ta có: y = f(x) = ax2 + bx + c = a[(x +

)

2 -

].

101

Suy ra af(x) = a2(x +

)

2 -

. (1)

Từ đẳng thức (1) ta thấy nếu < 0 thì af(x) > 0 và ≥ 0 khi và chỉ khi tồn tại x

để f(x) = 0.

Ví dụ 1. Tìm giá trị nhỏ nhất của y = x2 + 3x – 1.

Lời giải. Ta có x2 + 3x – 1 – y = 0. (1)

Để phương trình (1) có nghiệm thì: = 32 – 4(-1 – y) = 13 + 4y ≥ 0

⇔ y ≥

.

Dấu “=” xảy ra khi = 0 hay x =

.

Vậy Miny =

khi x =

.

Ví dụ 2. Tìm giá trị lớn nhất và nhỏ nhất của biểu thức: P =

.

Lời giải. Ta có x2 – x + 1 = (x -

)

2 +

> 0, do đó P luôn xác định với mọi x.

Ta có: P =

⇔ (P – 1)x

2 – Px + P -1 = 0

Với P = 1 thì x = 0.

Với P ≠ 1, ta có: = P2 – 4(P – 1)

2 = -3P

2 + 8P – 4.

≥ 0 ⇔ P ≥

(1) hoặc P ≤ 2 (2)

Dấu bằng ở (1) xảy ra khi x = -1.

Dấu bằng ở (2) xảy ra khi x = -1.

Vậy MinP =

khi x = - 1, MaxP = 2 khi x = 1.

Ví dụ 3. Tìm giá trị lớn nhất và giá trị nhỏ nhất của biểu thức P =

với x, y

là các số thực thỏa mãn: x2y

2 + 2y + 1 = 0.

(Đề vào lớp 10 THPT chuyên KHTN Hà Nội năm 2015)

Lời giải. Ta có: x2y

2 + 2y + 1 = 0 ⇔ y =

.

P =

( ) =

⇔ 3Px

2y

2 + 2xy + P = 0. (1)

Trường hợp 1. P = 0 thì xy = 0.

Trường hợp 2. P ≠ 0 ta có (1) là phương trình bậc hai với ẩn là xy, do đó

để phương trình có nghiệm thì: = 4 – 12P2 ≥ 0 ⇔ -

√ ≤ P ≤

√ .

Vậy MaxP =

√ thì x =

, y =

102

MinP = -

√ thì x = -

√ và y =

.

Ví dụ 4. Cho các số thực x, y. Tìm giá trị nhỏ nhất của biểu thức:

P = 5x2 + y

2 + 4xy – 18x – 12y + 2017

Lời giải. Xét Q = 5x2 + 2(2y – 9)x + y

2 - 12y + m, với m là tham số.

Ta có x = - y2 + 24y – 144 + 225 – 5m = - (y -12)

2 + 225 – 5m.

Ta sẽ sẽ tìm m để Q ≥ 0 ∀x, y tức là ta cần có y ≤ 0 ∀y. Điều này xẩy ra khi m

= 45.

Suy ra P ≥ 1972.

Dấu “=” xảy ra khi x = -3, y = 12.

Vậy MinP = 1972 khi x = -3, y = 12.

Ví dụ 5. Cho x, y, z là các số thực không âm. Chứng minh rằng

xyz + x2

+ y2 + z

2 + 5 ≥ 3(x+y+z).

Lời giải. Ta nhận thấy đẳng thức xảy ra khi x = y = z = 1. Theo nguyên lí

Dirichlet, trong 3 số x, y, z luôn tồn tại 2 số cùng không lớn hơn hoặc cùng

không nhỏ hơn 1. Không mất tính tổng quát giả sử y, z thỏa mãn tính chất đó.

Suy ra (y – 1)(z – 1) ≥ 0 ⇔ yz ≥ y + z – 1 ⇒ xyz ≥ xy + xz – x.

Ta phải chứng minh: f(x) = x2 + (y + z – 4)x + y

2 + z

2 – 3y – 3z + 5 ≥ 0.

Mặt khác ta lại có: = (y + z – 4)2 – 4(y

2 + z

2 – 3y – 3z + 5)

= -(y – z)2 – 2(y – 1)

2 - 2(z – 1)

2 ≤ 0

⇒ 1.f(x) ≥ 0 ∀x ⇒ f(x) ≥ 0. Dấu bằng xảy ra khi x = y = z = 1.

Suy ra đpcm.

Ví dụ 6. Tìm số thực x, y, z thỏa mãn:

x + y + z = 1 (1) và x2 + 2y

2 + 3z

2 = 4 (2)

sao cho x đạt giá trị lớn nhất.

Lời giải. Từ (1) suy ra z = 1 – x – y, thay vào biến đổi ta được

5y2 + 6(x – 1)y + 4x

2 - 6x – 1 = 0. (3)

Để phương trình (3) có nghiệm thì:

= 9(x – 1)2 – 20x

2 + 30x + 5 = -11x

2 + 12x + 14 ≥ 0

⇔ √

≤ x ≤

. Vì x đặt giá trị lớn nhất nên

x = √

⇒ y =

; z =

.

Bài tập.

Bài 1. Chứng minh rằng với mọi số thực x, y thì

103

2x2 + y

2 – xy – 11x + y +16 ≥ 0

Bài 2. Xét hai số thực x, y thỏa mãn x + y = 1. Tìm giá trị lớn nhất, giá trị nhỏ

nhất của biểu thức thỏa mãn: P = ( )

Bài 3. Xét các số thực dương x, y thỏa mãn x2y = 1. Tìm giá trị lớn nhất, giá trị

nhỏ nhất của biểu thức P = x√x + y + x2.

Bài 4. Xét các số thực dương x, y thỏa mãn x2

+ y2 + xy – 6(x + y) + 11

= 0. Tìm giá trị lớn nhất, giá trị nhỏ nhất của biểu thức P = 2x + y.

Bài 5. Xét các số thực a, b, c thỏa mãn a3 > 36 và abc = 1. Chứng minh rằng:

+ b2 + c

2 > ab + bc + ca.

MỘT SỐ KĨ THUẬT SỬ DỤNG

BẤT ĐẲNG THỨC AM - GM

TẠ NGỌC MINH

(Thị trấn Triệu Sơn, Thanh Hóa)

Trong bài viết này chúng tôi giới thiệu một số bài toán khi chứng minh có sử

dụng bất đẳng thức AM – GM và cách áp dụng.

* Bất đẳng thức AM – GM

Với các số thực không âm a, b, c thì: a + b ≥ 2√ab.

Dấu bằng xảy ra khi a = b.

a + b + c ≥ 3√abc

. Dấu bằng xảy ra khi a = b = c.

Bài toán 1. Cho các số thực dương a, b, c thỏa mãn a + b + c = 6.

Tìm giá trị lớn nhất của biểu thức: P = √a + b

+ √b + c

+ √c + a

* Phân tích. Vì có căn bậc 3 nên ta nghĩ đến dùng bất đẳng thức

AM – GM ba số, dự đoán giá trị lớn nhất đạt được khi a = b = c = 2. Khi đó a +

b = b + c = c + a = 4

* Lời giải. Áp dụng bất đẳng thức AM – GM ta có:

√a + b

= √ ( )

√ ≤

( )

√ =

( )

√ .

Tưng tự ta có: √b + c

≤ ( )

√ ; √c + a

( )

√ .

Cộng theo vế các bất đẳng thức trên ta được: P ≤ ( )

√ = 3√4

Dấu bằng xảy ra khi a = b = c = 2.

104

Vậy MaxP = 3√4

khi a = b = c = 2.

Bài toán 2. Cho các số dương a, b, c thỏa mãn a2b

2 + b

2c

2 + c

2a

2 = 12. Tìm giá

trị lớn nhất của biểu thức: Q = √ab c + √bc a

+ √ca b

* Phân tích. Ta dự đoán Q đạt giá trị lớn nhất khi a = b = c = √2.

Lời giải. Áp dụng bất đẳng thức AM – GM ta có:

√ab c = c√ab

≤ c.

= c

.

Tương tự: √bc a ≤ a

; √ca b

≤ b.

Cộng theo vế các bất đẳng thức trên ta được:

Q ≤ ab + bc + ca =

+

+

+

+

=

=

= 6. Dấu bằng xảy ra khi a = b = c = √2

Vậy MaxQ = 6 khi a = b = c = √2

Bài toán 3. Cho các số thực a, b thỏa mãn a2 + b

2 + c

2 = 48

với a, b, c [1;4]. Tìm giá trị nhỏ nhất của biểu thức

Tìm giá trị nhỏ nhất của biểu thức: A = (2a + 2b – c)3 + (2b + 2c – a)

3 + (2c + 2a

– b)3.

Lời giải. Đặt x = 2a + 2b – c; y = 2b + 2c – a; z = 2c + 2a – b

Suy ra x2 + y

2 + z

2 = 9(a

2 + b

2 + c

2) = 9.48 = 432.

Vì a, b, c [1;4] nên a, b, c > 0. Áp dụng bất đẳng thức AM – GM ta có

2x3 + 12

3 = x

3 + x

3 + 12

3 ≥ 3.√x x 12

= 36x2.

Tương tự: 2y3 + 12

3 ≥ 36y

2 ; 2z

3 + 12

3 ≥ 36z

2.

Cộng theo vế các bất đẳng thức trên ta được: 2A ≥ 36(x2 + y

2 + z

2) – 3.12

3

= 36.432 – 3.123 = 10368 ⇒ A ≥ 5184.

Dấu bằng xảy ra khi x = y = z = 12 hay a = b = c = 4.

Vậy MinA = 5184 khi a = b = c = 4.

Bài toán 4. Cho các số dương x, y, z thỏa mãn x + y + z + 2 = xyz. Chứng minh

rằng:

+

+

.

Lời giải. Đặt a =

, b =

, c =

.

Bài toán trở thành: Cho a, b, c dương thỏa mãn ab + bc + ca + 2abc = 1. Chứng

minh rằng: a + b + c ≥

Ta có: (a – b)2 + (b – c)

2 + (c – a)

2 ≥ 0 nên (a + b + c)

2 ≥ 3(ab + bc + ca).

105

Áp dụng bất đẳng thức AM – GM ta có: .

/ ≥ abc

⇒ ( )

+ 2..

/ ≥ ab + bc + ca + 2abc = 1

.(a + b + c -

).[2(a+ b + c)

2 + 12(a + b + c) +18] ≥ 0

⇔ a + b + c ≥

. Dấu bằng xảy ra khi a = b = c =

⇔ x = y = z = 2.

Bài toán 5. Cho các số dương a, b, c thỏa mãn a2 + b

2 + c

2 + (a + b +c)

2

≤ 4. Chứng minh rằng:

( ) +

( ) +

( ) ≥ 3.

Lời giải. Ta có: a2 + b

2 + c

2 + (a + b +c)

2 ≤ 4

⇔ a2 + b

2 + c

2 + ab + bc + ca ≤ 2

Do đó:

( ) =

.

( ) ≥

( ) =

,( ) ( )( )-

( )

=

+

( )( )

( ) .

Chứng minh tương tự:

( ) ≥

+

( )( )

( ) ;

( ) ≥

+

( )( )

( ) .

Cộng theo vế ta được:

( ) +

( ) +

( ) ≤

+

( )( )

( ) +

( )( )

( ) +

( )( )

( ) (1)

Áp dụng bất đẳng thức AM – GM ta có:

( )( )

( ) +

( )( )

( ) +

( )( )

( ) ≥ 3√

( )( )

( ) ( )( )

( ) ( )( )

( )

= 3 (2) .

Từ (1) và (2) suy ra điều phải chứng minh.

Dấu bằng xảy ra khi a = b = c =

√ .

Bài toán 6. Cho các số dương a, b, c thỏa mãn abc = 1. Chứng minh rằng

(a2 + b

2 + c

2)

3 ≥ 9(a

3 + b

3 + c

3)

Phân tích. Ta sẽ đưa hai vế cần chứng minh về cùng bậc.

(a2 + b

2 + c

2)

3 ≥ 9abc(a

3 + b

3 + c

3).

Lời giải. Áp dụng bất đẳng thức AM – GM ta có:

9abc(a3 + b

3 + c

3) = 27.ab.ac.

≤ (ab + ac +

)

3

= (ab + bc + ca +

)

3

= (ab + bc + ca + ( )( )

)

3. Ta sẽ chứng minh:

106

ab + bc + ca + ( )( )

≤ a

2 + b

2 + c

2 (1)

Ta có: (1) ⇔ (a + b + c ab bc ca)(

) ≥ 0. (2)

Ta lại có: a + b + c ab bc ca = ( ) ( ) ( )

≥ 0.

Do đó nếu giả sử a = max*a, b, c+ thì (2) luôn đúng. Suy ra đpcm.

Dấu bằng xảy ra khi a = b = c = 1.

Bài toán 7. Cho các số dương a, b, c thỏa mãn a + b + c = 3. Chứng minh rẳng:

+

+

.

Phân tích. Áp dụng ngay bất đẳng thức AM – GM thì

+

+

+

+

. Mà vế phải của bất đẳng thức này không nhỏ hơn

nên ta

nghĩ đến dùng kĩ thuật AM – GM ngược dấu như sau: -

.

Lời giải. Áp dụng bất đẳng thức AM – GM ta có:

=

( )

= a -

≥ a -

= a -

.

Chứng minh tương tự ta có:

≥ b -

;

≥ c -

.

Cộng theo vế các bất đẳng thức trên ta được:

+

+

≥ a + b + c -

= 3 -

(1)

Ta lại có:

( )

=

=

. (2)

Từ (1) và (2) suy ra đpcm. Dấu bằng xảy ra khi a = b = c = 1.

Bài toán 8. Cho các số dương x, y, z thỏa mãn x + y + z = 3. Tìm giá trị nhỏ

nhất của biểu thức: P =

+

+

.

Lời giải. Áp dụng bất đẳng thức AM – GM ta có:

=

( )

= x -

≥ x -

√ = x -

. (1)

Tương tự:

≥ y -

(2) ;

≥ z -

(3).

Cộng theo vế (1), (2) và (3) ta có:

P =

+

+

≥ (x + y + z) -

√ √ √

= 3 -

√ √ √

(*)

Áp dụng bất đẳng thức AM – GM ta có: y√x = √xy y ≤

(4).

Tương tự ta có: z√y = √yz z ≤

(5) ; x√z = √xz x ≤

(6).

107

Cộng theo vế (4), (5), (6) ta có: y√x + z√y + x√z ≤ ( ) ( )

= ( )

=

( )

( )

=

=

= 3. (7)

Từ (*) và (7) suy ra: P ≥ 3 -

=

. Dấu bằng xảy ra khi x = y = z = 1.

Vậy MinP =

khi x = y = z = 1.

Bài tập.

Bài 1. Cho các số thực dương a, b, c thỏa mãn a2 + b

2 + c

2 = 3. Tìm giá trị lớn

nhất của biểu thức: P =

√ +

√ +

√ .

Bài 2. Cho các số dương x, y, z thỏa mãn x + y + z = xyz. Chứng minh rằng (x +

y + z)xyz ≥ 4(xy + yz + zx).

Bài 3. Cho các số thực dương a, b, c thỏa mãn a + b + c = 3abc. Tìm giá trị lớn

nhất của biểu thức: Q =

+

+

.

Bài 4. Cho các số thực dương a, b, c, d thỏa mãn a + b + c + d = 3. Tìm giá trị

lớn nhất của biểu thức: A =

+

+

+

.

Bài 5. Cho các số thực dương a, b, c, d thỏa mãn a + b + c + d = 4. Tìm giá trị

lớn nhất của biểu thức: B =

+

+

+

.

Bài 6. Cho các số thực dương x, y, z. Tìm giá trị lớn nhất của biểu thức

M = √

√ +

√ +

√ .

MỘT PHƢƠNG PHÁP

CHỨNG MINH BẤT ĐẲNG THỨC

NGUYỄN KHÁNH TOÀN

(GV THCS Bác Hải, Tiến Hải, Thái Bình)

Trong quá trình chứng minh bất đẳng thức(BĐT). Chúng ta gặp các BĐT ở mà

đó các biến được hoán vị vòng quanh hay các biến có vai trò như nhau. Bài viết

này, xin giới thiệu tới bạn đọc phương pháp khá hiêu quả để giải quyết một số

bài toán thuộc dạng này thông qua các thí dụ sau.

Thí dụ 1. Cho x, y, z > 0. Chứng minh rằng:

√x + xy + 2y + √y + yz + 2z + √z + zx + 2x ≥ 2(x + y + z)

Phân tích. Trong BĐT (1) các biến được hoán vị vòng quanh và đẳng thức xảy

ra khi x = y = z. Do vậy, nếu ta chọn được các số a, b để có BĐT

108

√x + xy + 2y ≥ a(x + by) (1*)

Thì nó là cơ sở để suy ra BĐT (1*). Với x = y thì BĐT (1*) trở thành

√x + x + 2x ≥ a(x + by) ⇔ 2 ≥ a(b + 1).

Từ đó ta có thể chọn a =

, b ≠ - 1, khi đó BĐT (1*) có dạng

√x + x + 2x ≥

(x + by)

⇔ (b2 + 2b – 3)x

2 + (b

2 – 6b + 1)xy + (-2b

2 + 4b + 2) ≥ 0.

Đặt t =

> 0, BĐT trên trở thành:

(b2 + 2b – 3)t

2 + (b

2 – 6b + 1)t + (-2b

2 + 4b + 2) ≥ 0.

⇔ (b2 + 2b – 3)(t – 1)(t -

– ) ≥ 0.

Để BĐT (1*) đúng, ta chọn b sao cho: b2 + 2b – 3 > 0 và

– = 1

⇔ b =

⇒ a =

.

Lời giải. Với x, y > 0, ta có: √x + xy + 2y ≥

(a)

⇔ (x – y)2 ≥ 0 (luôn đúng)

Tương tự ta có: √y + yz + 2z ≥

(b); √z + zx + 2x ≥

(c)

Cộng theo vế ba BĐT (a), (b), (c) ta thu đươc BĐT (1). Đẳng thức xảy ra khi x =

y = z.

Thí dụ 2. Cho a, b, c > 0 và a + b + c = 3. Chứng minh rằng:

+

+

(2)

Phân tích. Trong BĐT (2) các biến có vai trò như nhau và đẳng thức xảy ra khi

a = b = c. Do vậy nếu ta chọn được các số x, y để có BĐT

≥ xa + yb (2*) thì nó là cơ sở để suy ra bất đẳng thức (2)

Với a= b thì BĐT (2*) trở thành:

≥ (x + y)a. Từ đây ta có thể chọn

y =

x, Khi đó (2*) có dạng

≥ xa + (

x)b

⇔ (1 – x)a2 -

ab + (x -

)b

2 ≥ 0. Đặt t =

> 0 BĐT trên trở thành:

(1 – x)t2 -

t + (x -

) ≥ 0 ⇔ (1 – x)(t – 1)(t -

– ) ≥ 0.

Để chứng tỏa (2*) đúng ta chọn x sao cho: 1 – x > 0 và

– = 1

109

⇔ x =

⇒ y =

.

Lời giải. Với a, b > 0 ta có:

⇔ (a – b)

2 ≥ 0 (luôn đúng)

Tương tự với a, b, c > 0, ta cũng có:

;

.

Cộng theo vê ba BĐT trên thu gọn ta đươc BĐT (2). Đẳng thức xảy ra khi a = b

= c = 1.

Thí dụ 3. Cho a, b, c > 0. Chứng minh rằng:

+

+

. (3)

Phân tích. Trong BĐT (3) các biến được hoán vị vòng quanh và đẳng thức xảy

ra khi a = b = c. Như vậy, nếu ta chọn được các số x, y để có BĐT

≥ xa + yb (3*) thì nó là cơ sở để chứng minh BĐT (3).

Theo BĐT (3*) ta có:

+

+

≥ (x + y)(a + b + c).

Để có BĐT (3)chọn x, y sao cho x + y =

⇔ x =

– y.

Như vậy, cần tìm y để có BĐT:

≥ (

– y)a + yb.

⇔ (

+ y)a

3 -

a

3b -

ab

2 – yb

3 ≥ 0 ⇔ (

+ y)X

3 -

X

2 -

X – y ≥ 0 (X =

)

⇔(X – 1)((

+ y)X

2 + (

+ y)X + y) ≥ 0.

Khi đó tam thức bậc hai )((

+ y)X

2 + (

+ y)X + y có nghiệm X = 1 thì BĐT

(3*) đúng. Từ đó ta tìm được y = -

và x =

Lời giải. Với a, b > 0 ta có:

⇔ (a – b)

2 ≥ 0 (luôn đúng).

Tương tự:

;

.

Cộng theo vế ba BĐT trên và thu gọn ta được BĐT (3).

Đẳng thức xảy ra khi và chỉ khi a = b = c.

Thí dụ 4. Cho a, b, c> 0. Chứng minh rằng:

+

+

(4)

Phân tích. Trong BĐT (4) các biến được hoán vị vòng quanh và đẳng thức xảy

ra khi a = b = c. Do vậy nếu ta chọn được các số x, y để có BĐT:

≥ xa + yb (4*) thỉ đó là cơ sở để chứng minh BĐT (4).

110

Theo BĐT (4*) ta có:

+

+

≥ (x + y)(a + b + c).

Để có BĐT (4) chọn x, y sao cho x + y =

⇔ x =

– y.

Như vậy ta cần tìm y để có BĐT sau:

≥ (

– y)a + yb

⇔ (

+ y)a

4 – ya

3b + (2y -

)ab

3 – 2yb

4 ≥ 0

⇔ (

+ y)X

4 – yX

3 + (2y -

)X – 2y ≥ 0 (X =

> 0)

⇔ (X – 1)((

+ y)X

3 +

X

2 +

X + 2y) ≥ 0.

Nhận thấy đa thức: (

+ y)X

3 +

X

2 +

X + 2y có nghiệm X = 1 thì BĐT (4*)

luôn đúng. Từ đó ta tìm được y = -

và a = 1.

Lời giải. Áp dụng BĐT Cô – si, ta có:

= a -

≥ a -

= a -

(d)

Tương tự:

≥ b -

(e);

≥ c -

(f)

Cộng theo vế các BĐT (d), (e) và (f) ta được BĐT (4)

Đẳng thức xảy ra khi và chỉ khi a = b = c.

Thí dụ 5. Cho a, b, c > 0, a2 + b

2 + c

2 = 1. Chứng minh rằng:

+

+

(5)

Hƣớng dẫn. Dễ thấy BĐT (5) là hệ quả của BĐT:

+

+

(a

2 + b

2 + c

2).

Do đó ta cần Chứng minh BĐT:

a

2. Chứng minh BĐT (5

*) luôn đúng.

Xét hiệu: H =

-

a =

-

a

2 =

(√ )(√ )

( ).

Vì a (0;1). Từ đó suy ra BĐT (5).

Bài tập.

1. Cho a, b, c > 0 và

+

+

= 1. Chứng minh rằng

+

+

≥ √3.

2. Cho x, y, z > 0 và xy + yz + zx = 1. Chứng minh rằng:

x√2y + 3z + y√2z + 3x + z√2x + 3y ≥ √5.

3. Cho x, y, z > 0 và xyz = 1. Chứng minh rằng:

111

x√y + 2z + y√z + 2x + z√x + 2y ≥ 3√3.

4. Cho x, y, z > 0. Chứng minh rằng:

+

+

≥ √3.

5. Cho a, b, c > 0. Chứng minh rằng:

a)

+

+

≥ 0.

b)

+

+

≥ a + b + c.

6. Cho a, b, c > 0. Chứng minh rằng

a)

( ) +

( ) +

( ) ≥

.

b)

+

+

.

7. Cho a, b, c > 0. Chứng minh rằng

+

+

.

8. Cho a, b, c. Chứng minh rằng:

a)

+

+

.

b)

+

+

≥ 1.

9. Cho a, b, c, d và ab + bc + ca + da = 4. Chứng minh rằng:

+

+

+

.

KĨ THUẬT ĐẢO DẤU KHI CHỨNG MINH

BẤT ĐẲNG THỨC CHỨA PHÂN THỨC

CAO HẢI VÂN

(GV.THPT Nguyễn Chí Thanh, Gia Lai)

PHAN ĐÌNH ÁNH

(GV. THCS Thạch Kim, Lộc Hà, Hà Tĩnh)

Trong một số bài toán Chứng minh BĐT, việc sử dụng các bất đẳng thức cơ sở

hay các kết quả đã có trước đó là hết sức cần thiết. Tuy nhiên nhiều bài toán khi

sử dụng kết quả đó cho chúng ta dấu của BĐT ngược chiều. Một cách có thể

khác phục được vấn đề trên là tìm cách đảo dấu BDDT cần Chứng minh. Trước

hết ta có hai nhận xét sau:

Nhận xét 1. Với a, b > 0 thì theo BĐT AM – GM ta có:

112

a + b ≥ 2√ab ⇒

√ ⇒

√ .

Nhận xét 2. Với a, b, c > 0 theo BĐT bunhicopxki ta có:

(a + b + c)(

+

+

) ≥ (x + y + z)

2 ⇒

+

+

( )

.

Sau đây chúng tôi đưa ra một số ví dụ minh họa cho kĩ thuật trên.

Ví dụ 1. Cho các số dương a, b, c thỏa mãn a + b + c = 3. Chứng minh rằng:

+

+

.

Chú ý. Nếu ta áp dụng ngay BĐT AM – GM thì ta có:

+

+

+

+

(?)

Giải. Áp dụng BĐT AM – GM ta có:

=

( )

= a -

≥ a -

= a -

Tương tự ta có:

≥ b -

;

≥ c -

Cộng theo vế các BĐT trên ta được:

+

+

≥ (a + b + c) -

≥ (a + b + c) - ( )

= 3 -

=

Đẳng thức xảy ra khi a = b = c = 1.

Ví dụ 2. Cho các số dương a, b, c, d thỏa mãn a + b + c + d = 4. Chứng minh

rằng:

+

+

+

≥ 2.

Giải. Áp dụng BĐT AM – GM ta có:

=

( )

= a -

≥ a -

= a - √

≥ a -

≥ a -

( )

= a -

Chứng minh tương tự và cộng theo vế các BĐT nhận được ta có:

+

+

+

≥ (a + b + c) -

.(ab + bc + cd + ad + abc + bcd +

cda + dab). Ta lại có: ab + bc + cd + da ≤

.(a + b + c + d)

2 = 4

abc + bcd + cda + dab = cb(a + d) + ad(c + b) ≤

.(c + b)

2(a + d) +

.(a + d)

2(c + b)

=

(a + d)(c + b)(a + b + c + d) ≤

.(a + b + c + d)

3 = 4 (đpcm)

Ví dụ 3. Cho các số dương a, b, c. Chứng minh rằng:

+

+

Giải. Áp dụng BĐT AM – GM ta có:

= a -

≥ a -

= a -

.

Chứng minh tương tự và cộng theo vế các BĐT ta được điều phải chứng minh.

Ví dụ 4. Cho các số dương x, y, z thỏa mãn x + y + z = 3. Chứng minh rằng:

113

+

+

.

Giải. Ta có:

+

+

= 3 – (

+

+

)

Mặt khác theo nhận xét 2 ta có:

+

+

=

=

Suy ra điều phải chứng minh.

Chú ý. Trong thí dụ 4 đã sử dụng kĩ thuật đảo dấu để có thể áp dung nhận xét 2

và thí dụ này sẽ được áp dụng cho các ví dụ sau đây.

Ví dụ 5. (Rumani 1997) Cho 3 số thực dương a, b, c. Chứng minh rằng:

+

+

≤ 1. (1)

Giải. Ta có: (1) ⇔

+

+

≤ 2

⇔ 3 – (

+

+

) ≤ 2 ⇔

+

+

≥ 1.

Mặt khác theo nhận xét 2 ta có:

+

+

( )

=

( )

( ) = 1 (Đpcm)

Ví dụ 6. (Canada 2008) Cho ba số thực dương a, b, c thỏa mãn a + b + c = 1.

Chứng minh rằng:

+

+

Giải. Ta có: (1) ⇔ 3 -

-

-

+

+

.

Mặt khác theo nhận xét 2 ta có:

+

+

=

+

+

( )

=

( )

( )

= ( )

( ) ≥

(Do abc = abc(a + b + c) ≤

(ab + bc + ca)

2 ).

Ví dụ 7. Cho a, b, c dương. Chứng minh rằng:

+

+

≤ 1.

Giải. Áp dụng BĐT AM – GM ta có:

=

(1 -

)

Chứng minh tương tự cộng theo vế các BĐT ta được:

+

+

– (

+

+

)

Mặt khác theo nhận xét 2 ta có:

+

+

=

+

+

≥ ( )

=

( )

( ) = 1. Suy ra điều phải chứng minh.

M t số bài toán vận dụng.

114

Bài 1. Cho a, b, c dương thỏa mãn điều kiện a + b + c = 3. Chứng minh rằng:

+

+

≥ 1.

Bài 2. Cho các số dương a, b, c, d thỏa mãn điều kiện a + b + c + d = 4. Chứng

minh rằng:

+

+

+

≥ 2.

Bài 3. Cho 3 số dương a, b, c, d thỏa mãn điều kiện a2 + b

2 + c

2 = 1. Chứng

minh rằng:

+

+

Bài 4. Cho 3 số thực không âm a, b, c thỏa mãn a2 + b

2 + c

2 = 1. Chứng minh

rằng: 1 ≤

+

+

.

Bài 5. Cho a, b, c > 0 thỏa mãn điều kiện

+

+

= 2. Chứng minh

rằng: ab + bc + ca ≤

.

Bài 6. (Thi HSG Hà Tĩnh năm học 2011 – 2012)

Cho các số thực dương x, y, z thỏa mãn: x + y + z =

. Tìm giá trị nhỏ nhất của

biểu thức: P = √

+

+

.

TÌM GIÁ TRỊ LỚN NHẤT VÀ GIÁ TRỊ NHỎ NHẤT

CỦA ĐA THỨC BẬC HAI

GV. VŨ ĐỨC CẢNH (Ninh Bình)

Các bạn thân mến trong chương trình toán lớp 8 – 9, các bạn có thể gặp bài toán

tìm giá trị lớn nhất (GTLN), giá trị nhỏ nhất (GTNN) của đa thức bậc 2 chẳng

hạn như:

Thí dụ 1. Tìm GTNN của biểu thức: M = x2 + 2y

2 – 2xy + 2x – 10y.

Lời giải. Có thể giải bài toán này bằng cách đưa các biến vào các bình phương

tổng, có thể dựa vào hằng đẳng thức: (a + b + c)2 = a

2 + b

2 + c

2 + 2ab + 2bc +

2ca.

Ta có: M = x2 + y

2 + 1 – 2xy + 2x – 2y + y

2 – 8y + 16 – 17

= (x – y + 1)2 + (y – 4)

2 - 17 ≥ -17.

Khi x = 3 và y = 4 thì M = -17. Vậy Min M = -17.

Dưới đây sẽ nêu ra một số phương pháp tìm GTLN, GTNN của đa thức bằng

cách đưa dần các biến vào trong các bình phương của tổng theo hằng đẳng thức:

a2 + 2ab + b

2 = (a + b)

2.

Có thể giải thí dụ 1 như sau:

115

M = (x2 - 2xy + y

2) + (2x – 2y) + 1 + (y

2 – 8y + 16) – 17

= [(x - y)2 + 2(x – y) + 1] + (y – 4)

2 – 17 = (x – y + 1)

2 + (y – 4)

2 – 17 ≥ -17. Khi

x = 3 và y = 4 thì M = -17.

Vậy GTNN của M là -17.

Thí dụ 2. Tìm GTNN của biểu thức: A = (x + 1)2 + (x – 3)

2.

Lời giải. Ta có: A = (x2 + 2x + 1) + (x

2 – 6x + 9) = 2x

2 – 4x + 10

= 2(x2 – 2x + 1) + 8 = 2(x – 1)

2 + 8 ≥ 8. Dấu “=” xảy ra khi x = 1

Vậy min A = 8

Thí dụ 3. Tìm GTNN của biểu thức B = x3 + y

3 + x

2 + y

2, trong đó x, y thỏa

mãn điều kiện x + y = 1

Lời giải. Do (x + y)3 = x

3 + y

3 + 3xy(x + y) và (x + y)

2 = x

2 + 2xy + y

2 nên ta có:

B = (x + y)3 – 3xy(x + y) + (x + y)

2 – 2xy = 2 – 5xy = 2 – 5x(1 – x) = 5x

2 – 5x +

2 = 5(x2 – x +

) +

= 5(x -

)

2 +

.

Khi x = y =

thì B =

. Vậy Min B =

Thí dụ 4. Tìm GTLN của biểu thức C = xy + yz + zx, trong đó x, y, z thỏa mãn

điều kiện x + y + z = 3.

Lời giải. Ta có: C = xy + z(x + y) = xy + [3 – (x + y)](x + y) = xy + 3(x + y) –

(x + y)2 = - x

2 – y

2 – xy + 3x + 3y = - x

2 – (y – 3)x + 3y – y

2 = -[x

2 + (y – 3)x +

(

)

2] + 3y – y

2 + (

)

2 = -(x +

)

2

+

= -(x +

)

2 +

( )

= -(x +

)

2 -

(y – 1)

2

+ 3 ≤ 3. Khi x = y = z thì C = 3. Vậy Mix C = 3.

Thí dụ 5. Tìm GTNN của biểu thức D = x2 + 6y

2 + 14z

2 – 8yz + 6zx + 4xy.

Lời giải. Ta có: D = x2 + 6y

2 + 14z

2 – 8yz + 6zx + 4xy

= x2 + 2(3z – 2y)x + 6y

2 + 14z

2 – 8yz

= [x2 + 2(3z – 2y)x + (3z – 3y)

2] + 6y

2 + 14z

2 – 8yz – (3z – 3y)

2

= (x + 3z – 2y)2 + 2y

2 + 4yz + 5z

2 = (x + 3z – 2y)

2 + 2(y

2 + 2yz + z

2) + 3z

2 = (x

+ 3z – 2y)2 + 2(y + z)

2 + 3z

2 ≥ 0.

Khi x = y = z = 0 thì D = 0. Vậy Min D = 0.

Thí dụ 6. Tìm GTNN của biểu thức:

E = x2 + 2y

2 + 3z

2 – 2xy + 2xz – 2x – 2y – 8z + 1998.

Lời giải. Ta có: E = x2 – 2x(y – z + 1) + 2y

2 + 3z

2 – 2y – 8z + 1998

= [x2 – 2x(y – z + 1) + (y – z + 1)

2] + 2y

2 + 3z

2 – 2y – 8z + 1998 – (x + y + z)

2

116

= [x – ( y – z + 1)]2 + y

2 + 2z

2 + 2yz – 4y – 6y + 1997

= (x – y + z – 1)2 + (y

2 + 2y(z – 2) + (z – 2)

2 ) + 2z

2 – 6y + 1997 – (z – 2)

2

= (x – y + z – 1)2 + (y + z – 2)

2 + z

2 – 2z + 1993

= (x – y + z – 1)2 + (y + z – 2)

2 + (z – 1)

2 + 1992 ≥ 1992.

Khi x = y = z = 1 thì E = 1992.

Cuối cùng mời các bạn luyện tập với một số bài tập sau:

Bài 1. Cho x + 2y = 3. Tìm GTNN của biểu thức M = x2 + 2y

2.

Bài 2. Cho a, b, c, d là bốn số thực thỏa mãn a + b = c + d . Tìm GTNN của biểu

thức N = (x + a)(x + b)(x + c)(x + d).

Bài 3. Cho x + y + z = 3. Tìm GTNN của biểu thức:

P = x2 + y

2 + z

2 + xy + yz + zx

Bài 4. Biết rằng x ≠ 0. Tìm GTLN của biểu thức: Q =

.

Bài 5. Tìm GTNN của biểu thức: R = x2 + y

2 + xy + x + y.

V. Sử dụng biểu thức liên hợp để tìm giá trị lớn nhất, nhỏ nhất của biểu

thức.

Ví dụ 12. Cho các số thực x, y thỏa mãn điều kiện: √x 1 - y√y = √y 1 -

x√x. Tìm giá trị nhỏ nhất của biểu thức:

S = x2 + 3xy – 2y

2 – 8y + 5.

Lời giải. ĐK: x ≥ 1; y ≥ 1.

* Nếu x = y = 1 ⇒ S = -1.

* Xét x ≥ 1; y ≥ 1 và x ≠ 1 hoặc y ≠ 1. Ta có:

√x 1 - y√y = √y 1 - x√x ⇔ (√x 1 - √y 1) + (x√x - y√y) = 0

√ √ + (√x - √y)(x + √xy + y) = 0

⇔ (√ √ )(√ √ )

√ √ + (√x - √y)(x + √xy + y) = 0

⇔ (√x - √y)( √ √

√ √ + x + √xy + y) = 0 ⇔ √x - √y = 0

(A = √ √

√ √ + x + √xy + y > 0 với ∀x, y ≥ 1 { x ≠ 1 hoặc y ≠ 1)

⇔ x = y.

Khi đó, ta có: S = 2x2 – 8x + 5 = 2(x – 2)

2 – 3 ≥ -3.

S = -3 khi x = y = 2. Vậy minS = - 3 khi x = y = 2

Ví dụ 13. Cho các số thực x, y thỏa mãn điều kiện: √x + 2 – y3 = √y + 2 – y

3.

117

Tìm giá trị lớn nhất của biểu thức: P = x2 - 2xy + 4x – y

2 + 2017.

Lời giải. ĐK: x ≥ - 2, y ≥ -2.

* Nếu x = y = -2 ⇒ P = 2001.

* Nếu x ≥ - 2, y ≥ -2 và x ≠ -2 hoặc y ≠ -2.

Ta có: √x + 2 – y3 = √y + 2 – x

3 ⇔ (√x + 2 √y + 2) + (x

3 – y

3) = 0

√ √ + (x – y)(x

2 + xy + y

2) = 0

⇔ (x – y)(

√ √ + x

2 + xy + y

2) = 0

⇔ x – y = 0

(Do A=

√ √ + x

2 + xy + y

2 > 0 với ∀ x ≥ - 2, y ≥ -2 và x ≠ -2 hoặc y ≠ -2)

⇔ x = y .

Khi đó ta có: P = -2x2 + 4x + 2017 = 2019 – 2 (x – 1)

2 ≤ 2019

P = 2019 khi x = y = 1.

Dạng 3. Vận dụng định lý Vi – et vào một số bài toán chứng minh BĐT, tìm

GTLN và GTNN

Thí dụ 4. Gia sử cho 3 số thực a, b, c thỏa mãn điều kiện a > 0, bc = a2 , a + b +

c = abc. Chứng minh rằng: a ≥ √ √

.

Lời giải. Ta có: bc = a2 , b + c = abc – a = a

3 – a. Theo định lý Vi- et đảo thì b,c

là nghiệm của PT: x2 – (3a

3 – a)x + 3a

2 = 0 (*)

PT (*) có nghiệm khi và chỉ khi = a2(9a

4 – 6a

2 – 11) ≥ 0 ⇒ a

2 ≥

Kết hợp a > 0 ta được: a ≥ √ √

Thí dụ 5. Cho a, b, c là 3 số thực thỏa mãn a ≠ 0 và 4a + 9b + 24c = 0. Tính khỏa

cách (GTTĐ) nhỏ nhất của hai nghiệm của phương trình 2ax2 + 3bx + c = 0

Lời giải. Từ 4a + 9b + 24c = 0 ⇒ c = -

.

Phương trình đã cho có: = 9b2 – 32ac = 9b

2 + 32a(

)= 9(b+

)

2 +

> 0

(do a ≠ 0), nên phương trình đã cho luôn có hai nghiệm phân biệt x1, x2

Theo định lý Vi – et ta có: x1 + x2 = -

; x1.x2 =

= -

.

Do đó khoảng cách giữa hai nghiệm phương trình đã cho là:

|x1 – x2| = √(x + x ) 4x x = √.

/ + .

/ = √.

/ +

118

Suy ra: |x1 – x2| ≥ √

. Vậy khoảng cách nhỏ nhất hai nghiệm phương trình đã cho

là √

khi và chỉ khi 2a = -3b = -24c. Khi đó PT đã cho trở thành:

6ax2 – 6ax – a = 0 ⇔ x =

.

Thí dụ 6. Cho phương trình ax2 + by + c = 0 (a ≠ 0) có hai nghiệm m, n thỏa

mãn 0 ≤ m ≤ n ≤ 1. Tìm giá trị lớn nhất của biểu thức P =

.

Lời giải. Từ giả thiết 0 ≤ m ≤ n ≤ 1 ⇒ m2 ≤ mn và n

2 ≤ 1. Suy ra: (m + n)

2 -2mn

≤ mn + 1 ⇒ (m + n)2 ≤ 3mn + 1. Theo địnhh lý Vi-et ta có:

m + n =

; mn =

. Do đó: P =

.

/

= ( ) ( )

( )

≤ ( )

( ) = 3.

Đẳng thức xảy ra khi 2 = n = 1

hoặc m = n = 1 tức là 2b = a c =

hoặc 2c = - b = 2a

Vậy MaxP = 3 khi 2b = a c =

hoặc 2c = - b = 2a

Nhận xét. Bài toán này chúng ta thường hay sử dụng BĐT cổ điển hay dùng

(BĐT cô-si, BĐT bunhiacopxki, BĐT tam giác, ….) hoặc các tính chất của BĐT

cùng với việc vận dụng định lý Vi- et một cách nhuần nhuyễn sẽ giúp chúng ta

tìm ra được lời giải bài toán ngắn gọn đọc đáo thú vị.

PHẢI LÀM THƢỜNG XUYÊN

NGUYỄN ĐỨC TẤN (TP.Hồ Chí Minh)

Trong toán học việc đề xuất các bài toán tương tự , bài toán mới, bài toán tổng

quát là cong việc mà người yêu toán phải làm thường xuyên. Chúng ta cùng làm

điều này với bài toán sau:.

Bài toán. Cho x, y là các số thực thỏa mãn 2 ≤ x ≤ 10, 2 ≤ y ≤ 10 và xy = 16.

Tìm giá trị lớn nhất của biểu thức A = x + y

Tìm tòi lời giải. Ta dự đoán A đạt giá trị lớn nhất khi xảy ra ít nhất một trong

các dấu bằng của x, y tại các đầu mút. Với x = 2 thì y = 8 và A = 10

Với x = 10 thì y =

không thỏa mãn y ≥ 2.

Hơn nữa vì vai trò bình đẳng của x, y ta nghĩ đến (x – 2)(y – 2) ≥ 0.

Từ đó 2(x + y) ≤ xy + 4. Từ đó ta có lời giải sau.

Lời giải. Do x ≥ 2, y ≥ 2 ⇒ x – 2 ≥ 0, y – 2 ≥ 0. Do đó (x – 2)(y – 2) ≥ 0

119

Suy ra 2(x + y) ≤ xy + 4 mà xy ≤ 16 nên x + y ≤ 10. Đẳng thức xảy ra khi và chỉ

khi x = 2 và y = 8 hoặc x = 8 và y = 2

Vậy MaxA = 10 khi x = 2 và y = 8 hoặc x = 8 và y = 2.

Nhận xét. Ta có các đẳng thức

+

=

, x

2 + y

2 = (x + y)

2 – 2xy,

+

=

; x

3 + y

3 = (x + y)

3 – 3xy(x + y) ;

+

=

;

+

=

Ta có một số bài toán tương tự:

Bài toán. Cho x, y là các số thực thỏa mãn 2 ≤ x ≤ 10, 2 ≤ y ≤ 10 và xy = 16.

Tìm giá trị lớn nhất của biểu thức B =

+

; C = x

2 + y

2 ; D =

+

E = x3 + y

3 ; F =

+

; G =

+

Nhận xét. Tăng số biến lên ta có bài toán như sau.

Bài toán. Cho x, y, z là số thực thỏa mãn 2 ≤ x, y, z ≤ 10 và xy + yz + zx = 36.

Tìm giá trị lớn nhất của biểu thức M = x + y + z

Nhận xét. Tổng quát ta có bài toán sau.

Bài toán. Cho x1, x2, .., xn là các số thực thỏa mãn 2 ≤ x1, x2, .., xn ≤ 10 và

x1x2 + x2x3 + … + xn- 1.xn = 4n + 24. Tìm giá trị lớn nhất của biểu thức P = x1 +

x2 + x3 + …+ xn

Nhận xét. Nếu tiếp tục tìm kiếm, mở rộng theo các hướng ta sẽ có nhiều bài

toán mới. Chúc các bạn thành công!

VẬN DỤNG LINH HOẠT PHƢƠNG PHÁP BIẾN ĐỔI TƢƠNG ĐƢƠNG

TRONG CHỨNG MINH BẤT ĐẲNG THỨC

Tác giả: Nguyễn Quốc Bảo

(AD của Page tài liêu khối a,b,d)

I. Lời nói đâu:

Hôm nay tôi sẽ giới thiệu một kĩ thuật về chứng minh bất đẳng thức mà ít

được nói kĩ trong các bài giảng cũng như sách khảm khảo, nhưng cực kì hiệu

quả và dễ hiểu, dễ làm trong việc chứng minh nhiều bất đẳng thức ở THCS.

II. Trình tự các bước làm:

- Đưa bất đẳng thức về một ẩn

- Xác định điều kiện ẩn, dự đoán dấu bằng của bất đẳng thức hay

biểu thức cực trị

- Biến đổi tương đương

120

III. Bài tập:

Bài 1: Cho a, b, c, d là các số thực dương. Tìm giá trị nhỏ nhất của:

A =

√ +

Bài làm:

Theo BĐT Cô – si: a + b ≥ 2√ab ; c + d ≥ 2√cd

Suy ra: a + b + c + d ≥ 2√ab + 2√cd ≥ 2√2√ab 2√cd =4.√abcd

Do đó:

√ ≥ 4 . Đặt: t =

√ (t≥4)

Khi đó bất đẳng thức trở thành: A = f(t) = t +

với (t≥4).

(Với t [4;+∞) trong ta dự sẽ đoán A đặt giá trị nhỏ nhất điểm mút t = 4 khi

đó A =

, biến đổi tương đương theo nhân tử t - 4)

Ta sẽ chứng minh: A ≥ f(4) ⇔ t +

(*)

Thật vậy: (*) ⇔

≥ 0 ⇔

( )( )

≥ 0 (đúng với mọi t ≥ 4)

Vậy MinA =

khi t = 4 tức a = b = c = d.

Bài 2: (Chuyên Lam Sơn – Thanh Hóa) Tìm giá trị nhỏ nhất của biểu thức: A

= ( +

) + ( +

) với u + v = 1 và u > 0 và v > 0

Bài làm:

Ta có: ( + ) ≥ 4uv suy ra uv ≤ ( )

=

A = ( +

) + ( +

) = u

2 + 2 +

+ v

2 + 2 +

⇒ A = (u2 + v

2) + (

+

) + 4 = [(u+v)

2 – 2uv] +

+4

= 1 – 2uv +

+4 =

Đặt t =uv. Ta có: A = f(t) =

với t ≤

(Cũng như bài trên ta dự đoán A đạt giá trị nhỏ nhất tại điểm mút t =

khi đó

thay vào A ta được A =

, biến đổi tương phân tích theo nhân tử 4t -1)

Ta sẽ chứng minh: A ≥ f(

) ⇔

(*)

Thật vậy: (*) ⇔

≥ 0

⇔ ( )( )

≥ 0 ( đúng với mọi t ≤

)

121

Vậy MinA =

Khi t =

hay u = v =

Bài 3: (Vào lớp 10 Trường ĐHKHTN, ĐHQG Hà Nội)

Số thực x thay đổi và thỏa mãn điều kiện x2 + (3 – x)

2 ≥ 5. Tìm giá trị nhỏ nhất

của biểu thức: P = x4 + (3-x)

4 + 6x

2.(3-x)

2

Bài làm

Đặt y = 3- x ta có: x+ y = 3 và x2 + y

2 ≥ 5

Ta có: P = [ x2 + (3 – x)

2]

2 + 4x

2(3-x)

2

=[ x2 + (3 – x)

2]

2 + {[ x + (3 –x)]

2 – [x

2 + (3-x)

2]}

2

=(x2+y

2)

2 + [(x+y)

2 – (x

2 +y

2)]

2

= (x2+y

2)

2 + [ 9 – (x

2 + y

2)]

2

= 2.(x2 + y

2)

2 - 18.(x

2 + y

2) + 81

Đặt t = x2 + y

2 với t ≥ 5

Ta có bài toán trở thành: Tìm Min của P = f(t) = 2t2 - 18t + 81 với t ≥ 5

(Cũng giống như các bài toán trước ta dự đoán đại giá trị nhỏ nhất tại t=5 khi đó

P = 41, ta chứng minh tương đương dồi về nhân tử t-5)

Ta sẽ chứng minh: P ≥ f(5) ⇔ 2t2 - 18t + 81 ≥ 41 (*)

Thật vậy: (*) ⇔ t2 – 9t +20 ≥ 0

⇔ (t-5)(t-4) ≥ 0 ( đúng với mọi t ≥ 5)

Vậy Min P = 41 khi t=5 hay x2 + (5 – x)

2 = 5

Hay Min P = 41 khi x= 1 hay x= 2.

Bài 4: (Chuyên Trần Đại Nghĩa – TP. Hồ Chí Minh)

Cho x ≥ 0 , y ≥ 0, z ≥0 thỏa mãn : 4x + y + 2z = 4 và 3x + 6y – 2z = 6

Tìm giá nhỏ nhất và lớn nhất và nhỏ nhất của Biểu thức: A = 5x – 6y + 7z

Bài làm :

Từ giả thiết ta có: y + 2z = 4 – 4x và 6y – 2z = 6 – 3x. Giải hệ này ta được: y =

– x và z =

x +

Vì y ≥ 0 và z≥0 nên:

– x ≥ 0 và

x +

≥ 0 do đó : x ≤

Ta có: A = 5x – 6y + 7z = 5x – 6.(

– x ) + 7.(

x +

) =

x +

Vậy cần tim Min và Max của A = f(x) =

x +

Với 0 ≤ x ≤

A =

x +

. Vậy Min(A) =

Khi x=0

A =

x +

.

+

=

. Max(A) =

Khi x=

122

Bài 5: (Chuyên Hưng Yên – Hưng Yên)

Cho a, b > 0 và a + b = 1. Chứng minh rằng: P =

+

≥ 14

Bài làm:

Ta có: a2 + b

2 ≥

( )

=

P =

+

=

+

=

( ) ( ) +

=

( ) +

Đặt t = a2 + b

2 ( t ≥

)

Ta có bài toán trở thành:

Chứng minh: P = f(t) =

+

≥ 14 (*) với t ≥

Thật vậy: (*) ⇔ 4t + 3(1 – t) ≥ 14.t.(1 – t)

⇔ 14t2 – 13t + 3 ≥ 0 ⇔ (2t – 1)(7t – 3) ≥ 0 (luôn đúng với mọi t ≥

)

Vậy bất đẳng thức được chứng minh, dấu“=”xẩy ra khi t =

hay a = b =

Bài 6: (Thi học sinh giỏi Hà Tĩnh 2014) Cho 2 số thực không âm x, y thỏa mãn:

x + y = 2. Chứng minh rằng:

2 ≤ √x + y + √xy ≤ √6

Bài làm:

Ta có: Theo Cô– si: 2 = x + y ≥ 2√xy ⇒ 0 ≤ xy ≤ 1

(do x, y không âm nên xy ≥ 0)

Đặt P = √x + y + √xy

Ta có P = √x + y + √xy = √(x + y) 2xy + √xy

= √4 2xy + √xy

Đặt xy = t. Bài toán trở thành chứng minh:

2 ≤ √4 2t + √t ≤ √6 với 0 ≤ t ≤ 1

Thật vậy: *Chứng minh vế phải √4 2t + √t ≤ √6

⇔ 4 – t + 2√4t 2t ≤ 6 ⇔ 2√4t 2t ≤ t +2 ⇔ 16t – 8t2 ≤ t

2 + 4t + 4 ⇔ 9t

2 -

12t + 4 ≥

⇔ (3t – 2)2 ≥ 0 ( đúng) (1)

* Chứng minh vế trái: 2 ≤ √4 2t + √t ⇔ 4 ≤ 4 – t + 2√4t 2t

⇔t ≤ 2√4t 2t ⇔ t2 ≤ 16t – 8t

2 ⇔ t(16 – 9t) ≥ 0 (đúng do 0 ≤ t ≤ 1) (2)

Từ (1) và (2) bài toán được chứng minh.

123

Bài 7: (Sở giáo dục và đào tạo Hà Nội)

Cho x, y là các số thực dương thỏa mãn điều kiện: x ≥ 2y, tìm giá trị nhỏ nhất

của biểu thức: A =

Bài làm:

Ta có: x ≥ 2y ⇒

≥ 2

A =

=

.

/

. Đặt t =

(t ≥ 2).

Bài toán trở thành tìm giá trị nhỏ nhất của: A = f(t) =

với t ≥ 2

(Cũng như các bài trước ta dự đoán giá trị nhỏ nhất của A là

khi t =2, biến đổi

tương đương theo nhân tử x - 2)

Ta chứng minh: A ≥ f(2) ⇔

(*)

Thật vậy: (*) ⇔ 2t2 + 2 ≥ 5t ⇔ 2t

2 – 5t +2 ≥ 0

⇔ (2t -1)(t - 2 ) ≥ 0 ( đúng với mọi t ≥ 2)

Vậy Min (A) =

khi t =2 hay x = 2y.

Bài 8: (Chuyên Phan Bội Châu - Nghệ An)

Cho a, b, c là các số thực dương thay đổi thỏa mãn: a + b + c = 3.

Tìm giá trị nhỏ nhất của: A = a2 + b

2 + c

2 +

Bài làm:

Ta chứng minh: X = a2 + b

2 + c

2 ≥ a

2b + bc

2 + c

2a (1)

Thật vậy: 3(a2 + b

2 + c

2) = (a + b + c)( a

2 + b

2 + c

2)

= (a3 + b

3 + c

3) + (a

2b + b

2c + c

2a) + (ab

2 + bc

2 + ca

2)

Áp dụng Cô – si: a3 + ac

2 ≥ 2.√a c = 2a

2c

Tương tự: b3 + bc

2 ≥ 2b

2c ; c

3 + ca

2 ≥ 2c

2a

Nên: 3(a2 + b

2 + c

2) ≥ 3(a

2b + bc

2 + c

2a). Do đó (1) đúng.

A = a2 + b

2 + c

2 +

≥ a

2 + b

2 + c

2 +

⇒ A ≥ a2 + b

2 + c

2 +

( )

( ) = X +

X = a2 + b

2 + c

2 ≥

( )

= 3

Bài toán trở thành tìm Min của: f(X) = X +

với X ≥ 3

124

(Cũng như mấy lần trước dự đoán Min của f(X) đại 4 khi X = 3, Biến đổi

tương đương theo nhân tử X – 3)

Ta chứng Minh: f(X) ≥ f(2) ⇔ X +

≥ 4

Thật vậy: (*) ⇔

≥ 0 ⇔

( )( )

≥ 0 với mọi X ≥ 3

Vậy Min(A) = 5 khi X =3 hay a = b = c =1

Bài 9: (Chuyên KHTN – ĐHQG Hà Nội)

Với x, y là các số thực dương thỏa mãn: x + y ≤ 1.

Tìm giá trị nhỏ nhất của P = (

+

)√1 + x y

Bài làm:

P = (

+

)√1 + x y =

√1 + x y = √

( ) ( )

⇒ P = √(x + y) + ( )

≥ √(x + y) +

( )

[( )

] =

= √(x + y) +

( )

Đặt t = x + y ≤ 1 . Ta có: P ≥ f(t) = √t +

với 0 < t ≤ 1

( Cũng như các bài trước ta dự đoán dấu bằng tại điểm mút t = 1)

Ta sẽ chứng minh: f(t) ≥ f(1 ) ⇔ √t +

≥ √17 (*)

Thật vậy: (*) ⇔ t2 +

≥ 17 ⇔

≥ 0

⇔ ( )( )

( luôn đúng với 0 < t ≤ 1)

Qua các 9 ví dụ với lời giải cụ thể trên chắc chắn các bạn đã nắm vững được nội

dung Phương pháp giải toán mà tôi đã nêu. Sau đây là các bài tập cho các bạn

luyện tập.

1) (Chuyên Quang Trung – Bình Phước)

Cho a, b là hai số dương thỏa mãn: a + b ≥ 1. Tìm Min của

F = (a3 + b

3)

2 + a

2 + b

2 +

ab

2) (Chuyên Trần Phú – Hải Phòng)

Cho các số thực dương a, b, c thỏa mãn: a + b + c ≤ 3

125

Chứng minh rằng:

+

≥ 670

3) (Chuyên Lam Sơn – Thanh Hóa)

Biểu thức: P = a2 + b

2 + c

2 + d

2 + ac + bd, trong đó: ad – bc = 1

Chứng minh: P ≥ √3

4) (Chuyên tỉnh Hải Dương năm 2017)

Cho a, b, c là các số thực dương thay đổi thỏa mãn: a + b + c = 1. Tìm giá

trị nhỏ nhất của biểu thức:

Q = 14(a2 + b

2 + c

2) +

5) ( Chuyên Tỉnh Nam Định năm 2017)

Cho x, y, z là các số thực thỏa mãn: (x – y)(x – z) =1 và y ≠ z. Chứng

minh:

( ) +

( ) +

( ) ≥ 4.

6) (Chuyên tỉnh Ninh Thuân năm 2017)

Cho a, b, c thỏa mãn điều kiện: ab + bc +ca = 3. Tìm giá trị nhỏ nhất của

biểu thức:

P = a2 + b

2 + c

2 – 6(a+b+c) + 2017

.

Thí dụ 6. Cho phương trình ax2 + by + c = 0 (a ≠ 0) có hai nghiệm m, n thỏa

mãn 0 ≤ m ≤ n ≤ 1. Tìm giá trị lớn nhất của biểu thức P =

.

Lời giải. Từ giả thiết 0 ≤ m ≤ n ≤ 1 ⇒ m2 ≤ mn và n

2 ≤ 1. Suy ra: (m + n)

2 -2mn

≤ mn + 1 ⇒ (m + n)2 ≤ 3mn + 1. Theo địnhh lý Vi-et ta có:

m + n =

; mn =

. Do đó: P =

.

/

= ( ) ( )

( )

≤ ( )

( ) = 3.

Đẳng thức xảy ra khi 2 = n = 1

hoặc m = n = 1 tức là 2b = a c =

hoặc 2c = - b = 2a

Vậy MaxP = 3 khi 2b = a c =

hoặc 2c = - b = 2a

Nhận xét. Bài toán này chúng ta thường hay sử dụng BĐT cổ điển hay dùng

(BĐT cô-si, BĐT bunhiacopxki, BĐT tam giác, ….) hoặc các tính chất của BĐT

cùng với việc vận dụng định lý Vi- et một cách nhuần nhuyễn sẽ giúp chúng ta

tìm ra được lời giải bài toán ngắn gọn đọc đáo thú vị.

126

BẤT ĐẲNG THỨC AM – GM (CÔ – SI)

VỚI CÁC SỐ TỰ NHIÊN

ĐÀO HUY TRƯỜNG (GV.THCS Lập Thành, Vĩnh Phúc)

Trong các đề thi học sinh giỏi lớp 9 và thi vào lớp 10 THPT chuyên bài

BĐT hay cực trị thường là bài toán khó. Để giải bài toán này, ta thường sử dụng

BĐT Cô – si

Bất đẳng thức AM – GM ( Cô – si): Với các số thực x1, x2, x3,…, xn ta có:

≥ √

Ngoài sử dụng AM – GM với các biến số là các số thực không âm, ta còn gặp

các bài toán biến số là các số tự nhiên. Bài viết này giúp các bạn cũng cố kiến

thức về BĐT AM – GM với số tự nhiên.

Bài toán 1. Cho x, y, z là các số tự nhiên thỏa mãn x + y + z = 2010. Tìm giá trị

lướn nhất của biểu thức P = xyz.

Lời giải. Áp dụng BĐT AM – GM cho các số thực không âm ta có:

P = xyz ≤ .

/ = 670

3. Đẳng thức xảy ra khi x = y = z = 670.

Vậy MaxP = 6703 khi x = y = z = 670.

Bài toán 2. Cho các số tự nhiên x, y, z thỏa mãn x + y + z = 2008. Tìm giá trị

lớn nhất của biểu thức P = xyz

Bài làm. Nhận xét không thể sử dụng BĐT AM – GM như bài toán 1 vì khi đó x

= y = z =

không là số tự nhiên.

Câu hỏi đặt ra: Phải chăng không thể áp dụng BĐT AM – GM cho 3 số thực

không âm vào bài toán này? Nếu áp dụng được BĐT AM – GM thì áp dụng như

thế nào? Để trả lười được câu hỏi này mời các bạn theo dõi cách làm sau:

Bằng suy luận ta thấy P đạt giá trị lớn nhất khi các giá trị x, y, z gần bằng nhau

(bằng nhau hoặc hơn kém nhau một đơn vị). Ta có lời giải sau:

Bài làm. Do vài trò bình đẳng của x, y, z ta có thể giải sử x ≥ y ≥ z. Vì x, y, z

nên không thể xảy ra khả năng x = y = z. Do đó: x ≥ [

- + 1 = 670

127

Suy ra: P = xyz ≤ x.(

)

2= x..

/ (do áp dụng BĐT AM – GM với hai số

thực không âm)

=

.(

). .

/ .

/ ≤

.{

[(

) + .

/ + .

/]}

3

=

.[

.(2008 -

)]

3 ≤

.[

.(2008 -

)]

3 = 670.669

2

Đẳng thức xảy ra khi:

{x + y + z = 2 8

x = 67 y = z

⇔ x = 670 và y = z = 669

Vậy MaxP = 670.6692 Đạt được khi (x, y, z) =(670, 669, 669) và các hoán vị

Bài toán 3. Cho x, y, z là các số tự nhiên thỏa mãn x + y + z = 2009. Tìm giá trị

lớn nhất của P = xyz

Nhận xét. Bằng suy luận tương tự bài toán hai ta có lời giải bài toán như sau.

Bài làm. Do vai trò bình đẳng của x, y, z ta có thể giả sử x ≥ y ≥ z. Vì x + y + z

= 2009 nên

z ≤ [

] = 669. Suy ra P = xyz ≤ (

)

2.z = (

)

2.z =

. (

).

(

).

[

.(

+

+

)]

3 =

[

(2009 +

)]

3 ≤

[

(2009 +

)]

3=

669.6702

Đẳng thức xảy ra khi z = 669 và x = y = 670

Vậy MaxP = 669.6702 khi z = 669 và x = y = 670

Nhận xét. Bằng cách tương tự các bạn giải bài toán sau.

Bài toán. Cho số nguyên dương n. Giả sử x, y, z là các số tự nhiên thay đổi

thỏa mãn x + y + z = n. Tìm giá trị lớn nhất của biểu thức

P = xyz

XUNG QUANH BÀI BẤT ĐẲNG THỨC

THI HỌC SINH GIỎI QUỐC GIA NĂM 2 8

KIỀU ĐÌNH MINH

(GV.THCS chuyên Hùng Vương, Phú Thọ)

Trong kì thi học sinh giỏi Quốc gia năm 2008 có bài toán BĐT sau

Bài toán 1. Cho x, y, z là số thực không âm, đôi một khác nhau. Chứng minh

128

rằng:

( ) +

( ) +

( ) ≥

Bài toán trên là một BĐT hay và khó và chỉ cần dùng kiến thức THCS để giải.

Trãi qua thời gian, nhiều bài toán tương tự lấy ý tưởng từ bài toán trên vẫn xuất

hiện trong các kì thi học sinh giỏi. Trong quá trình giảng dạy chúng tôi đã bắt

gặp những bài toán liên quan đến bài toán này và chân trọng giới thiệu cùng bạn

đọc. Nhưng trước tiên chúng ta hãy nói đến phuong pháp tham số hóa để giải bài

toán trên.

Lời giải. Vì vai trò x, y, z như nhau nên không mất tính tổng giả sử

x > y > z ≥ 0. Đặt x = a + z , y = b + z (a > b > 0). Bất đẳng thức cần chứng

minh tương đương với: (

+

+

( ) )[(a + z)(b + z) + (b + z)z + (a + z)z] ≥ 4

⇔ (

+

+

( ) )(3z

2 + 2z(a + b) + ab) ≥ 4

Ta có: (

+

+

( ) )(3z

2 + 2z(a + b) + ab) ≥ ab(

+

+

( ) )

=

+

+

( ) =

( )

+

( ) + 2 ≥ 2.√

( )

( ) + 2 = 4

Đẳng thức xảy ra khi:

{z =

ab = (a b) ⇔ {z =

x = √

y

Bài 2. Cho x, y, z là các số thực đôi một khác nhau. Chứng minh rằng :

(x2 + y

2 + z

2 – xy – yz – zx)(

( ) +

( ) +

( ) ) ≥

.

(Đề thi chọn đội tuyển Phú Thọ năm 2015)

Lời giải. Đặt x = y + a , y = z + b ( a , b > 0) thì bất đẳng thức đã cho trở thành:

(a + b)2(

+

) +

( ) +

+

+ 3 ≥

Ta có: (a + b)2(

+

) ≥ (a + b)

2.

(

+

)

2 ≥ (a + b)

2.8.

( ) = 8;

( ) ≥

;

+

≥ 2.

Suy ra: (a + b)2(

+

) +

( ) +

+

+ 3 ≥ 8 +

+ 2 + 3 =

(Đpcm)

Đẳng thức xảy ra khi: a = b ⇔ z + x = 2y.

Bài tập rèn luyện.

Bài 1. Cho x, y, z là các số thực đôi một khác nhau. Chứng minh rằng :

( ) +

( ) +

( ) ≥

129

Bài 2. Cho x, y, z là các số thực đôi một khác nhau. Chứng minh rằng :

(x + y + z )(

( ) +

( ) +

( ) ) ≥

.

Bài 3. Cho x, y, z là các số thực đôi một khác nhau. Tìm giá trị nhỏ nhất của

biểu thức :

A = (x + y + z)(

+

+

) .

Bài 4. Cho x, y, z là các số thực đôi một khác nhau. Chứng minh rằng :

(

)

2 + (

)

2 + (

)

2 ≥ 1.

TÌM CỰC TRỊ CÓ ĐIỀU KIỆN

THÁI HỮU HUỆ (GV. THCS Quang Lộc, Can Lộc, Hà Tĩnh)

Có rất nhiều phương pháp giải một bài toán bất đẳng thức (BĐT), nhưng

chúng ta cần tìm phương pháp giải ngắn gọn nhất. Đặc biệt chúng ta cần tạo kĩ

năng chứng minh BĐT. Khi tìm hiểu về BĐT, chúng tôi nhận thấy có một số

BĐT có số biến xác định trong các đoạn thì cần xem xét kĩ giá trị các biến số đó

để lập ra một BĐT đúng, rồi từ BĐT này, ta biến đổi thành BĐT cần chứng

minh.

Bài toán 1. Cho các số thực x, y, z [-1; 2] thỏa mãn điều kiện x + y + z = 0

Chứng minh rằng: x2 + y

2 + z

2 ≤ 6.

Giải. Vì x [-1; 2] nên (x + 1)(x – 2) ≤ 0 ⇔ x2 ≤ x + 2.

Tương tự: y2 ≤ y + 2 ; z

2 ≤ z + 2. Cộng theo vế ba BĐT trên ta được:

x2 + y

2 + z

2 ≤ (x + y + z) + 6 = 6

Đẳng thức xảy ra khi và chỉ khi x + y + z = 0 và x, y, z {-1; 2}

⇔ 3 số x, y, z có 2 số bằng – 1 và 1 số còn lại bằng 2.

Bài toán 2. Cho các số thực x, y, z [0; 2] thỏa mãn điều kiện x + y + z = 3.

Chứng minh rằng: x2 + y

2 + z

2 ≤ 5.

Giải. Vì x, y, z [0; 2] nên (x – 2)(y – 2)(z – 2) ≤ 0

⇔ xyz – 2(xy + yz + zx) + 4(x + y + z) – 8 ≤ 0

⇔ xyz + (x2 + y

2 + z

2) – (x + y + z)

2 + 4(x + y + z) – 8 ≤ 0 (1)

Mà x + y + z = 3 (giả thiết) ⇒ (1) ⇔ xyz + (x2 + y

2 + z

2) – 9 + 12 – 8 ≤ 0

⇔ x2 + y

2 + z

2 ≤ 5 – xyz

Do x, y , z ≥ 0 (vì x, y, z [0; 2]) ⇒ xyz ≥ 0 ⇒ - xyz ≤ 0

Suy ra: x2 + y

2 + z

2 ≤ 5 (Đpcm)

130

Dấu “=” xảy ra khi: (x – 2)(y – 2)(z – 2) = 0, xyz = 0 và x + y + z = 3

⇔ (x, y, z) = (2; 1; 0) và các hoán vị.

Bài toán 3. Cho các số thực a, b, c [-2; 5] thỏa mãn điều kiện a + 2b + 3b ≤ 2

Chứng minh rằng: a + 2b + 3c ≤ 2. Chứng minh rằng: a2 + 2b

2 + 3c

2 ≤ 66.

(Đề thi tuyển sinh lớp 10, năm học 2009 – 2010 Sở giáo dục Hà Tĩnh)

Giải. Tương tự bài toán 1, từ (a + 2)(a – 5) ≤ 0 ta suy ra: a2 ≤ 3a + 10 cùng các kết

quả tương tự b2 ≤ 3b + 10 , c

2 ≤ 3c + 10.

Suy ra: a2 + b

2 + c

2 ≤ 3(a + 2b + 3c) + 60 = 66 (Đpcm)

Đẳng thức xảy ra khi và chỉ khi a= -2; b = - 5; c = - 2.

Bài toán 4. Cho các số thực a, b, c [0 ; 1].

Chứng minh rằng a2 + b

2 + c

2 ≤ 1 + a

2 b + b

2c + c

2a

Giải. Vì a, b, c [0 ; 1] nên a(1 – b) ≥ a2(1 – b).

Tương tự: b(1 – c) ≥ b2(1 – c) ; c(1 – a) ≥ c

2(1 – a).

Cộng theo vế 3 BĐT trên ta suy ra:

(a + b + c) – (ab + bc + ca) ≥ (a2 + b

2 + c

2) – (a

2 b + bc

2 + c

2a)

⇔ 1 – (1 – a)(1 – b)(1 – c) – abc ≥ (a2 + b

2 + c

2) – (a

2 b + bc

2 + c

2a)

Do (1 – a)(1 – b)(1 – c) ≥ 0 và abc ≥ 0 nên suy ra đpcm.

Bài toán 5. Cho 2010 số thực a1, a2, a3,…, a2010 [0 ; 1].

Chứng minh rằng: (1 + a1 + a2 + …+ a2010)2 ≥ 4(

+ + … +

)2

Giải. Với số thực x, y bất kì ta có: (x – y)2 ≥ 0 ⇔ (x + y)

2 ≥ 4xy.

Áp dụng với x = 1, y = a1 + a2 + …+ a2010 ta có:

(1 + a1 + a2 + …+ a2010)2 ≥ 4(a1 + a2 +…+ a2010)

2

Mà với mỗi i {1; 2;….; 2010} ta có: ≥ (vì theo giả thiết [0 ; 1]).

Từ đó suy ra đpcm.

Đẳng thức xảy ra khi trong 2010 số đã cho có 2009 số bằng 0 và số còn lại

bằng 1.

Bài tập luyện tập.

Bài tập 1. Cho các số thực a, b, c [0 ; 1]. Chứng minh rằng:

a + 2b + 3c ≤ 60; 2(a3 + b

3 + c

3) ≤ 3 + a

2 b + b

2c + c

2a.

Bài 2. Cho các số thực a, b, c [0 ; 1]. Chứng minh rằng:

+

+

≤ 2.

131

Bài 3. Cho

với a1, a2…, a2010 và b1, b2…, b2010 là các số thực dương.

Chứng minh rằng:

TỪ NHỮNG ĐẲNG THỨC ĐẸP

ĐINH NGỌC THUẦN (GV. THPT Thái Phiên, Hải Phòng)

NGUYỄN ĐÌNH LỘC (GV. HS 9C. THCS Vĩnh Tường, Vĩnh Phúc)

Trong quá trình giải toán, hẳn các bạn đã gặp những đẳng thức hay, là

chìa khóa để giải các bài toán khác. Bài viết này tôi muốn chia sẽ với các bạn

một số đẳng thức như vậy.

Đẳng thức 1. Cho 3 số x, y, z thỏa mãn xyz = 1 Chứng minh rằng:

A =

+

+

= 1. (1)

(Đề thi vào lớp 10. PTNK, ĐHQG Hồ Chí Minh năm học 1997-1998)

Chứng minh. Vì xyz = 1 nên:

=

=

=

=

. từ đó: A =

= 1 ta có đpcm.

Nhận xét. Từ (1) ta thu được đẳng thức sau

+

+

= 1 (2)

với x, y, z là ba số thực thỏa mãn xyz = 1.

Ví dụ 1. Cho ba số thực dương x, y, z thỏa mãn xyz = 1. Chứng minh rằng:

2B ≤ 1, với B =

+

+

Lời giải. Vì (x – y)2 + (x – 1)

2 ≥ 0 nên x

2 – 2xy + y

2 + x

2 – 2x + 1 ≥ 0

⇔ 2x2 + y

2 + 3 ≥ 2(xy + x + 1). Do đó:

( )

Tương tự:

( ) ;

( )

Cộng theo vế ba BĐT trên ta được:

B ≤

(

+

+

) =

(theo (1)). Từ đó suy ra đpcm.

Đẳng thức xảy ra khi x = y = z = 1

Ví dụ 2. Cho ba số thực dương x, y, z thỏa mãn xyz = 1. Tìm giá trị lớn nhất của

biểu thức: C =

( ) +

( ) +

( )

132

Lời giải. Ta có: (x + 1)2 + y

2 + 1 = x

2 + 2x + 1 + y

2 + 1 = (x

2 – 2xy + y

2) + 2(xy +

x + 1) = (x – y)2 + 2(xy + x + 1) ≥ 2(xy + x + 1). Suy ra:

( ) ≤

( ).

Tương tự:

( ) ≤

( ) ;

( ) ≤

( ).

Cộng theo vế ba BĐT trên ta được:

C =

( ) +

( ) +

( )

(

+

+

) =

(theo (1)). Từ đó suy ra đpcm

Đẳng thức xảy ra khi x = y = z = 1.

Ví dụ 3. Cho 3 số thực dương a, b, c thỏa mãn abc = 1. Chứng minh rằng:

D ≥ 1 với D =

+

+

.

Lời giải. Vì abc = 1 nên

=

=

(do áp dụng BĐT Cô – si với hai số dương b2 và b

2c

2)

Tương tự:

;

Cộng ba BĐT trên theo vế ta được:

D =

+

+

+

+

= 1

(theo (2)). Từ đó suy ra đpcm.

Đẳng thức xảy ra khi x = y = z = 1

Nhận xét. Qua các ví dụ trên, chúng ta thấy điểm mấu chốt để giải được các bài

toán ở trên là nhận ra các đẳng thức (1) và (2)

Đẳng thức 2. Cho 3 số thực dương x, y, z thỏa mãn xyz = 2. Chứng minh rằng:

M =

+

+

= 1 (3)

(Đề thi vào lớp 10, PTNK,ĐHQG HCM;Năm học 2009-2010)

Lời giải. Vì xyz = 2 nên:

=

=

;

=

=

Cộng theo vế các BĐT trên ta được:

M =

+

+

=

+

+

= 1 (đpcm)

Ví dụ 4. Cho ba số thực dượng x, y, z thỏa mãn xyz = 2. Chứng minh rằng:

+

+

Lời giải. Tương tự ví dụ 1 ta có:

( );

133

( ) =

( ) ;

( ) =

.

Cộng theo vế BĐT trên và kết hợp với (3), ta suy ra đpcm.

Đẳng thức xảy ra khi và chỉ khi x = y = 1; z = 2.

Đẳng thức 3. Cho a, b, c là các số thực không đồng thời bằng 0. Đặt

M =

+

+

; N =

+

+

Thế thì M = N

Chứng minh. Ta có: M – N =

+

+

= (a – b) + (b – c) + (c – a) = 0, suy ra đpcm.

Bài toán 1. Cho a, b, c là các số thực dương. Chứng minh rằng:

M =

+

+

Đẳng thức xảy ra khi nào?

Lời giải. Vì M = N nên 2M = M + N =

+

+

= ( )( )

+

( )( )

+

( )( )

Với x, y là số thực dương bất kì, ta chứng minh:

(1)

Thật vậy, ta có: (1) ⇔ 3x2 – 3xy + 3y

2 ≥ x

2 + xy + y

2 ⇔ 2(x – y)

2 ≥ 0 (đúng)

Đẳng thức xảy ra ở (1) khi và chỉ khi x = y. Áp dụng (1) ta suy ra:

2M ≥

+

+

⇔ M ≥

(Đpcm)

Đẳng thức xảy ra khi a = b = c.

Nhận xét. 1) Ta viết 2M = M + N để tạo ra biểu thức đối xứng với a, b, c.

2) Phân thức:

có tử thức và mẫu thức đều là các đa thức đối xứng với

các biến x, y. Hơn nữa các đa thức này có cùng bậc 2 với mỗi đa thức là tổng

các đơn thức cùng bậc.

Những điều đó cho ta dự đoán sự so sánh phân thức này với một số thực.

Hơn nữa khi x = y thì

=

. Điều đó cho ta dự đoán

Hoặc

. Biến đổi tương đương một trong 2 đẳng thức ta thấy (1)

đúng.

Đẳng thức 4. Cho a, b, c là các số thực đôi một khác nhau. Đặt:

134

P =

( )( ) +

( )( ) +

( )( ). Thế thì P = 0.

Chứng minh. Ta có: P = ( ) ( ) ( )

( )( )( ) = 0, suy ra đpcm.

Bài toán 2. Cho a, b, c là các số thực không âm và đôi một khác nhau. Chứng

minh rằng:

( ) +

( ) +

( ) ≥

. (2)

Đẳng thức xảy ra khi nào ?

Lời giải. Đặt vế trái của (2) là Q. Vì P = 0 nên

Q = 2P + Q = (

+

+

)

2

Do vai trò bình đẳng của a, b, c trong biểu thức nên ta có thể đặt a = min(a, b, c)

Với x, y là các số thực bất kì ta có: (x – y)2 ≥ 0 ⇔ (x + y)

2 ≥ 4xy.

Đẳng thức xảy ra khi và chỉ khi x = y.

Áp dụng ta có: Q = (

+

+

)

2 ≥

.(

+

) =

( )( )

. Từ đó suy ra đpcm

Dấu bằng xảy ra khi 1 trong trong 3 số bằng 0. Hai số còn, chẳng hạn b, c thỏa

mãn

=

+

⇔ b

2 – 3bc + c

2 = 0 ⇔ b =

.c

Đẳng thức 5. Cho a, b, c là các số thực đôi một khác nhau. Đặt

R =

( )( ) +

( )( ) +

( )( ) thì R = - 1

Chứng minh. Ta có: R = ( ) ( ) ( )

( )( )( )

Vì ab(a – b) + bc(b – c) + ca(c – a) = ab(a – b) + bc(b – c) – ca(a – b) – ca(b – c) =

a(a – b)(b – c) + c(b – c)(b – a) = - (a – b)(b – c)(c – a) nên R = -1 suy ra đpcm

Bài toán 3. Cho a, b, c là các số thực đôi một khác nhau. Chứng minh rằng:

( ) +

( ) +

( ) ≥ 2. (3)

Lời giải. Vì R = - 1. Nên (3) ⇔

( ) +

( ) +

( ) ≥

-2[

( )( ) +

( )( ) +

( )( )]

⇔ (

+

+

)

2 ≥ 0 (đúng)

Nhận xét. Bạn đọc thử tìm điều kiện để xảy ra đẳng thức ở (3).

Cuối cùng bạn đọc hãy thử chứng minh một số đẳng thức và thửu tìm ứng dụng

của các đẳng thức này.

135

1) ab + bc + ca + ad = (a + c)(b + d).

2) (a + b)(b + c)(c + a) = (a + b + c)(ab + bc + ca) – abc .

3) a3 + b

3 + c

3 - 3abc = (a + b + c)(a

2 + b

2 + c

2 - ab – bc – ca)

4) (a2 + b

2 + c

2)

2 – 2(a

4 + b

4 + c

4) = (a + b + c)(a + b – c)(a + c - b)(b + c – a).

5)

( )( ) +

( )( ) +

( )( ) = 0.

6)

( )( ) +

( )( ) +

( )( ) = 1.

Chúc các bạn tìm thêm được nhiều đẳng thức khác nữa cùng ứng dụng của nó.

BẤT ĐẲNG THỨC VÀ TIM GIÁ TRỊ NHỎ RẤT LỚN NHẤT

ĐẶNG THÀNH NAM

I) NỘI DUNG KIẾN THỨC

1. Bất đẳng thức Cô – si.

Cho 2 số không âm ta có:

≥ √

Dấu “=” xảy ra khi a = b

Cho 3 số không âm ta có:

≥ √

Dấu “=” xảy ra khi a = b = c

2. Bất đẳng thức BunhiaCopsky

Cho x, y, a, b là các số thực ta luôn có: (x2 + y

2)(a

2 + b

2) ≥ (ax + by)

2

Dấu “=” xảy ra khi { = =

( k R)

Một số biến dạng của hai BĐT trên.

Cho a, b dương ta luôn có: (a + b)(

+

) ≥ 4 ⇒

+

Chứng minh: Theo BĐT Cô – si: { + ≥ 2√

+

⇒(a + b)(

+

) ≥ 4

Mở rộng tương tự cho 3 số a, b, c dương ta luôn có:

+

+

Cho x, y, a, b là các số thực ta luôn có: (

+

)(x

2 + y

2) ≥ (a + b)

2

+

( )

.

Đặt u = x2 , v = y

2 ta có BĐT:

+

( )

Mở rộng BĐT cho 3 số:

+

+

( )

136

II) BÀI TẬP VẬN DỤNG.

Ví dụ 1. Cho x, y > 0 thỏa mãn x2 + y

2 = 2. Chứng minh:

+

≥ 4

Lời giải. Ta có: (

+

)( + + 2 ) ≥ (

+ 3y)

2

Mặt khác:

+ 3y =

+ 3y =

( )

≥ 4

Ta có: x + x + 2y = 2(x + y) ≤ 2√2( + ) = 4

+

(

)

( ) ≥

= 4. Dấu “=” xảy ra khi x = y = 1.

Cách 2: BĐT ⇔ x3(x + 2y) + 9y

4 ≥ 4y

2(x + 2y)

Cách 3: 2y ≤ y2 + 1 = 3 – x

2

+

+

=

+

( )

≥ 4, ∀ ( √2)

Ví dụ 2. (*) Bất đẳng thức phụ:

+

( ) , ∀ ,

Áp dụng: Cho x, y, z ≥ 0 thỏa mãn x2 + y

2 + z

2 ≤ 3y. Tìm GTNN của biểu thức:

P =

( ) +

( ) +

( )

Lời giải. C/m BĐT phụ, ta có:

+

. Mặt khác: ab ≤

( )

+

( )

=

( ) (đpcm). Dấu “=” xảy ra khi a = b

P =

( ) +

.

/

+

( ) ≥

.

/

+

( ) ≥

.

/

Ta có: (x2 + 1) + (y

2 + 4) + (z

2 + 1) ≥ 2x + 4y + 2z ⇒ 2x + 4y + 2z ≤ 3y + 6

⇔ 2x + y + 2z ≤ 6 ⇔ x +

+ z ≤ 3. Vậy P ≥

( ) = 1

Vậy MinP = 1 khi x = 1, y = 2, z = 1.

Cách 2: Ta có: (x + 1)2 ≤ 2(x

2 + 1); (z + 3)

2 ≤ 4(z

2 + 3)

P ≥

( ) +

( ) +

=

( ) +

+

+

( )

( ) +

( ) ≥

( ) +

( ) (bđtd)

Ví dụ: Bài toán phụ , cho x, y thỏa mãn xy ≥ 1. Ta có:

+

. Với – 1< xy ≤ 1 bất đẳng thức đổi chiều.

Lời giải. BĐT⇔

( )( ) ≥

137

⇔ (2 + x2 + y

2)(1 + xy) ≥ 2(1 + x

2)(1 + y

2)

⇔ x2 + y

2 + xy(x

2 + y

2) + 2xy + 2 ≥ 2 + 2(x

2 + y

2) + 2x

2y

2

⇔ (x – y)2(xy – 1) ≥ 0 (luôn đúng ∀ ≥ 1)

Chứng minh BĐT đổi chiều làm tương tự.

Ví dụ. Cho a, b, c ≥ 1. Chứng minh:

+

+

.

Lời giải. Ta có:

+

√ ;

+

(Trong đó d là số thực bất kì và d ≥ 1)

VT +

≥ 2(

√ +

√ ) ≥

√ .

Chọn d3 = √

⇔ d = 3 ⇒ VT +

⇔ VT ≥

(đpcm)

Áp dụng. Cho x, y, z > 2, thỏa mãn:

+

+

= 1

Chứng minh: (x – 1)(y – 1)(z – 1) ≥ 8.

Lời giải. Ta có:

( ) +

( ) +

( ) ≥

√( )( )( )

⇔ 1 ≥

√( )( )( ) ⇔ (x – 1)(y – 1)(z – 1) ≥ 8

Ví dụ: Cho x, y > 0 thỏa mãn 2(x2 + y

2) +

= 5

Tìm GTLN của P =

+

-

Lời giải. 5 = 2(x2 + y

2) +

+ 4xy ⇔ 4x

2y

2 – 5xy + 1 ≤ 0

⇔ (xy – 1)(4xy – 1) ≤ 0 ⇔

≤ xy ≤ 1.

Áp dụng: P ≤ f(t) =

-

.

Đặt t = xy, t [

; 1] ⇒ P =

-

f‟(t) =

( ) +

( ) =

( ) ( ) < 0 ∀ t [

; 1]

Vậy MaxP = f(

)

Ví dụ 1. Cho x, y, z [

; 3]. Tìm GTNN của biểu thức:

P =

+

+

Lời giải. Không mất tính tổng quát, giả sử: z = max{x, y, z}.

138

Ta có: P =

+

+

+

Đặt t = √

(1 ≤ t ≤ 3). P ≥ f(t) =

+

=

+

P ≥ f(t) ≥ f(3) =

.

DÙNG THAM SỐ TRONG BÀI TOÁN CỰC TRỊ

HỒ KHÁNH DUY (HS. 11A1, THPT Quỳnh Lưu 1, Nghệ An)

Khi giải bài toán cực trị của biểu thức, ta cần tìm được điểm rơi phù hợp

để xảy ra đẳng thức. Sử dụng tham số, kết hợp giải thiết để xét dấu đẳng thức ,

ta có thể có lời giải của dạng toán này.

Bài toán 1. Cho x, y ≥ 0 thỏa mãn x + y ≤ 1. Tìm giá trị lớn nhất của biểu thức:

P = √ + 2√ .

Lời giải. Ta sử dụng tham số phụ a, b ≥ 0.

Áp dụng BĐT Cô – si cho hai số không âm ta có:

a + x ≥ 2√ ; b + y ≥ 2√ Suy ra: a + b + x + y ≥ 2(√ + √ )

Các tham số a, b cần thỏa mãn √ = 2√ (để tạo thành P) và a = x , b = y

⇒ a + b = x + y = 1 (để xảy ra đẳng thức trong giả thiết)

Từ đó b = 4a và 5a = 1 ⇒ a =

⇒ b =

Vậy Pmax = √5 tại x =

, y =

Bài toán 2. Tìm giá trị nhỏ nhất của biểu thức: Q =

√ - √ ( 1), với x ≥ 1

Lời giải. Xét a > 0. Áp dụng BĐT Cô – si cho 2 số dương ta có:

√ ( 1) =

√ . 2√ ( 1) ≤

√ . (ax + x – 1)

Đẳng thức xảy ra khi và chỉ khi ax = x – 1 ⇔ x =

(với a < 1)

Điều kiện a ≥ 1 ⇔

≥ 1⇔ a ≥ 0. Vậy 0 ≤ a < 1.

Từ đó Q ≥ (

√ -

√ ).x +

√ . Ta cần tìm a thỏa mãn

√ -

√ = 0

⇔ 7√ = 2√3.(a + 1) ⇔ 49a = 12(a + 1)2 ⇔ 12a

2 – 25a + 12 = 0

⇔ a =

hoặc a =

. Do 0 ≤ a < 1 nên a =

⇒ x = 4

Vậy Qmin =

√ =

tại x = 4

Bài toán 3. Tìm giá trị lớn nhất của biểu thức:

139

R = 9x√ + 1 + 13x√1 với 0 ≤ x ≤ 1.

Lời giải. Xét a, b > 0. Viết biểu thức dưới dạng:

R =

√ .2√ ( + 1) +

√ 2√ (1 )

Áp dụng BĐT Cô – si cho 2 số dương ta có:

2√ ( + 1) ≤ ax2 + x

2 + 1 ; 2√ (1 ) ≤ bx

2 + 1 – x

2

Đẳng thức xảy ra khi ax2 = x

2 + 1 và bx

2 = 1 – x

2 Suy ra x

2 =

=

(a > 1)

Do đó b = a – 2 (a > 2). Mặt khác: R ≤

√ (ax

2 + x

2 + 1) +

√ (bx

2 + 1 – x

2)

= ( ( )

√ +

( )

√ ).x

2 +

√ +

√ .

Xét a, b thỏa mãn ( )

√ +

( )

√ = 0 ⇔

( )

√ +

( )

√ = 0 (do b = a – 2)

⇔ 9(a + 1)√ 2 = 13(3 - a)√ ⇔ 81(a + 1)2(a – 2) = 169(3 – a)

2a (a ≤ 3)

⇔ 44a3 – 507a

2 + 882a + 81 = 0 ⇔ (4a – 9)(11a

2 – 102a – 9) = 0

vì 2 < a ≤ 3 nên chọn a =

⇒ b =

; x =

√ . Vậy Rmax = 16 tại x =

Bài toán 4. Tìm giá trị lớn nhất cảu biểu thức:

S = √

với x > 4.

Lời giải. Xét a > 0. Ta có:6 √ 2 + 2 = 6√( + 1)( 2)

=

√ . 2√( + 1)( 2) ≤

√ .(x

2 + 1 + ax – 2a)

(theo BĐT Cô – si cho 2 số dương).

Đẳng thức xảy ra khi x2 + 1 = ax – 2a ⇔ x

2 – ax + 2a + 1 = 0 (1)

Khi đó: S ≤

√ ( – )

=

.

√ / √

Tìm a thỏa mãn: .2 +

√ / : 3√ (

√ + 5) = 1 : 3 : (- 4).

Từ: .2 +

√ / : 3√ = 1 : 3 Suy ra: 2 +

√ = √ ⇔ (√ )

2 - 2√ – 3 = 0

⇔ √ = 3 ⇔ a = 9 : thỏa mãn (2). Khi đó: (1) ⇔ x2 – 9x + 19 = 0 ⇔ x =

Do điều kiện a > 4 nên x = √

. Với a = 9 thì S ≤ 3.

S = 3 ⇔ x = √

. Vậy Smax = 3 khi x =

140

Bài toán 5. Cho x, y là các số thực thỏa mãn x2 + y

2 = 1. Tim GTLN và GTNN

của biểu thức: T = ( )

Lời giải. Ta có: x2 + y

2 = 1 nên 1 + 2xy + 2y

2 = x

2 + 2xy + 3y

2

= (x + y)2 + 2y

2 > 0 (do (x ; y) (0 ; 0).

Khi y = 0 thì x2 = 1 và T = 2.

Xét y ≠ 0. Đặt t =

. Với a R, ta có: T – a =

( )

– a =

( ) ( )

ta tìm a ≠ 2 để tử số là một bình phương. Tức là =

⇔ (6 – a)2 + 3a(2 – a) = 0 ⇔ a

2 + 3a – 18 = 0 ⇔ a = 3 hoặc a = - 6

Với a = 3 ta có: T – 3 =

=

( )

≤ 0.

Suy ra T ≤ 3. Khi T = 3 thì t = 3 ⇔ {

= 3

+ = 1 ⇔ 10y

2 = 1 ⇔ y

2 =

Có hai cặp số (x; y) thỏa mãn là: (

√ ;

√ ) ; (

√ ;

√ )

Với t = -6 ta có: t = -

⇔ {

=

+ = 1 ⇔ y

2 =

Có hai cặp số (x ; y) thỏa mãn là: (

√ ;

√ ) , (

√ ;

√ ).

Vậy Tmin = - 6 và Tmax = 3.

Bài tập luyện tập.

Bài 1. Cho x, y, z ≥ 0 thỏa mãn ab + bc + ca = 1. Tìm giá trị nhỏ nhất của biểu

thức: A = 2x2 + y

2 + z

2.

Bài 2. Tìm giá trị nhỏ nhất của biểu thức: B = x(2010 + √2 11 ).

Bài 3. Tìm giá trị nhỏ nhất của biểu thức: C = √ + 4√1

, với x ≤ 2.

MỘT SỐ CÁCH ĐỔI BIẾN

TRẦN BÁ DUY LINH (Đại học kinh tế TP. Hồ Chí Minh)

141

Từ những điều kiện ban đầu của bài toán, bằng một số phép biến đổi ta có

thể làm cho bài toán cần giải quyết trở nên đơn giải hơn. Bài viết này sẽ điểm

lại một số phép đổi biến từ quen đến lạ.

Dạng 1. Khi điều kiện bài toán là x1, x2…., xn và x1.x2…xn = 1 thì phép biến đổi

mà chúng ta hay liên tưởng tới là: Đặt x1 = .

/

; x2 = .

/ ;…; xn = .

/

Với a1, a2,…, an , > 0.

Ví dụ 1. Cho a, b, c là các số thực dương thỏa mãn abc = 1. Chứng minh rằng:

+

+

. (1)

Lời giải. Đặt a =

, b =

, c =

(với x, y, z > 0).

Biến đổi (1) ta được: (1) ⇔

+

+

(1‟)

Đặt u = xy + yz , v = yz + zx, t = zx + xy.

Khi đó: 2xy = v + t – u, 2yz = t + u – v, 2zx = u + v – t

Ta được: (1‟) ⇔

+

+

≥ 3.

⇔ (

+

) + (

+

) + (

+

) ≥ 6: đúng do BĐT Cô – si.

Ví dụ 2. Cho các số thực dương a, b, c thỏa mãn abcd = 1. CMR:

( ) +

( ) +

( ) +

( ) ≥ 2. (2)

Lời giải. Đặt a =

, b =

, c =

, d =

(với x, y, z, t > 0). Biến đổi ta được:

(2) ⇔

+

+

+

≥ 2 ⇔

+

+

+

≥ 2

Ta có: [(xy + yz) + (zx + tx) + (yz + zt) + (xt + yt)](

+

+

+

) ≥ (x + y + z + t)

2

+

+

+

( )

( ) ( ) ( ) ( )

Ta chứng minh: ( )

( ) ( ) ( ) ( ) ≥ 2 (a)

Thật vậy, biến đổi (a) thành: x2 + y

2 + z

2 + t

2 ≥ 2(yz + xt)

⇔ (x – t)2 + (y – z)

2 ≥ 0 (đúng)

Ví dụ 2. Cho ba số thực dương x, y, z thỏa mãn xyz = 1.CMR:

C =

( ) +

( ) +

( )

142

Lời giải. Ta có: (x + 1)2 + y

2 + 1 = x

2 + 2x + 1 + y

2 + 1 = (x

2 – 2xy + y

2) + 2(xy +

x + 1) = (x – y)2 + 2(xy + x + 1) ≥ 2(xy + x + 1).

Suy ra:

( ) ≤

( ).

Tương tự:

( ) ≤

( ) ;

( ) ≤

( ).

Vậy C ≤

( ) +

( ) +

( )

Đặt x =

, y =

, z =

(a, b, c > 0) Ta được:

( ) +

( ) +

( ) =

, Suy ra đpcm.

Ví dụ 4. Cho a, b, c là các số thực dương thỏa mãn abc = 1. CMR:

a(b2 - √ ) + b(c

2 - √ ) + c(a

2 - √ ) ≥ 0. (4)

Lời giải. Đặt a =

, b =

, c =

(với x, y, z > 0)

Biến đổi, nhân hai vế của BĐT với x2y

2z

2 ta được:

+

+

+

+

⇔ (

-

)

2 + (

-

)

2 + (

-

)

2 ≥ 0 (Đúng)

Dạng 2. Khi bài toán cho điều kiện x, y, z > 0 và x + y + z + 2 = xyz, ta biến đổi

thành:

+

+

= 1 rồi đặt a =

, b =

, c =

.

Khi đó: a + b + c = 1và x =

=

; y =

=

; z =

=

Ví dụ 5. Cho x, y, z là các số thực dương thỏa mãn xy + yz + xz + 2xyz = 1

Chứng minh rằng

+

+

≥ 4(x + y + z). (5)

Lời giải. Từ giả thiết suy ra:

+

+

+ 2 =

.

Đặt x =

, y =

, z =

, với a, b, c > 0 và a + b + c = 1. Khi đó (5) trở

thành:

+

+

≥ 4(

+

+

).

Vì (b + c)2 ≥ 4bc nên

+

Suy ra:

+

Viết hai BĐT tương tự rồi cộng theo vế ta được đpcm.

Ví dụ 6. Cho x, y, z là các số thực dương thỏa mãn x + y + z + 2 = xyz. CMR:

2(√ + √ + √ ) ≤ x + y + z + 6. (6)

Lời giải.Với chú ý: 2(√ + √ + √ ) ≤ (√ + √ + √ )2 – (x + y + z) (6)

Ta đưa (6) về dạng: √ + √ + √ ≤ √2( + + + 3)

143

⇔ √ + √ + √ ≤ √2,( + 1) + ( + 1) + ( + 1)-

Theo cách đặt ẩn phụ ở trên ta được BĐT tương đương là:

+ √

+ √

≤ √.

+

+

/ ,( + ) + ( + ) + ( + ): đúng

Ví dụ 7. Cho x, y, z là các số thực dương thỏa mãn xy + yz + zx + xyz = 4.

CMR: x + y + z ≥ xy + yz + zx . (7)

Lời giải. Biết đổi giả thiết trở thành:

.

+

.

+

.

+ 2.

.

.

= 1

Từ đó đặt x =

, y =

, z =

với a, b, c > 0 và a + b + c = 1. Khi đó

(7) trở thành:

+

+

( )( ) +

( )( ) +

( )( )

Biến đổi ta được: a(a – b)(a – c) + b(b – a)(b – c) + c(c – a)(c – a) ≥ 0

(Vì đây là BĐT Schur).

Bài tập tự luyện.

Bài 1. Cho n ≥ 3 và x1.x2…xn = 1.

CMR:

+

+…+

≥ 1.

Bài 2. Cho x, y, z là các số thực dương thỏa mãn: x + y + z + 2 = xyz. CMR:

a) xy + yz + zx ≥ 2(x + y + z).

b) √ + √ + √ ≤

.√ .

Bài 3. Cho x, y, z là các số thực dương thỏa mãn: xy + yz + zx + 2xyz = 1.

CMR: a) xyz ≤

b) xy + yz + xz ≥

.

Bài 4. Cho a, b, c là các số thực dương. CMR:

+

+

+

+

+

.

144

Bài 1

Lời giải.

Bài 2.

Lời giải.

145

Câu 3.

Lời giải.

146

Bài 10.

Lời giải.

Bài 11.

Lời giải.

147

Bài 12.

Lời giải.

Câu 13.

148

Lời giải.

Câu 14.

Lời giải.

149

Bài 15.

Lời giải.

Bài 16.

150

Lời giải.

Áp dụng BĐT Mincowski ta có:

Bài 17.

Lời giải.

Bài 18.

151

Lời giải.

Bài 19.

Lời giải.

Bài 20.

Lời giải.

Bài 21.

152

Lời giải.

Bài 22.

Lời giải.

153

Bài 23.

Lời giải.

Bài 24.

Lời giải.

154

155